Colon, Rectum and Anus Flashcards

1
Q

A 78-year old female vendor consults for a fleshy mass protruding from the anus. Inspection of the mass shows concentric ridges around the mass encircling the anal opening. This would probably be:

a. Hemorrhoids
b. Rectal Prolapse
c. Verruca vulgaris
d. Anal cancer

A

b. Rectal Prolapse

How well did you know this?
1
Not at all
2
3
4
5
Perfectly
2
Q

Proctosigmoidoscopic findings in a 37-year old male include a necrotic, fungating 2 cm mass obstructing 20% of the lumen. Mass is located 7 cm from the anal verge. Most appropriate surgical procedure for him is:

a. Abdominoperineal resection
b. Coloanal pull through
c. Hartmann’s procedure
d. Low Anterior Resection

A

d. Low Anterior Resection

How well did you know this?
1
Not at all
2
3
4
5
Perfectly
3
Q

A 25-year old school teacher is brought to the ER because of severe anal pain and bleeding of 2 days duration ff passage of hard stool. There are no gross anal lesions or visible masses. Attempts of digital examination produce severe pain, particularly at the posterior anal verge. The anal opening feels tight, and further attempts at examination are aborted because of severe pain. What is the most ideal management for the patient?

a. Lateral internal sphincterotomy
b. Hemorrhoidectomy
c. Analgesics, sitz baths and laxatives
d. Incision and drainage

A

d. Incision and drainage

How well did you know this?
1
Not at all
2
3
4
5
Perfectly
4
Q

A 66-yr old male is admitted for passage of fresh and clotted blood per rectum. For the past 6 months he has had 3 similar episodes. The most important aspect in the management of this patient is:

a. fluid resuscitation and stabilisation
b. identification of the exact cause of bleeding
c. emergency laparotomy
d. transfusion

A

b. identification of the exact cause of bleeding

How well did you know this?
1
Not at all
2
3
4
5
Perfectly
5
Q

On exploration, a 58-yr old male is found to have a carcinoma in the distal ascending colon with a 2.5 cm solitary metastatic nodule in segment 3 of the liver. The treatment of choice for this patient is:

a. right hemicolectomy with chemotherapy
b. extended right hemicolectomy with RFA
c. extended right hemicolectomy with left hepatectomy
d. right hemicolectomy with left lateral segmentectomy

A

d. right hemicolectomy with left lateral segmentectomy

How well did you know this?
1
Not at all
2
3
4
5
Perfectly
6
Q

A 23-yr old student undergoes surgery for supposed appendicitis. At surgery, he is found to have a collection of pus plastered by omentum to the area of the cecum, which is markedly edematous and friable on attempted dissection. Recommended treatment is:

a. drainage and appendectomy
b. drainage and interval appendectomy
c. right hemicolectomy
d. closure antibiotic therapy

A

b. drainage and interval appendectomy

How well did you know this?
1
Not at all
2
3
4
5
Perfectly
7
Q

A 40y/o male patient was admitted at the ER complaining of stool admixed with blood amounting to 1-2 cups per episode. He also claimed to have passage of tarry stools one week prior to admission. After resuscitation, what is the most logical thing to do?

a. Do a rectal exam to look for hemorrhoids
b. Place an nasogastric tube
c. Schedule patient for colonoscopy to see if he has colon cancer
d. Send patient to the Nuclear medicine to RBC tagging

A

b. Place an nasogastric tube

How well did you know this?
1
Not at all
2
3
4
5
Perfectly
8
Q

A 67y/o patient has been complaining left lower quadrant pain for several years which was describe to be colicky, non-radiating, bearable and is associated with constipation. Until 1 day prior to admission she complained of severe generalized abdominal pain. She underwent surgery and intraoperative finding was purulent peritonitis from a perforated diverticulitis. Base on the Hinchey classification of diverticulitis, what is the possible stage?

a. I
b. II
c. III
d. IV

A

c. III

How well did you know this?
1
Not at all
2
3
4
5
Perfectly
9
Q

A 55-year-old male presents to the emergency room (ER) with 12 hours of left lower quadrant (LLQ) pain. He has no significant past medical history and has never had a colonoscopy. His heart rate is 110 bpm and blood pressure is 120/70 mmHg. Computed tomography (CT) scan shows segmental colitis of the sigmoid colon with an associated 7 cm pelvic abscess. The patient is treated successfully with percutaneous drainage of the abscess and has resolution of his symptoms. He presents to your office for follow-up 6 weeks later. What is the next BEST step in management?

a. Elective sigmoidectomy with primary anastomosis
b. Elective sigmoidectomy with end colostomy
c. Colonoscopy
d. Observation

A

c. Colonoscopy

How well did you know this?
1
Not at all
2
3
4
5
Perfectly
10
Q

A 43-year-old female with a history of an anorectal abscess that was drained two months ago presents to your office complaining of drainage coming from an opening near her anus. Anoscopy is performed and a fistula tract is identified from the peri-rectal opening to the intersphincteric plane. The external anal sphincter does not appear to be intimately involved. What is the BEST treatment option for the patient?

a. draining seton
b. fistulotomy
c. fibrin glue application
d. sitz baths and close follow up

A

b. fistulotomy

How well did you know this?
1
Not at all
2
3
4
5
Perfectly
11
Q

A 32-year-old female with a family history of HNPCC has a polyp found in the right colon during a screening colonoscopy. Pathology reports evidence of poorly differentiated invasive adenocarcinoma in the polyp. Her recent endometrial and ovarian screening tests were negative for pathologic disease. She has three children and does not desire any further childbearing. The BEST management for this patient?

A. Repeat colonoscopy in one year

B. Right hemicolectomy

C. Total abdominal colectomy with ileorectal anastomosis

D. Total abdominal colectomy with ileorectal anastomosis with a hysterectomy and bilateral salphingo-oophorectomy

E. Total abdominal protocolectomy with J-pouch and ileoanal anastomosis

A

D. Total abdominal colectomy with ileorectal anastomosis with a hysterectomy and bilateral salphingo-oophorectomy

How well did you know this?
1
Not at all
2
3
4
5
Perfectly
12
Q

A 60-year-old female has a pedunculated villous adenoma found on routine colonoscopy. The polyp is removed and sent to pathology where invasive adenocarcinoma is found. All of the following will require a segmental resection EXCEPT:

A. Lymphovascular invasion

B. Poorly differentiated

C. A 3-mm margin

D. Invasion of the stalk base

E. A 4cm polyp

A

C. A 3-mm margin

How well did you know this?
1
Not at all
2
3
4
5
Perfectly
13
Q

A 61-year-old male is found to have rectal bleeding two days after an open AAA repair. On exam, the patient is febrile and has abdominal tenderness and distension. His WBC is elevated but remains hemodynamically stable. What is the BEST next step to manage this patient?

A. Urgent CT scan abdomen and pelvis

B. NPO, IVF, empiric antibiotics

C. Sigmoidoscopy

D. Abdominal xray

E. Emergent reexploration

A

C. Sigmoidoscopy

How well did you know this?
1
Not at all
2
3
4
5
Perfectly
14
Q

A 55-year-old male presents to the ER with 12 hours of left lower quadrant pain. He has no significant past medical history and has never had a colonoscopy. His heart rate is 110bpm and BP is 120/70mmHg. CT scan shows segmental colitis of the sigmoid colon with an associated 7cm pelvic abscess. what is his Hinchey classification?

A. Hinchey I

B. Hinchey II

C. Hinchey III

D. Hinchey IV

E. Hinchey V

A

B. Hinchey II

How well did you know this?
1
Not at all
2
3
4
5
Perfectly
15
Q

A 67-year-old male is found to have a T3N1M0 adenocarcinoma in the sigmoid colon. What is the BEST next step in management?

A. Neoadjuvant chemotherapy with 5-FU/Leucovorin followed by sigmoidectomy

B. Sigmoidectomy alone

C. Sigmoidectomy with adjuvant 5-FU/Leucovorin chemotherapy

D. Sigmoidectomy with adjuvant 5-FU/Leucovorin/Oxaliplatin chemotherapy

E. Sigmoidectomy with adjuvant 5-FU/Leucovorin/irinotecan chemotherapy

A

D. Sigmoidectomy with adjuvant 5-FU/Leucovorin/Oxaliplatin chemotherapy

How well did you know this?
1
Not at all
2
3
4
5
Perfectly
16
Q

A 12-year-old boy with a strong family history of colorectal cancer is diagnosed with familial adenomatous polyposis (FAP). Which of the following management strategies is TRUE regarding this disease?

A. Can be treated definitively with Sulindac

B. Screening in this patient involves a proctosigmoidoscopy every three years

C. The patient should have surgery before the age of twenty, regardless of the development of polyps

D. If the rectum is preserved with surgery, he will need follow up screening every five years due to the risk of rectal cancer

E. Upper endoscopy screening should start by the age of 35

A

C. The patient should have surgery before the age of twenty, regardless of the development of polyps

How well did you know this?
1
Not at all
2
3
4
5
Perfectly
17
Q

A 63-year-old female presents to your office complaining of blood in her stools. A colonoscopy is performed and a mass is identified in the rectum. All of the following are true regarding adenocarcinoma of the rectum EXCEPT:

A. Transanal excision is reserved for T1 lesions within 6cm of the anal verge

B. Abdominoperineal resection (APR) is indicated if the tumor is within 2cm of the sphincter muscles

C. Neoadjuvant chemoradiation is recommended for Stage II and higher rectal cancers

D. A total mesorectal excision for low rectal cancers improves survival but has an increased incidence of postoperative bladder dysfunction

E. Adjuvant chemotherapy guidelines are similar to that of colon cancer

A

D. A total mesorectal excision for low rectal cancers improves survival but has an increased incidence of postoperative bladder dysfunction

How well did you know this?
1
Not at all
2
3
4
5
Perfectly
18
Q

A 54-year-old female with a history of ulcerative colitis has her scheduled colonoscopy performed. Biopsies find typical mucosal inflammation with some dysplastic changes. The patient’s symptoms have been well controlled with medical management. Which of the ff describes the BEST management of this patient?

A. Repeat colonoscopy in 5 years

B. Repeat colonoscopy in 2 years

C. Total abdominal colectomy with colorectal anastomosis

D. Total proctocolectomy with end ileostomy

E. Total proctocolectomy with ileoanal anastomosis

A

E. Total proctocolectomy with ileoanal anastomosis

How well did you know this?
1
Not at all
2
3
4
5
Perfectly
19
Q

A 67-year-old male presents to the ER with bright red blood per rectum. The patient noticed the blood after defecating earlier in the day, but it has continued to bleed since the onset. On exam, the patient has no abdominal tenderness, but gross blood is noted at the rectum. His pulse is 112bpm and BP is 85/52mmHg. His Hgb is 7.3g/dL. The patient’s VS improve with 2U of blood, but he continues to have bleeding.

Which of the ff is TRUE regarding the management of this patient?

A. He should have a colonoscopy immediately.

B. A tagged RBC scan can pick up bleeding with a rate as low as 0.5ml/min

C. The best localization study in this patient is angiography

D. He should be taken to the OR emergently

E. The source is likely diverticulosis, and he should be taken for a sigmoidectomy once stabilized

A

C. The best localization study in this patient is angiography

How well did you know this?
1
Not at all
2
3
4
5
Perfectly
20
Q

A 27-year-old male presents to the primary care physician with pain on defecation and blood on the toilet paper. He typically has hard stools and has to strain. All of the following regarding his condition are true EXCEPT:

A. The usual location is in the posterior midline.

B. Digital and proctoscopic exam are necessary.

C. Frequently associated with a sentinel pile

D. Diagnosis is made with history and PE

E. Lateral locations should prompt workup for other causes

A

B. Digital and proctoscopic exam are necessary.

How well did you know this?
1
Not at all
2
3
4
5
Perfectly
21
Q

A 52-year-old male presents with anal bleeding and pruritus. A 2cm mass is noticed on exam once cm above the dentate line. A biopsy is performed which shows cloacogenic epidermoid carcinoma. Which of the following is the recommended management?

A. Wide local excision followed by chemoradiation

B. Chemotherapy alone

C. Neoadjuvant chemoradiation followed by wide local excision

D. Wide local excision

E. Nigro protocol

A

E. Nigro protocol

How well did you know this?
1
Not at all
2
3
4
5
Perfectly
22
Q

A 67-year-old male is noted to have a one cm mass at the anal margin. Biopsy shows squamous cell cancer. There is no evidence of regional lymph node spread on PE and imaging. What is the MOST appropriate management?

A. Chemotherapy alone

B. Nigro protocol

C. APR

D. Wide local excision

E. Wide local excision with regional lymphadenectomy

A

D. Wide local excision

How well did you know this?
1
Not at all
2
3
4
5
Perfectly
23
Q

A 61-year-old female patient presents with rectal bleeding. On exam, the patient has a 1cm mass in her anal canal, above the dentate line. A biopsy of the lesion is taken and the pathology notes an adenocarcinoma. Which of the following is the best treatment option for the patient?

A. Wide local excision

B. Low anterior resection

C. Nigro protocol

D. Abdominoperineal resection

E. Abdominoperineal resection with 5-FU and mitomycin

A

A. Wide local excision

How well did you know this?
1
Not at all
2
3
4
5
Perfectly
24
Q

A 43-year-old female with a history of an anorectal abscess that was drained two months ago presents to your office complaining of drainage coming from an opening near her anus. Anoscopy is performed and a fistula tract is identified from the peri-rectal opening in the intersphincteric plane. The external anal sphincter does not appear to be intimately involved. What is the BEST treatment option for the patient?

A. Draining seton

B. Cutting seton

C. Fistulotomy and curettage of the tract

D. Fibrin glue application

E. Sitz baths and close followup

A

C. Fistulotomy and curettage of the tract

How well did you know this?
1
Not at all
2
3
4
5
Perfectly
25
Q

A 57-year-old male develops urinary retention after an uncomplicated hemorrhoidectomy. A foley is placed to decompress the bladder. What is the MOST likely cause of his urinary retention?

A. Local anesthesia

B. Muscle spasms

C. Inadequate resuscitation

D. Pelvic splanchic nerve damage

E. Loss of bladder tone

A

B. Muscle spasms

How well did you know this?
1
Not at all
2
3
4
5
Perfectly
26
Q

Where does the hindgut begin and end?

A

Hindgut begins at distal third of the transverse colon and extends to the rectum

How well did you know this?
1
Not at all
2
3
4
5
Perfectly
27
Q

The hindgut relies on which artery for its blood supply?

A

Inferior mesenteric artery

How well did you know this?
1
Not at all
2
3
4
5
Perfectly
28
Q

What are the white lines of Toldt?

A

The lateral peritoneal reflections of the ascending and descending colon

How well did you know this?
1
Not at all
2
3
4
5
Perfectly
29
Q

What parts of the gastrointestinal (GI) tract do not have a serosa?

A

Esophagus, middle, and distal rectum

How well did you know this?
1
Not at all
2
3
4
5
Perfectly
30
Q

What are the major anatomic differences between the small bowel and colon?

A

The small bowel is smooth, whereas the colon has fat appendages (appendices epiploicae), haustra, and taenia coli

How well did you know this?
1
Not at all
2
3
4
5
Perfectly
31
Q

What is the arterial blood supply to the rectum?

A

Proximal: superior hemorrhoidal artery (superior rectal artery) from the inferior mesenteric artery

Middle: middle hemorrhoidal artery (middle rectal artery) from the hypogastric artery (internal iliac artery)

Distal: inferior hemorrhoidal artery (inferior rectal artery) from the pudendal artery, which is a branch of hypogastric artery (internal iliac artery)

How well did you know this?
1
Not at all
2
3
4
5
Perfectly
32
Q

What is the venous drainage of the rectum?

A

Proximal: inferior mesenteric vein that joins the splenic vein to drain into the portal vein

Middle: iliac vein into the inferior vena cava

Distal: iliac vein into inferior vena cava

How well did you know this?
1
Not at all
2
3
4
5
Perfectly
33
Q

What is the purpose of the colon?

A

Water, sodium, and bile salt absorption and stool storage

How well did you know this?
1
Not at all
2
3
4
5
Perfectly
34
Q

What is the main nutrient of a colonocyte?

A

Short-chain fatty acids (SCFAs) (butyrate)

How well did you know this?
1
Not at all
2
3
4
5
Perfectly
35
Q

How long is the rectum?

A

12 to 15 cm

How well did you know this?
1
Not at all
2
3
4
5
Perfectly
36
Q

What are the approximate proximal and distal extents of the anal canal/rectum/ rectosigmoid junction from the anal verge?

A

Anal canal: 0 to 5 cm

Rectum: 5 to 15 cm

Rectosigmoid junction: 15 to 18 cm

How well did you know this?
1
Not at all
2
3
4
5
Perfectly
37
Q

What 2 points are considered to mark the location of the rectosigmoid junction?

A

The distal point at which the taeniae converge at the level of the sacral promontory

How well did you know this?
1
Not at all
2
3
4
5
Perfectly
38
Q

What do anatomists consider the distal extent of the rectum versus what surgeons consider the distal extent of the rectum?

A

Anatomists: dentate line

Surgeons: proximal border of the anal sphincter complex

How well did you know this?
1
Not at all
2
3
4
5
Perfectly
39
Q

What is the eponym for the extension of the peritoneal cavity between the rectum and back wall of the uterus in the female human body?

A

Pouch of Douglas
(Ehrhardt-Cole recess/
rectouterine excavation/
rectouterine pouch)

How well did you know this?
1
Not at all
2
3
4
5
Perfectly
40
Q

What is the deepest point of the peritoneal cavity in women?

A

Pouch of Douglas

How well did you know this?
1
Not at all
2
3
4
5
Perfectly
41
Q

What is another name for the rectovesicular fascia in men/rectovaginal fascia in women?

A

Denonvilliers fascia

How well did you know this?
1
Not at all
2
3
4
5
Perfectly
42
Q

What is the eponym for a mass that forms in the pelvic cul-de-sac from a drop metastases from a visceral tumor that may be detected by a digital rectal examination?

A

Blumer’s shelf

How well did you know this?
1
Not at all
2
3
4
5
Perfectly
43
Q

What is the thick condensation of endopelvic fascia that connects the presacral fascia to the fascia propria at the level of S4, which then extends to the anorectal ring called?

A

Waldeyer fascia (rectosacral fascia)

How well did you know this?
1
Not at all
2
3
4
5
Perfectly
44
Q

What artery is contained within the lateral rectal stalks?

A

The middle rectal artery

How well did you know this?
1
Not at all
2
3
4
5
Perfectly
45
Q

What muscles make up the pelvic floor (pelvic diaphragm)?

A

Pubococcygeus, iliococcygeus, puborectalis (which form the levator ani)

How well did you know this?
1
Not at all
2
3
4
5
Perfectly
46
Q

Where does the pelvic floor (pelvic diaphragm) lie?

A

Between the pubis, obturator fascia, sacrum, and ischial spines

How well did you know this?
1
Not at all
2
3
4
5
Perfectly
47
Q

What artery runs close to the bowel in the mesentery as part of a vascular arcade that connects the superior mesenteric artery and inferior mesenteric artery?

A

The marginal artery of Drummond

How well did you know this?
1
Not at all
2
3
4
5
Perfectly
48
Q

What artery found low in the mesentery, near the root, is part of a vascular arcade that connects the proximal middle colic artery to the proximal inferior mesenteric artery?

A

The arc of Riolan (meandering mesenteric artery)

How well did you know this?
1
Not at all
2
3
4
5
Perfectly
49
Q

What is the most proximal branch of the inferior mesenteric artery?

A

Left colic artery

How well did you know this?
1
Not at all
2
3
4
5
Perfectly
50
Q

Describe the venous drainage of the colon and rectum:

A

Right and proximal transverse colon drain into the superior mesenteric vein, which joins with the splenic vein to become the portal vein

Distal transverse colon, descending colon, sigmoid, and most of the rectum drain into the inferior mesenteric vein, which drains into the splenic vein that joins with the superior mesenteric vein to become the portal vein

Anal canal drains by way of the middle and inferior rectal veins into the internal iliac veins, which drain into the inferior vena cava

How well did you know this?
1
Not at all
2
3
4
5
Perfectly
51
Q

What nodal chain do lymphatics from the colon and proximal two-thirds of the rectum drain into?

A

Para-aortic nodal chain

How well did you know this?
1
Not at all
2
3
4
5
Perfectly
52
Q

What nodal chains do lymphatics from the distal rectum and anal canal drain into?

A

Para-aortic nodal chain or superficial inguinal nodal chain

How well did you know this?
1
Not at all
2
3
4
5
Perfectly
53
Q

What kind of neurologic injury after rectal surgery generally results in sympathetic dysfunction characterized by retrograde ejaculation and bladder dysfunction?

A

Severing of the hypogastric nerves near the sacral promontory

How well did you know this?
1
Not at all
2
3
4
5
Perfectly
54
Q

What kind of neurologic injury after rectal surgery generally results in impotence and atonic bladder?

A

Injury to the mixed parasympathetic and sympathetic periprostatic plexus

How well did you know this?
1
Not at all
2
3
4
5
Perfectly
55
Q

What is the most prevalent species of bacteria in the colon?

A

Bacteroides species

How well did you know this?
1
Not at all
2
3
4
5
Perfectly
56
Q

What is the most common aerobe in the colon?

A

Escherichia coli

How well did you know this?
1
Not at all
2
3
4
5
Perfectly
57
Q

In what colonic segment are bacteria the most metabolically active?

A

The cecum

How well did you know this?
1
Not at all
2
3
4
5
Perfectly
58
Q

How is diarrhea defined?

A

> 3 loose stools/day

How well did you know this?
1
Not at all
2
3
4
5
Perfectly
59
Q

How is constipation defined?

A

<3 stools/wk

How well did you know this?
1
Not at all
2
3
4
5
Perfectly
60
Q

What are absolute contraindications to bowel preparation?

A

Complete bowel obstruction, free perforation

How well did you know this?
1
Not at all
2
3
4
5
Perfectly
61
Q

How are the diverticula with colonic diverticulosis formed?

A

Mucosa herniates through the colon at sites of penetration of the muscular wall by arterioles on the side of the antimesenteric taeniae

How well did you know this?
1
Not at all
2
3
4
5
Perfectly
62
Q

What area of the colon is most commonly affected by diverticula?

A

Sigmoid colon (80%)

How well did you know this?
1
Not at all
2
3
4
5
Perfectly
63
Q

What segment of the colon has the smallest intraluminal diameter?

A

Sigmoid colon

How well did you know this?
1
Not at all
2
3
4
5
Perfectly
64
Q

What is the treatment for uncomplicated diverticulitis?

A

Antibiotics on an outpatient basis; if patient has significant pain (localized peritonitis), admit to the hospital and give intravenous (IV) antibiotics for -48 hours

How well did you know this?
1
Not at all
2
3
4
5
Perfectly
65
Q

How should patients be followed after an episode of uncomplicated diverticulitis?

A

After symptoms have subsided for at least 3 weeks, a colonoscopic examination should be performed to establish the presence/location of the diverticula and to exclude cancer (mimics diverticulitis)

How well did you know this?
1
Not at all
2
3
4
5
Perfectly
66
Q

What is the approximate chance of a patient who recovered from an initial episode of uncomplicated diverticulitis developing a second attack of diverticulitis?

A

<25%

How well did you know this?
1
Not at all
2
3
4
5
Perfectly
67
Q

What is the estimated percentage of patients who recovered from an initial episode of uncomplicated diverticulitis requiring a subsequent emergency colectomy or colostomy?

A

5%

How well did you know this?
1
Not at all
2
3
4
5
Perfectly
68
Q

Although controversial, what types of patients might you offer an elective sigmoid colectomy to?

A

Young patients with an episode of diverticulitis ( <45 years old), patients with 2 episodes of diverticulitis, and immunocompromised patients

How well did you know this?
1
Not at all
2
3
4
5
Perfectly
69
Q

What is the treatment for an abscess complicating diverticulitis?

A

The preferred treatment is computed tomography (CT) or ultrasound-guided percutaneous drainage or drainage of a pelvic abscess into the rectum through a transanal approach followed by elective surgery -6 weeks after drainage of the abscess, when the patient has completely recovered from the infection, versus a more undesirable transabdominal approach by laparotomy

How well did you know this?
1
Not at all
2
3
4
5
Perfectly
70
Q

What are the 2 causes of generalized peritonitis resulting from diverticulitis?

A

A perforated diverticulum into the peritoneal cavity that is not sealed by the body’s normal defenses; an initially localized abscess that expands and suddenly bursts into the peritoneal cavity

How well did you know this?
1
Not at all
2
3
4
5
Perfectly
71
Q

What procedure would you perform for generalized peritonitis from perforated diverticulitis?

A

A Hartmann procedure: resection of the diseased sigmoid colon; creation of a descending colostomy using noninflamed tissue; closure of the divided end of the rectum with suture/ staples

How well did you know this?
1
Not at all
2
3
4
5
Perfectly
72
Q

What is the usual time period to wait before restoring intestinal continuity by reversing a Hartmann procedure for perforated diverticulitis?

A

At least 10 weeks (when patient has completely recovered from their illness)

How well did you know this?
1
Not at all
2
3
4
5
Perfectly
73
Q

What is the Hinchey classification grading system for diverticulitis?

A

Stage I: diverticulitis with associated pericolic abscess

Stage II: diverticulitis associated with distant abscess (retroperitoneal or pelvic)

Stage III: diverticulitis associated with purulent peritonitis

Stage IV: diverticulitis associated with fecal peritonitis

How well did you know this?
1
Not at all
2
3
4
5
Perfectly
74
Q

What is the most common colonic segment to be involved in a volvulus?

A

Sigmoid colon

How well did you know this?
1
Not at all
2
3
4
5
Perfectly
75
Q

What is the least common colonic segment to be involved in a volvulus?

A

Transverse colon

How well did you know this?
1
Not at all
2
3
4
5
Perfectly
76
Q

What is a cecal bascule?

A

The cecum folds anteromedial to the ascending colon from the presence of a constricting band across the ascending colon

How well did you know this?
1
Not at all
2
3
4
5
Perfectly
77
Q

What findings might you see on plain film, CT scan, and barium enema with a sigmoid volvulus?

A

Plain film: bent inner tube with apex in the right upper quadrant

CT scan: mesenteric whorl

Barium enema: bird’s beak deformity

How well did you know this?
1
Not at all
2
3
4
5
Perfectly
78
Q

What is the treatment for sigmoid volvulus?

A

Appropriate resuscitation;
decompression with placement of a soft rectal tube through the proctoscope past the twist of the volvulus and leaving the rectal tube in place.

If a rectal tube cannot be passed, detorsion of the volvulus with a colonoscope.

If unable to detorse volvulus by rectal tube or colonoscopy, perform Hartmann operation procedure.

Confirm the reduction with an abdominal radiograph, attempt a full colonoscopic examination after cleansing the bowel with cathartics, perform an elective sigmoid colon resection.

How well did you know this?
1
Not at all
2
3
4
5
Perfectly
79
Q

What is the recurrence rate for sigmoid volvulus without surgical intervention?

A

~50%

How well did you know this?
1
Not at all
2
3
4
5
Perfectly
80
Q

What is a cecal (cecocolic) vohulus?

A

An axial rotation of the terminal ileum, cecum, and ascending colon with concomitant twisting of the associated mesentery from a lack of fixation of the cecum to the retroperitoneum

How well did you know this?
1
Not at all
2
3
4
5
Perfectly
81
Q

What might you see on plain abdominal radiographs with a cecal volvulus?

A

A gas-filled comma shape with the concavity facing inferiorly and to the right (upsidedown comma sign), a circular shape with. a narrow, triangular density pointing to the right and superiorly, a dilated cecum displaced to the left side of the abdomen

How well did you know this?
1
Not at all
2
3
4
5
Perfectly
82
Q

What is the treatment for a cecal vohulus?

A

The procedure of choice is a right colectomy with primary anastomosis;

if frankly gangrenous bowel, resect right colon and create an ileostomy;

cecopexy is another option (higher recurrence rates)

How well did you know this?
1
Not at all
2
3
4
5
Perfectly
83
Q

What is the most common cause of large intestinal obstruction in the United States?

A

Colorectal cancer

How well did you know this?
1
Not at all
2
3
4
5
Perfectly
84
Q

What is the most common cause of large Intestinal obstruction in Russia, Eastern Europe and Africa?

A

Colonic volvulus (high-fiber diets)

How well did you know this?
1
Not at all
2
3
4
5
Perfectly
85
Q

What is a closed loop obstruction?

A

An obstruction featuring the occlusion of the proximal and distal parts of the bowel

How well did you know this?
1
Not at all
2
3
4
5
Perfectly
86
Q

What is pseudo-obstruction of the colon otherwise known as?

A

Ogilvie syndrome

How well did you know this?
1
Not at all
2
3
4
5
Perfectly
87
Q

What study should be performed in all stable patients with a suspected diagnosis of colonic pseudo obstruction?

A

Water-soluble contrast enema can reliably differentiate between mechanical obstruction and pseudo-obstruction; can consider colonoscopy

How well did you know this?
1
Not at all
2
3
4
5
Perfectly
88
Q

What is the treatment for Ogilvie syndrome?

A

Initial treatment with
nasogastric decompression,
fluid replacement,
correction of electrolyte abnormalities,
discontinuation of all medications that inhibit bowel motility, serial abdominal examinations and radiographs;

consider administration of 2.5 mg of neostigmine given intravenously over 3 minutes after ruling out distal obstruction with water-soluble contrast enema or colonoscopy (do not give to patients with significant cardiac history, and watch for bradycardia by monitoring patient with telemetry during administration of the drug with atropine immediately available);

can consider colonoscopic decompression, but high recurrence

How well did you know this?
1
Not at all
2
3
4
5
Perfectly
89
Q

What is the law of Laplace?

A

Tension is directly proportional to increased pressure times radius.

Stated differently, for a given pressure, increased radius requires increased wall thickness to accommodate a stable wall tension; also, increased pressure requires increased thickness to maintain a stable wall tension.

It is remembered by the following equation:

T = Yi (P x R)/t
(T =wall tension, P =pressure, R =radius, T =wall thickness)

The law explains why the cecum, with its largest diameter, is the most common site of colonic rupture secondary to distal obstruction or increased pressure (e.g., overzealous insuftlations with excessive air during colonoscopy).

How well did you know this?
1
Not at all
2
3
4
5
Perfectly
90
Q

What layers of the bowel wall are affected by ulcerative colitis?

A

Mucosa and submucosa

How well did you know this?
1
Not at all
2
3
4
5
Perfectly
91
Q

What is the sine qua non of ulcerative colitis?

A

Rectal involvement (proctitis)

How well did you know this?
1
Not at all
2
3
4
5
Perfectly
92
Q

What diagnostic characteristics of ulcerative colitis help differentiate it from Crohn disease?

A

Continuous, uninterrupted inflammation of the colonic mucosa beginning in the distal rectum and extending proximally

Crohn disease features normal segments of colon [skipped areas] interspersed between distinct segments of colonic inflammation),

ulcerative colitis does not involve the terminal ileum, except with backwash ileitis;

ulcerative colitis affects the mucosa and submucosa, where Crohn disease can have transmural involvement.

How well did you know this?
1
Not at all
2
3
4
5
Perfectly
93
Q

What 3 features are suggestive of a malignant stricture in the setting of ulcerative colitis?

A

Stricture appears later in the course of ulcerative colitis;

stricture is proximal to the splenic flexure and causes large-bowel obstruction

How well did you know this?
1
Not at all
2
3
4
5
Perfectly
94
Q

What are the extraintestinal manifestations of ulcerative colitis?

A

Arthritis, ankylosing spondylitis, erythema nodosum, pyodenna gangrenosum, primary sclerosing cholangitis

How well did you know this?
1
Not at all
2
3
4
5
Perfectly
95
Q

What extraintestinal manifestation of ulcerative colitis does not improve with colectomy?

A

Primary sclerosing cholangitis, ankylosing spondylitis

How well did you know this?
1
Not at all
2
3
4
5
Perfectly
96
Q

What are the American Cancer Society guidelines regarding surveillance colonoscopy for ulcerative colitis patients?

A

Colonoscopy every 1 to 2 years beginning 8 years after the onset of pancolitis 12 to 15 years after the onset of left-sided colitis

How well did you know this?
1
Not at all
2
3
4
5
Perfectly
97
Q

What classes of drugs are used in the treatment of ulcerative colitis?

A

Aminosalicylates (sulfasalazine, mesalamine)

Corticosteroids (prednisone)

Immunosuppressive agents ( azathioprine, cyclosporine)

How well did you know this?
1
Not at all
2
3
4
5
Perfectly
98
Q

What are indications for surgery in a patient with ulcerative colitis?

A
Dysplasia carcinoma, 
long-standing disease, 
intractability, 
massive colonic bleeding, 
toxic megacolon (may consider surgery for malnutrition/growth retardation in pediatric/ adolescent patients), 
persistent obstruction or stricture, 
perforation
How well did you know this?
1
Not at all
2
3
4
5
Perfectly
99
Q

What is the treatment for toxic megacolon?

A

Nasogastric decompression; IV hydration (with consideration given to IV hyperalimentation depending on length of illness before fulminant episode and patient’s nutritional status); broad-spectrum IV antibiotics; high-dose IV steroids if patient is steroid-dependent; follow with serial abdominal exams and leukocyte counts; if deteriorates or if lack of improvement with medical therapy for 24 to 48 hours, perform an urgent procedure

How well did you know this?
1
Not at all
2
3
4
5
Perfectly
100
Q

What is the preferred operation for toxic megacolon?

A

Total abdominal colectomy with ileostomy and preservation of the rectum

How well did you know this?
1
Not at all
2
3
4
5
Perfectly
101
Q

What is the procedure of choice in a patient with massive hemorrhage from ulcerative colitis?

A

Subtotal colectomy; if severe bleeding from rectal mucosa, may also require emergency proctectomy

How well did you know this?
1
Not at all
2
3
4
5
Perfectly
102
Q

What are your options for elective surgery in patients with ulcerative colitis?

A

Total proctocolectomy with ileostomy; restorative proctocolectomy with ileal pouch anal anastomosis;

total proctocolectomy with a continent ileal reservoir (Kock pouch)

How well did you know this?
1
Not at all
2
3
4
5
Perfectly
103
Q

What are indications for surgery in a patient with Crohn disease?

A
Cancer Fistula
Growth retardation 
Intraabdominal abscess 
Intractability 
Toxic megacolon 
Massive bleeding 
Intestinal obstruction
How well did you know this?
1
Not at all
2
3
4
5
Perfectly
104
Q

What operation would you perform for massive bleeding from Crohn disease?

A

Abdominal colectomy and ileostomy versus ileorectal anastomosis if the rectum is not inflamed

How well did you know this?
1
Not at all
2
3
4
5
Perfectly
105
Q

What organisms should be evaluated for in a stool sample from a patient with suspected infectious colitis?

A

Campylobacter jejuni, Yersinia enterocolitica, Salmonella typhi, Clostridium difficile

How well did you know this?
1
Not at all
2
3
4
5
Perfectly
106
Q

What is the most common form of intestinal ischemia?

A

Colonic ischemia

How well did you know this?
1
Not at all
2
3
4
5
Perfectly
107
Q

What sign may be seen on radiographic imaging with intestinal wall edema or submucosal hemorrhage?

A

Thumb printing

How well did you know this?
1
Not at all
2
3
4
5
Perfectly
108
Q

Which colonic segment is the most prone to ischemia?

A

Splenic flexure and rectosigmoid junction (Griffith and Sudeck point)

How well did you know this?
1
Not at all
2
3
4
5
Perfectly
109
Q

What are indications for surgery with acute colonic ischemia?

A

Peritoneal signs, massive bleeding, universal fulminant colitis with or without toxic megacolon

How well did you know this?
1
Not at all
2
3
4
5
Perfectly
110
Q

What are the Amsterdam criteria for hereditary nonpolyposis colorectal cancer (HNPCC)?

A

3-2-1:

At least 3 first-degree relatives with colon cancer and all of the following:

  • 2 successive generations affected
  • At least 1 case of colon cancer diagnosed before age 50 years; familial adenomatous polyposis excluded
How well did you know this?
1
Not at all
2
3
4
5
Perfectly
111
Q

What is the difference between the criteria and the modified Amsterdam criteria?

A

The modified Amsterdam criteria is almost the same as the Amsterdam criteria, except that with the modified Amsterdam criteria the cancer must be associated with HNPCC (colon, endometrium, small bowel, ureter, renal pelvis) instead of specifically colon cancer

How well did you know this?
1
Not at all
2
3
4
5
Perfectly
112
Q

What are the screening recommendations for patients with HNPCC?

A

Colonoscopy at age 20 to 25 years and repeat every 1 to 3 years

Transvaginal ultrasound or endometrial aspirate at age 20 to 25 years and repeat annually

How well did you know this?
1
Not at all
2
3
4
5
Perfectly
113
Q

What is the mainstay of the diagnosis of HNPCC?

A

A detailed family history

How well did you know this?
1
Not at all
2
3
4
5
Perfectly
114
Q

What is the procedure of choice when colon cancer is detected in a patient with HNPCC?

A

Abdominal colectomy with ileorectal anastomosis

If the patient is female with no further plans for childbearing, prophylactic total abdominal hysterectomy and bilateral salpingo-oophorectomy are recommended

How well did you know this?
1
Not at all
2
3
4
5
Perfectly
115
Q

What are the screening recommendations for patients with familial adenomatous polyposis(FAP)/Gardner syndromef?

A

Flexible proctosigmoidoscopy at age 10 to 12 years and repeat every 1 to 2 years until age 35;

after age 35 repeat every 3 years

Upper GI endoscopy every 1 to 3 years starting when polyps are first identified.

How well did you know this?
1
Not at all
2
3
4
5
Perfectly
116
Q

What is the most commonly recommended procedure for the treatment of FAP syndrome?

A

Restorative proctocolectomy with ileal pouch anal anastomosis with distal rectal mucosectomy

Alternative includes total abdominal colectomy with ileorectal anastomosis.

How well did you know this?
1
Not at all
2
3
4
5
Perfectly
117
Q

What percentage of patients with the adenomatous polyposis coli (APC) mutation express the gene?

A

100%

How well did you know this?
1
Not at all
2
3
4
5
Perfectly
118
Q

Do you have to worry about duodenal and ampullary polyps found in a patient with FAP syndrome?

A

Yes, because the duodenal and ampullary polyps are usually neoplastic

All large polyps should be removed with endoscopic polypectomy, and pancreatoduodenectomy (Whipple procedure) is indicated with ampullary cancer discovered at an early stage.

How well did you know this?
1
Not at all
2
3
4
5
Perfectly
119
Q

How many juvenile polyps need to be present to make the diagnosis of familial juvenile polyposis?

A

10 juvenile polyps

How well did you know this?
1
Not at all
2
3
4
5
Perfectly
120
Q

What are the most common colonic polyps?

A

Hyperplastic polyps

How well did you know this?
1
Not at all
2
3
4
5
Perfectly
121
Q

Are hyperplastic polyps considered to have malignant potential?

A

No, but adenomatous changes can be found in hyperplastic polyps

How well did you know this?
1
Not at all
2
3
4
5
Perfectly
122
Q

What autosomal dominant syndrome is characterized by a combination of hamartomatous polyps of the intestinal tract and hyperpigmentation of the buccal mucosa, lips, and digits?

A

Peutz-Jeghers syndrome

How well did you know this?
1
Not at all
2
3
4
5
Perfectly
123
Q

True or False: patients with Peutz-Jeghers syndrome are not at increased risk for the development of cancer.

A

False; Peutz-Jeghers syndrome is associated with an increased risk for cancer throughout the intestinal tract, from the stomach to the rectum, and extraintestinal malignancies (breast, ovary, cervix, fallopian tubes, thyroid, lung, gallbladder, bile ducts, pancreas, testicles)

How well did you know this?
1
Not at all
2
3
4
5
Perfectly
124
Q

What is the approximate lifetime risk of colon cancer in the general U.S. population?

A

6%

How well did you know this?
1
Not at all
2
3
4
5
Perfectly
125
Q

On what chromosome is the APC gene located?

A

Chromosome 5q21

How well did you know this?
1
Not at all
2
3
4
5
Perfectly
126
Q

What kind of gene is the APC gene?

A

Tumor suppressor gene

How well did you know this?
1
Not at all
2
3
4
5
Perfectly
127
Q

What is the most frequently mutated tumor suppressor gene in human neoplasia?

A

p53

How well did you know this?
1
Not at all
2
3
4
5
Perfectly
128
Q

On what chromosome can p53 be found?

A

Chromosome 17p

How well did you know this?
1
Not at all
2
3
4
5
Perfectly
129
Q

On which chromosome can the ras proto-oncogene be found?

A

Chromosome 12

How well did you know this?
1
Not at all
2
3
4
5
Perfectly
130
Q

What do right-sided colon cancers tend to do?

A

Bleed

How well did you know this?
1
Not at all
2
3
4
5
Perfectly
131
Q

What do left-sided colon cancers tend to do?

A

Obstruct

How well did you know this?
1
Not at all
2
3
4
5
Perfectly
132
Q

What is the gold standard for establishing the diagnosis of colon cancer?

A

Colonoscopy

How well did you know this?
1
Not at all
2
3
4
5
Perfectly
133
Q

In patients with colon cancers causing a complete obstruction, what study can you order to establish the anatomic level of the obstruction when colonoscopy cannot be performed?

A

A water-soluble contrast enema

How well did you know this?
1
Not at all
2
3
4
5
Perfectly
134
Q

What is the treatment for an obstructing cancer of the proximal colon?

A

Right colectomy with primary anastomosis between the ileum and the transverse colon

How well did you know this?
1
Not at all
2
3
4
5
Perfectly
135
Q

What is the procedure of choice for a colon tumor involving the cecum, ascending colon, or the hepatic flexure?

A

Right hemicolectomy

How well did you know this?
1
Not at all
2
3
4
5
Perfectly
136
Q

What is the procedure of choice for most transverse colon tumors?

A

Extended right hemicolectomy

How well did you know this?
1
Not at all
2
3
4
5
Perfectly
137
Q

What is the procedure of choice for a descending colon tumor?

A

Left hemicolectomy

How well did you know this?
1
Not at all
2
3
4
5
Perfectly
138
Q

What is the procedure of choice for a sigmoid colon tumor?

A

Sigmoidectomy

How well did you know this?
1
Not at all
2
3
4
5
Perfectly
139
Q

When is abdominal colectomy for colon cancer indicated?

A

Patients with multiple primary tumors Individuals with HNPCC

Occasionally for patients with completely obstructing sigmoid cancers

How well did you know this?
1
Not at all
2
3
4
5
Perfectly
140
Q

According to the AJCC TNM Staging System for Colorectal Cancer, what is a Tis lesion?

A

Carcinoma in situ: intraepithelial or invasion of lamina propria

How well did you know this?
1
Not at all
2
3
4
5
Perfectly
141
Q

According to the AJCC TNM Staging System for Coloredal Cancer, what is a T1 lesion?

A

Tumor invades submucosa

How well did you know this?
1
Not at all
2
3
4
5
Perfectly
142
Q

According to the AJCC TNM Staging System for Colorectal Cancer, what is a T2 lesion?

A

Tumor invades muscularis propria

How well did you know this?
1
Not at all
2
3
4
5
Perfectly
143
Q

According to the AJCC TNM Staging System for Coloredal Cancer, what is a T3 lesion?

A

Tumor invades through the muscularis propria into the subserosa or into nonperitonealized pericolic or perirectal tissues

How well did you know this?
1
Not at all
2
3
4
5
Perfectly
144
Q

According to the AJCC TNM Staging System for Coloredal Cancer, what is a T4 lesion?

A

Tumor directly invades other organs or structures and/or perforates the visceral peritoneum

How well did you know this?
1
Not at all
2
3
4
5
Perfectly
145
Q

According to the AJCC TNM Staging System for Colorectal Cancer, what is N0 nodal status?

A

No regional lymph node metastasis

How well did you know this?
1
Not at all
2
3
4
5
Perfectly
146
Q

According to the AJCC TNM Staging System for Colorectal Cancer, what is Nl nodal status?

A

Metastasis in 1 to 3 regional lymph nodes

How well did you know this?
1
Not at all
2
3
4
5
Perfectly
147
Q

According to the AJCC TNM Staging System for Colorectal Cancer, what is N2 nodal status?

A

Metastasis in 4 or more regional lymph nodes

How well did you know this?
1
Not at all
2
3
4
5
Perfectly
148
Q

According to the AJCC TNM Staging System for Coloredal Cancer, what does M0 mean?

A

No distant metastasis

How well did you know this?
1
Not at all
2
3
4
5
Perfectly
149
Q

According to the AJCC TNM Staging System for Colorectal Cancer, what does M1 mean?

A

Distant metastasis

How well did you know this?
1
Not at all
2
3
4
5
Perfectly
150
Q

According to the AJCC TNM Staging System for Colorectal Cancer, what is stage 0 colon cancer?

A

Tis, N0, M0

How well did you know this?
1
Not at all
2
3
4
5
Perfectly
151
Q

According to the AJCC TNM Staging System for Colorectal Cancer, what is a stage 1 colon cancer?

A

Tl, NO, MO; T2, NO, MO

How well did you know this?
1
Not at all
2
3
4
5
Perfectly
152
Q

According to the AJCC TNM Staging System for Colorectal Cancer, what is a stage 2A colon cancer?

A

T3,NO,MO

How well did you know this?
1
Not at all
2
3
4
5
Perfectly
153
Q

According to the AJCC TNM Staging System for Colorectal Cancer, what is a stage 2B colon cancer?

A

T4,NO,MO

How well did you know this?
1
Not at all
2
3
4
5
Perfectly
154
Q

According to the AJCC TNM Staging System for Colorectal Cancer, what is a stage 3A colon cancer?

A

Tl to T2, N1, MO

How well did you know this?
1
Not at all
2
3
4
5
Perfectly
155
Q

According to the AJCC TNM Staging System for Colorectal Cancer, what is a stage 3B colon cancer?

A

T3 to T4, N1, MO

How well did you know this?
1
Not at all
2
3
4
5
Perfectly
156
Q

According to the AJCC TNM Staging System for Colorectal Cancer, what is a stage 3C colon cancer?

A

AnyT,N2,MO

How well did you know this?
1
Not at all
2
3
4
5
Perfectly
157
Q

According to the AJCC TNM Staging System for Colorectal Cancer, what is a stage 4 colon cancer?

A

Any T, any N, Ml

How well did you know this?
1
Not at all
2
3
4
5
Perfectly
158
Q

What is the 5-year survival rate for patients who undergo appropriate resection of a T stage 1 colon cancer?

A

95%

How well did you know this?
1
Not at all
2
3
4
5
Perfectly
159
Q

What is the 5-year survival rate for a patient with stage 2 colon cancer treated by appropriate surgical resection?

A

80%

How well did you know this?
1
Not at all
2
3
4
5
Perfectly
160
Q

What is the survival rate for stage 3 cancer treated by surgery alone?

A

65%

How well did you know this?
1
Not at all
2
3
4
5
Perfectly
161
Q

What is the 5-year survival rate for a patient with colon cancer with distant metastatic disease (stage 4)?

A

<10%

How well did you know this?
1
Not at all
2
3
4
5
Perfectly
162
Q

What would you do if you found an isolated hepatic or pulmonary lesion while performing a right colectomy for colon cancer?

A

Attempt to resect the isolated hepatic/pulmonary metastasis

How well did you know this?
1
Not at all
2
3
4
5
Perfectly
163
Q

What stage of colon cancer shows a clear benefit with chemotherapy?

A

Stage 3 colon cancer

How well did you know this?
1
Not at all
2
3
4
5
Perfectly
164
Q

What is FOLFOX?

A

Oxaliplatin, 5-fluorouracil, leucovorin regimen

How well did you know this?
1
Not at all
2
3
4
5
Perfectly
165
Q

What is Xeloda?

A

Capecitabine (an oral fluoropyrimidine)

How well did you know this?
1
Not at all
2
3
4
5
Perfectly
166
Q

What is bevacizumab (Avastin)?

A

A monoclonal antibody that is a vascular endothelial growth factor inhibitor

How well did you know this?
1
Not at all
2
3
4
5
Perfectly
167
Q

What is cetuximab (Erbitux)?

A

A monoclonal antibody that binds to and inhibits the epidermal growth factor receptor (EGFR)

How well did you know this?
1
Not at all
2
3
4
5
Perfectly
168
Q

What percentage of colon cancer recurrences is detected within 2 years of the time of resection?

A

85%

How well did you know this?
1
Not at all
2
3
4
5
Perfectly
169
Q

Risk factors associated with colorectal cancer include which one of the following?

A. Irritable bowel syndrome

B. Low intake of dietary fiber

C. Low intake of red meat

D. Chronic aspirin therapy

A

Answer: B. Inflammatory bowel disease is associated with an increased risk of colorectal cancer, but there is no association with irritable bowel syndrome. Other factors associated with increased risk are a low intake of dietary fiber and a high intake of red meat. Risk has been shown to be reduced by regular ingestion of aspirin.

How well did you know this?
1
Not at all
2
3
4
5
Perfectly
170
Q

Surgery for liver metastases from colorectal cancer is:

A. Associated with a 10% mortality

B. Contraindicated if more than 1 lesion is present

C. Associated with 16% to 40% 5-year survival

D. Usually followed with consolidation radiotherapy

A

Answer: C. Surgery for liver metastases from colorectal cancer is associated with a 2% mortality, can be performed if more than I lesion is present, and is associated with 16% to 40% 5-year survival. Radiotherapy is not used for liver metastases, as the toxicity to normal liver tissue would be too severe.

How well did you know this?
1
Not at all
2
3
4
5
Perfectly
171
Q

What is bevadzumab?

A. A monoclonal antibody that targets vascular endothelial growth factor (VEGF)

B. A monoclonal antibody that targets EGFR

C. A drug that has no proven survival benefit in the metastatic setting

D. A standard treatment for metastatic colorectal cancer in the UK

A

Answer: A. Bevadzumab is a monoclonal antibody that targets VEGF. It has been shown to improve survival in the metastatic setting when used in combination with chemotherapy.

172
Q

Which statement about ulcerative colitis with malignancy is true?

A. It has a better prognosis

B. It is related to disease activity

C. It is related to the duration of ulcerative colitis

D. Malignancy is greater in anorectal ulcerative colitis

A

Answer: C.

Carcinoma of the colon afilicts patients with ulcerative colitis 7 to 30 times more frequently than it does the general population.

The risk of colon cancer in ulcerative colitis is related to 2 factors: (1) duration of the colitis and (2) extent of colonic involvement

The risk of colon cancer for patients who have had the disease less than 10 years is low, but this risk steadily increases.

The cancer risk for patients who have had disease activity for 10 to 20 years is 23 times that of the general population, while disease duration of more than 20 years is associated with a cancer risk 32 times greater than that of the general population.

The extent of colonic involvement in colitis also influences the risk of cancer. The incidence of cancer when ulcerative colitis is limited to the rectum or to the left side of the colon is much lower than when ulcerative colitis involves the entire colon.

The colonic malignancy associated with ulcerative colitis is generally an adenocarcinoma evenly scattered throughout the colon. The adenocarcinoma is often flatter than cancers in the general population and has fewer overhanging margins. It is generally considered extremely aggressive.

173
Q

Which of the following is false about the physiology of the colon?

A. Colectomy increases Na requirement by 80 to 100 mEq/day

B. SCFAs are produced by bacteria from sugar

C. SCFAs have nutrient value for colonocytes

D. Motility of colon is 8 to 12 cm/h

E. Colon absorbs Na and water

A

Answer: D.

In the proximal colon bacteria ferments organic carbohydrates to the SCFAs acetate, propionate, and butyrate.

These SCFAs augment Na, Cl, and water absorption and are the fuel for colonocytes.

They also regulate proliferation, differentiation gene expression, and immune function in the colon.

Three types of electrical activity in colonic circular smooth muscle are slow wave, 2 to 4/ min; membranous potential oscillations (MPOs), 18/min; and action potential superimposed on slow waves and MPO.

In humans colonic motor activity varies from quiescence to isolated contractions, bursts of contractions, and propagated contractions.

Irregular contractions are predominant. Propagated contractions are low (5-40 mm Hg) and high (75 mm Hg).

High propagated contractions occur 6 times/ day originating in the cecum, are responsible for mass movement, and occur more while awake and after meals

174
Q

What is the second most common cause of death in patients with FAP?

A. Adenocarcinoma of the colon

B. Adenocarcinoma of the stomach

C. Desmoid tumors

D. Adenocarcinoma of the ovaries

E. Adenocarcinoma of the duodenum

A

Answer: E.

Colonic adenocarcinoma is the primary cause of death in patients with FAP. If colectomy is not performed, 100% of these patients will progress to cancer of the colon.

The second most common cause of death is duodenal adenocarcinoma.

These patients develop periampullary and duodenal adenomatous polyps, which present 10 to 15 years after the colonic polyps and will also progress to adenocarcinoma.

Surveillance endoscopy is required every 1 to 3 years to excise or fulgurate these polyps.

The third most common cause of death in FAP patients is development of desmoid tumors, which can locally expand and involve mesenteric, vascular, or urologic structures, making surgery difficult or impossible

175
Q

What is the most common site of involvement in patients with Crohn disease?

A. Stomach

B. Isolated small bowel

C. lleocecal

D. Colon

E. Anus

A

Answer: C. Ileocecal Crohn disease represents the most common site of presentation, occurring in -40% to 50% of patients. This is followed by colonic disease (which can occur in any segment of the colon) in -30% of patients.

Crohn disease isolated to the small bowel (not including the terminal ileum) occurs in approximately another 30%. Oral, gastric, and duodenal diseases are thought to be histologically present in 20% to 40% of patients, but are only symptomatic in about 4%. Perianal involvement occurs in -15% of patients and presents as abscesses and fistulas.

176
Q

Which of the following is not considered a tenet of colon cancer resection?

A. Proximal and distal resection margins of 5 cm

B. High ligation of the mesentery and vascular structures

C. Negative radial margins of resection

D. Sentinel lymph node assessment prior to mesenteric resection

E. More than 12 lymph nodes in the resected specimen

A

Answer: D. Studies have shown that the extent of mural spread from colon cancer is approximately 2 cm.

These findings have led to the convention that 5 cm are appropriate proximal and distal margins for colon cancer resection. In patients with mid-to-low rectal cancers, a 2-cm distal margin is considered acceptable if it results in sphincter preservation.

High ligation of the main feeding vascular pedicle and mesentery is done because this is the drainage pattern for venous and lymphatic flow from the tumor. Negative radial margins are required to prevent local recurrence of cancer.

This means that involved structures should be resected en bloc if possible. Twelve lymph nodes within a specimen are considered the appropriate number to allow adequate pathologic staging of a tumor. Sentinel lymph node assessment and/or biopsy in colorectal cancer has not been adopted as the standard of care.

177
Q

Which of the following patients is at an average risk of colorectal cancer?

A. A 45-year-old female with an aunt and cousin with colorectal cancer

B. A 35-year-old female with ulcerative colitis diagnosed at age 25

C. A 75-year-old male with a history of recurrent diverticulitis

D. A 60-year-old male whose grandfather was diagnosed with colon cancer at age 60

E. A 25-year-old male whose older brother had hundreds of polyps on colonoscopy

A

Answer: C.

Screening and surveillance for colorectal cancer are based on a patient’s risk factors.

A personal history of adenomas or colorectal cancer, a family history of adenomas or colorectal cancer in I first-degree relative or 2 second-degree relatives, hereditary colon cancer syndromes, or IBD places a person at increased risk.

Age by itself and diverticulitis do not increase the risk of colorectal cancer above that of the average population.

Actual screening and surveillance recommendations vary by organization and have been put forth by the U.S. Preventive Services Task Force, the NCCN, and the American College ofGastroenterology, among others.

178
Q

Which of the following statements regarding C. difficile colitis is false?

A. Endoscopy may demonstrate pseudomembranes on colonoscopy

B. PO vancomycin is a therapeutic option

C. N or PO metronidazole is a therapeutic option

D. Gel or foam hand cleansing is indicated after contact with a C. difficile-positive patient

E. Symptoms are caused by the toxin released with bacterial overgrowth

A

Answer: D. Pseudomembranous plaques are the classic colonoscopic findings in C. difficile colitis, although inflammation or even normal mucosa can be seen.

Patients may present with a variety of clinical signs and symptoms, including but not limited to abdominal pain, diarrhea, or sepsis after even a single dose of antibiotics, and a high clinical suspicion is necessary.

Clindamycin and cephalosporins are common culprits.

Treatment options include IV and oral metronidazole, oral vancomycin, and vancomycin enemas. Gels and foams are not sufficient for hand cleansing after coming in contact with C. difficile spores. Washing the hands with soap and water for at least 60 seconds is necessary, and rooms and objects must be disinfected with bleach.

179
Q
  1. A 74-year-old male presents to the ED stating that he has been constipated for the past week; he is found to have a palpable obstructing mass on DRE, and CT scan shows a large-bowel obstruction with a cecum measuring 15 cm. What is the most appropriate treatment?

A. Place a nasogastric tube for decompression

B. Consult gastroenterology for stent placement

C. Take the patient to the operating room for a diverting loop ileostomy

D. Take the patient to the operating room for diverting loop colostomy

E. Take the patient to the operating room for an emergent abdominoperineal resection

A

Answer: D. This patient has a distal obstructing rectal tumor and needs surgery to relieve his obstruction. A stent would not work due to the close proximity to the patient’s anus. A loop ileostomy would be incorrect because it would create a closed loop obstruction as long as his ileocecal valve is competent.

180
Q

A 35-year-old male with ulcerative colitis that has been well controlled with medication is found to have high-grade dysplasia in the sigmoid colon on routine colonoscopy. What is the next appropriate step?

A. Optimize medical management and repeat colonoscopy in 6 months

B. Repeat colonoscopy in 3 months

C. Perform a segmental colon resection

D. Proctocolectomy with ileal-anal pouch anastomosis

E. Total abdominal colectomy with end ileostomy

A

Answer: D. The patient has ulcerative colitis and is now found to have dysplasia on colonoscopy. This is an indication for operative management. In an elective setting it is possible to create a restorative proctocolectomy with a Jpouch anastomosis. In an emergent setting, a total abdominal colectomy with end ileostomy would be an appropriate option. It is not appropriate to perform a segmental resection due to the increased risk for colon cancer in patients with ulcerative colitis.

181
Q

What is the best imaging modality to stage rectal cancer?

A. CT scan with IV and rectal contrast

B. Flexible sigmoidoscopy

C. MRI

D. Endoscopic ultrasound

E. Barium enema

A

Answer: C. The best imaging modality for rectal cancer has now become MRI. This allows to evaluate for depth of invasion, proximity to the sphincter muscles, and if there are any abnormal lymph nodes in the mesorectal fat.

182
Q

A 25-year-old female underwent a laparoscopic appendectomy and was found to have a 1.5-cm appendiceal adenocarcinoma at the tip of the appendix. What is the next step in management?

A. Continue observation

B. Colonoscopy in 5 years

C. Right hemicolectomy

D. Ileocectomy

E. Total abdominal colectomy with end ileostomy

A

Answer: C. This patient was found to have adenocarcinoma of the appendix, which will require a formal resection. Carcinoid tumor of the appendix is the only mass that does not require further resection if it is less than 2 cm and is far from the base of the appendix. A right hemicolectomy is recommended for any noncarcinoid tumor greater than 1 cm.

183
Q

A 65-year-old male presents from a psychiatric facility with sudden onset of abdominal distension and abdominal pain. On x-ray the patient’s colon is dilated with a bent appearance, with the apex pointing to the left upper quadrant. What is the next step in operative management of this patient?

A. Place a nasogastric tube for decompression

B. Consult gastroenterology for endoscopic detorsion

C. Laparoscopic lysis of adhesions

D. Right hemicolectomy

E. Neostigmine

A

Answer: D. The description of this patient’s presentation and imaging findings suggest a cecal volvulus. There have been limited reports of detorsion with a colonoscope, and the majority of these patients require a right hemicolectomy. This condition occurs due to a lack of retroperitoneal attachments. If there is gangrenous bowel at the time of operation, an end ileostomy can be created. Cecopexy, or fixation of the cecum, is oflimited benefit due to a high recurrence rate.

184
Q

A 77-year-old man is admitted with bright red blood per rectum.

He takes warfarin for atrial fibrillation and has hypertension, aortic stenosis, and chronic obstructive pulmonary disease. The patient’s initial hemoglobin upon presentation is 6.7 g/dL, and his international normalized ratio (INR) is 2.2. Initially, his blood pressure is 86/56, which improves to 112/74 with infusion of two units of packed red blood cells (RBCs), four units of fresh frozen plasma, and crystalloids. A nasogastric tube is placed, and lavage reveals bilious contents and no blood. Rectal examination reveals gross blood in the rectal vault but no hemorrhoids or fissures. Colonoscopy reveals blood throughout the colon but no identifiable source. What should be the next step in management?

A. Tagged RBC scan

B. H2 blocker

C. Mesenteric angiography

D. Exploratory laparoscopy

E. Total abdominal colectomy

A

The workup of this patient has revealed probable lower gastrointestinal (GI) bleeding given the negative nasogastric lavage and the presence of blood throughout the colon on colonoscopy.

The source of this bleed, however, has not yet been localized. Fifteen percent of cases of upper GI bleeding may present as lower GI bleeding because of rapid transit through the GI tract.

As is often the case with GI bleeding, this patient has a very high perioperative risk for an acute coronary event and even death.

A tagged RBC scan is often helpful in localizing the region of slower or recurrent bleeding, but it is not therapeutic for an unstable patient who continues to bleed. Suppression of gastric acid secretion with an H2 blocker or a proton pump inhibitor (PPI) is not indicated in this setting.

Mesenteric angiography would be the next step in managing this patient since it can be both diagnostic and therapeutic. Mesenteric angiography can pinpoint the site of active bleeding that may be treated with superselective embolization. Although a concern in the past, the risk of intestinal ischemia is low with this approach.

Whenever possible, the site or at least the region of intestinal bleeding should be identified prior to surgery. Laparoscopy is not useful in identifying the source of bleeding. Emergency total abdominal colectomy is rarely necessary and should be a last resort since it is associated with significant morbidity and mortality in elderly and vasculopathic patients

185
Q

With regard to the anatomy of the colon and rectum, which of the following statements is true?

A. The colon has a complete outer longitudinal and an incomplete inner circular muscle layer.

B. The haustra are separated by plicae circulares.

C. The ascending colon and descending colon are usually fixed to the retroperitoneum.

D. The rectum is totally invested by three complete muscle layers.

E. The distal part of the rectum begins at the point where the taeniae coli merge.

A

ANSWER: C

COMMENTS: A thorough understanding of anatomy is integral to the surgical management of problems of the colon and rectum.

The colon has two muscle layers: an outer longitudinal layer and an inner circular layer. The inner layer completely encircles the colon.

The outer layer, unlike in the small intestine, is in the form of three grossly recognizable longitudinal strips, or taeniae coli, that do not cover the full circumference of the colon.

At the rectosigmoid junction, the three taeniae coli spread out and fuse together to cover the rectum with a longitudinal coat. This explains why acquired diverticula do not form in the rectum.

The plicae semilunares are spaced, transverse, crescentic folds that separate the tissue between the taeniae coli and form haustra. They produce a characteristic, intermittently bulging pattern that radiologically permits differentiation of the colon from the small intestine, which has circular mucosal folds known as plicae circulares or valvulae conniventes. In contrast to the plicae semilunares, the plicae circulares go all the way around the small bowel lumen, thereby facilitating radiographic distinction.

Usually, the ascending and descending portions of the colon are fused to the retroperitoneum, whereas the transverse and sigmoid portions are free. However, developmental anomalies of fixation, as seen with malrotation and in some cases of volvulus, are not uncommon. Cecal volvulus, for example, could not occur unless incomplete fixation to the retroperitoneum makes it possible for a mobile cecum to rotate around a narrow mesenteric pedicle.

Surgeons have traditionally placed the upper border of the rectum at the peritoneal reflection, but this is an external and unreliable landmark. Indeed, the peritoneum may only cover the upper anterior portion of the rectum or may go all the way down to the pelvic floor, creating a deep cu-de-sac. A better definition is the point at which the taeniae have completely merged. The rectum therefore lacks taeniae and appendices epiploicae. This is generally easy to recognize intraoperatively.

Posteriorly, the upper rectal mesentery may be narrow since it is a continuation of the sigmoid mesentery and derived from the inferior mesenteric vessels. The rectal mesentery spreads out laterally as it descends and then narrows and thins out as it reaches the anorectal ring. All of this must be removed when operating for middle and lower rectal cancers, the so-called total mesorectal excision (TME).

186
Q

A 22-year-old man in whom Crohn’s disease has recently been diagnosed has just recovered from his first Crohn’s flare-up. Currently, he has no perianal involvement. What medical therapy is not used as a first-line agent to maintain remission?

A. 6-Mercaptopurine

B. Metronidazole

C. Mesalamine

D. Infliximab

E. Prednisone

A

ANSWER: E

COMMENTS: Currently, there is no cure for Crohn’s disease. It is felt to be a systemic autoimmune disease. There is early evidence of genetic predisposition and a weak inheritance pattern, but this seems to require an additional push for the patient to develop the clinical picture of Crohn’s disease (the two-hit theory). This push might be an infection or exposure to an allergen to which the immune system responds.

Normally, the immune response will decrease after the inciting event passes and will return to homeostasis. In Crohn’s disease, this ability to downregulate seems to be abnormal and inadequate, leading to persistent inflammation. Medical therapy for Crohn’s disease is aimed at general or targeted control of the immune system and inflammation. Most medications such as steroids and many of the older immunosuppressants like azathioprine, 6-mercaptopurine, and methotrexate have broad effects on the immune system and many have deleterious side effects, such as increased risk of infections. Steroids such as prednisone are well known for their very high complication rates when used for the long term, even in the young. These include osteoporosis; aseptic joint necrosis; adrenal insufficiency; GI, hepatic, and ophthalmologic effects (glaucoma); hyperlipidemia; growth suppression; central obesity and peripheral wasting; striae; poor healing; Cushingoid facies; buffalo hump; amenorrhea; irritability and wide mood swings; acne; and possible congenital malformations.

Sulfasalazine is commonly used for Crohn’s disease. Its active component is 5-aminosalicylic acid (5-ASA). It has been shown to be beneficial in patients with colitis and ileocolitis, but its effectiveness for Crohn’s disease limited to the small bowel is controversial. Sulfasalazine alone has not been proved to maintain remission. Many patients are allergic to the sulfa component of this medication, and so it is not used as much today because of newer options.

Mesalamine is a newer drug that also releases 5-ASA. It is available in several oral forms as well as a suppository. Mesalamine is considered a first-line therapy for Crohn’s disease and is also often used in combination with a corticosteroid. Corticosteroids, such as prednisone and budesonide, are very useful in the induction of remission of active Crohn’s disease. Corticosteroids alone are ineffective in maintaining remission and should not be used for longer t han 6 months, if at all possible. Tapering of steroids and transition to other maintenance medications is part of the standard approach to therapy.

Antibiotics have been used as adjuncts to other medications. Metronidazole is the most commonly used antibiotic for Crohn’s disease in the setting of perianal disease, enterocutaneous fistulas, or active colonic disease. It seems to have both antimicrobial and immunologic effects, although the mechanism is unclear.

Several newer medications have been developed over the last decade, which have had major effects on treatment paradigms. Most of these medications target specific points in the immune inflammatory cascade. These are anti–tumor necrosis factor-α (anti-TNF-α) antibodies that block the TNF-α receptor, which in turn decreases inflammation. It is used to reduce the signs and symptoms of inflammatory bowel disease (IBD) and maintain remission. Infliximab (Remicade) was the first in this class. It has proved to be very useful in treating patients with Crohn’s disease and has some effect on related fistulas. Other options now include adalimumab (Humira), certolizumab pegol (Cimzia), and vedolizumab (Entyvio). Although initially not used as a first-line therapy for Crohn’s disease, these targeting medications are now being added earlier and earlier because of superior response and possible maintenance of remission. Yet, even these medications have potentially serious consequences including the development of lymphoma and infections. Although much discussed, the risk of lymphoma is actually quite low. Attempts to estimate the actual incidence is confounded by the concomitant use of other immune modulators and reliance on retrospective and limited data.

Surgical therapy is reserved for consequences of the disease that cannot be well managed medically or to reduce the need for high-risk medications, particularly steroids. Typical surgical indications include fibrotic obstruction, abscesses, perforation, fistulas, anorectal disease, cancer, and the rare instance of life-threatening bleeding. Even with high-quality medical management, 50% of patients with Crohn’s disease will require an operation at some time in their life, and many will need more than one. Most medications have not yet proven to be particularly effective for maintaining remission after surgery.

187
Q

Which of the following statements is true regarding colon physiology?

A. Transit time through the colon is independent of the fermentability of nonstarch polysaccharides such as lignin, cellulose, and pectins.

B. The left colon is the segment of colon where bacteria are the most metabolically active in the fermentation process. The right colon is the site of storage and dehydration of stool.

C. Fifty percent of the daily energy expenditure is obtained from the absorption of short-chain fatty acids by the colon; this energy is used to stimulate blood flow, regulate the pH of the colonic environment, and renew colonic mucosal cells.

D. Butyrate is a short-chain fatty acid and a bacterial fermentation product that is the main fuel for colonic epithelial cells.

E. Colonic epithelium can use various fuels, but it prefers glutamine over n-butyrate, glucose, or ketone bodies.

A

ANSWER: D

COMMENTS: The colon plays an important role in the digestive process for fluids and electrolytes. In healthy subjects, the colon normally absorbs 1 to 2 L of water and up to 200 mEq of sodium and chloride per day. This absorptive capacity can increase up to 5 to 6 L/day, thereby protecting against severe diarrhea and dehydration.

The cecum and right colon absorb sodium and water the most rapidly, whereas the rectum is impermeable to sodium and water. Sodium is actively absorbed against chemical and electrical gradients in the colon.

Butyrate plays a role in stimulating sodium absorption in the colon. Potassium and chloride are secreted by the colon through sodium-potassium adenosine triphosphatase (Na+, K+-ATPase) and sodium/potassium/chloride (Na+/K+/Cl−) cotransporters. Chloride ions are actively absorbed at the expense of bicarbonate, which is secreted in exchange. The absence of luminal chloride inhibits secretion of bicarbonate. The main anions in stool include the short-chain fatty acids butyrate, acetate, and propionate.

The host and the colonic bacterial flora have a symbiotic relationship: the host promotes bacterial proliferation with energy substrates from the diet and cellular debris, whereas bacteria provide the host with butyrate, a bacterial fermentation product and short-chain fatty acid that fuels colonic epithelial cells. Nonstarch polysaccharides, or dietary fiber such as lignin, cellulose, and fruit pectins, are the main substrates for bacterial fermentation. Fermentation takes place mostly in the right colon, with the cecum being the colonic segment where bacteria are the most metabolically active. Colonic transit time and bulking of stool are dependent on the fermentability of nonstarch polysaccharides. The transit time of stool through the colon is also dependent on stool pH, the autonomic nervous system, and the gastrocolic reflex (postprandial increase in electrical activity and colonic tone).

188
Q

A 20-year-old healthy, active man with no previous medical problems is being evaluated for chronic constipation. His electrolyte levels are normal. He denies recent travel and is not currently taking any medications. Plain radiographs show a dilated colon. Transit studies are abnormal with slow transit times. What is the next best step in the management of this patient?

A. Flexible sigmoidoscopy

B. Modification of diet and antibiotics

C. Placement of a rectal tube proximal to the normal-caliber aganglionic bowel to decompress the dilated nondiseased bowel

D. Anal manometry, rectal biopsy, and barium enema

E. Exploratory laparotomy

A

ANSWER: D

COMMENTS: This patient should be evaluated for Hirschsprung’s disease. Megacolon may be congenital or acquired. Both forms are characterized by dilation, elongation, and hypertrophy of the colon proximal to a segment of nonperistaltic collapsed bowel causing obstruction. Both are associated with an increased risk for volvulus. Infection with Trypanosoma cruzi (Chagas disease), Hirschsprung’s disease, and neuronal intestinal dysplasia should all be considered in a patient with slow transit constipation and megacolon.

Hirschsprung’s disease is caused by congenital absence of ganglion cells in the myenteric plexus of the bowel, which results in the loss of peristaltic activity in that segment of the intestine. The rectosigmoid region is most frequently involved, with the variable extension of the disease proximally. There is a transition zone from normal bowel, which is dilated, to abnormal bowel, which is aganglionic, aperistaltic, and of normal or decreased caliber. Although Hirschsprung’s disease is primarily a disease of infants and children, occasionally patients with Hirschsprung’s disease will present later in life if an ultrashort distal rectal segment is involved. In these cases, patients usually relate a history of constipation dating back to infancy. In most cases, the diagnosis is apparent during the first 24 h of life when the infant fails to pass meconium. A rectal biopsy is diagnostic. In adolescents and young adults, Hirschsprung’s disease can be diagnosed by anal manometry. If the disease is present, the rectoanal inhibitory reflex (relaxation of the internal sphincter in response to rectal distention) is lost because of the absence of normal ganglion cells, necessary for this myenteric reflex. Treatment of Hirschsprung’s disease is surgical. The effected segment plus any markedly dilated bowel above it is resected, and a coloanal anastomosis is constructed.

Acquired megacolon may be seen in patients with protozoal colon infections with T. cruzi, which is endemic in South and Central America. This condition has not been reported in North America. T. cruzi causes widespread destruction of the intramural nervous system. Acquired megacolon may also occur in patients with colonic dilation due to chronic constipation because of the loss of voluntary defecatory muscles (e.g., in paraplegia), extreme inactivity (e.g., in poliomyelitis), or voluntary inhibition of defecation (e.g., in psychotic disorders). Resection of the excessive redundant colon is occasionally justified in the latter group of patients.

189
Q

A screening colonoscopy identifies a broad sessile villousappearing lesion of the rectum beginning 4 cm above the anal verge and extending 5 cm proximally. Biopsies show a villous adenoma with dysplasia. Endorectal ultrasound (EUS) shows that the muscularis propria is not involved. No suspicious lymph nodes are seen. Which of the following approaches is the most appropriate for the management of this patient?

A. Repeated biopsies

B. Fulguration

C. Transanal excision

D. Abdominoperineal resection (APR)

E. Intracavitary radiotherapy

A

ANSWER: C

COMMENTS: A villous adenoma with the dimensions given has a 30%–50% chance of harboring cancer. An endoscopic biopsy represents a very limited sample size and is not an adequate proof of the lesion’s precise histologic characteristics. In this instance, the finding of dysplasia suggests a high probability of cancer elsewhere in the adenoma. A complete, full-thickness transanal excision of the lesion should be performed so that if a carcinoma is present, its depth of penetration can be assessed accurately. If there is no invasive cancer, the patient is monitored by interval endoscopic examinations because the risk for recurrence is approximately 10%, even though the initial lesion was benign. If invasive cancer is found, the need for further treatment is determined on the basis of the depth of penetration. A T1 cancer is adequately treated by transanal excision, provided that the tumor s not poorly differentiated and lacks vascular or lymphatic invasion and the margins of excision are clear. T2 or T3 cancers should be treated by radical resection because of the much higher rate of lymphatic spread. Radiation therapy is not used for benign lesions and is not necessary for T1 lesions. Some patients may refuse radical resection for a T2 or T3 lesion; radiation with or without chemotherapy may be offered, but this is not the standard of care. Fulguration of these lesions can be performed in very poor-risk patients in whom precise histologic staging is not essential. However, this is not the standard of care for good-risk patients. If there is a local recurrence after the transanal excision of a benign lesion, reexcision, endoscopic fulguration, or argon plasma coagulation may be considered. APR is rarely indicated for benign polyps because many less radical treatment options are now available. Intracavitary radiotherapy was performed for superficially invading malignant lesions earlier, but the unit has not been made for many decades now and so is rarely available. Lesions extending above 6 to 8 cm from the anal verge are best excised with either transanal endoscopic microsurgery (TEM) or transanal minimally invasive surgery (TAMIS). TEM excision has been shown to be much more precise and reliable than transanal excision with standard operating anoscopes. Long-term data on TAMIS is not yet available.

190
Q

In which of the following settings should a low anterior resection (LAR) be performed?

A. A 56-year-old male with a circumferential villous adenoma beginning at the dentate line and extending 8 cm proximally

B. Palliation of obstructing rectal cancer just above the dentate line with minimal liver metastases in a 60-yearold female

C. An 80-year-old male with a large rectal cancer that produces anal pain and tenesmus and involves the anal sphincter

D. A 45-year-old female with an anastomotic recurrence after LAR of a distal rectal cancer

E. A 75-year-old male with urinary incontinence and a rectal cancer 5 cm above the dentate line

A

ANSWER: E

COMMENTS: APR of the sigmoid, rectum, and anus with a permanent colostomy was originally described by William Ernest Miles in his landmark paper from 1908. It is frequently performed for cancers of the distal part of the rectum and for recurrent or persistent anal cancer. Miles showed that there are three routes of lymphatic spread from rectal cancer: superiorly along the inferior mesenteric artery (IMA) chain, laterally along the middle rectal vessels to the pelvic and iliac chains, and inferiorly along the pudendal and inguinal pathways. At that time, most cancers presented in advanced stages. Subsequently, it has been found that virtually all spread of rectal adenocarcinoma is through the superior route. Spread in the other two directions only occurs when the superior route is obstructed by advanced disease.

The APR is a large operation and is associated with many complications such as sexual and urinary dysfunction and poor healing of the perineal wound, especially after radiation. Most benign lesions of the rectum including large and even circumferential rectal adenomas may be removed with a variety of transanal techniques that preserve the sphincter muscles and fecal continence. Curative resection of cancers in the mid and even distal portions of the rectum in some patients can be performed by LAR and either colorectal or coloanal anastomosis without the need for a permanent colostomy. Many studies from the 1940s to the 1960s showed that cure rates were equivalent when comparing APR with LAR as long as margins were not compromised and a complete mesenteric resection was performed. Limiting factors include the need for a distal mural margin of 2 cm, adequate mesorectal excision, and good sphincter function. The end-to-end surgical stapling devices introduced through the rectum have greatly facilitated anastomoses deep in the pelvis. Concerns that local recurrence rates would be higher as a result have not been borne out. Recurrence rates after stapled and hand-sewn anastomoses are the same.

APR is usually performed with curative intent, although it may be justified for symptomatic patients (palliative) with a minimal metastatic disease who are expected to survive 6 months or longer. A cancer that produces anal pain and tenesmus usually involves the sphincter muscle or is obstructing. Recurrent cancer following low resection of a distal cancer usually mandates APR. Fecal incontinence if already present will probably worsen following LAR and a deep pelvic anastomosis because of the loss of normal rectal compliance and capacity.

191
Q

With regard to ulcerative colitis, which of the following statements is true?

A. The entire colon is involved with skip areas in at least one-half of patients.

B. Crypt abscesses on pathology are diagnostic of ulcerative colitis and are not seen with other inflammatory conditions of the bowel.

C. The course of the disease is most commonly a chronic relapsing one, with acute fulminant colitis in only 10%–15% of patients.

D. Cancers arising in association with ulcerative colitis tend to be located in the rectum and sigmoid colon, similar to sporadic colorectal cancers (CRCs).

E. Histologic demonstration of granulomas reliably confirms the diagnosis.

A

ANSWER: C

COMMENTS: Ulcerative colitis is usually limited to the mucosal and submucosal layers of the colon and rectum. The rectum is almost always involved with continuous spread proximally to varying lengths of the colon. The entire colon is involved in at least one-half of patients. The characteristic pathologic finding is crypt abscesses and not noncaseating granulations that are seen in about one-third of patients with Crohn’s disease. Crypt abscesses are disruptions of the epithelium and lamina propria at the base of the colonic crypts of Lieberkühn. They are characterized by sloughing epithelial cells and infiltration of neutrophils and eosinophils in the epithelium and lamina propria. Crypt abscesses are always present with ulcerative colitis but may also be seen with other inflammatory conditions of the colon such as radiation colitis and certain infections. Therefore they are not pathognomonic for ulcerative colitis. An additional common pathologic finding is goblet cell depletion, as opposed to Crohn’s disease, in which the goblet cells are preserved.

Ulcerative colitis is most commonly chronic and relapsing in character, although in 10%–15% of patients the disease runs an acute and fulminant course that may present as toxic colitis.

Cancers associated with ulcerative colitis are often advanced when diagnosed because the signs and symptoms may be confused with an inflammatory relapse. For this reason, these cancers are associated with a poorer prognosis. Studies have shown that contrary to what was previously believed, cancers in colitis patients do not behave more aggressively than sporadic cancers when similar stages are compared. Cancers associated with ulcerative colitis are more evenly distributed throughout the colon than sporadic cancers that are more common in the rectum and sigmoid, have a higher incidence of proximal involvement, and are frequently multiple.

192
Q
  1. A 39-year-old man with a history of mild, long-standing ulcerative colitis controlled with sulfasalazine recently underwent routine colonoscopy that showed a lesion in the sigmoid colon. Pathologic evaluation reveals high-grade dysplasia. Which of the following is the best management option?

A. Sigmoid colectomy, provided that the rectum is minimally involved

B. Proctocolectomy with ileal pouch–anal anastomosis (IPAA)

C. Total abdominal colectomy with ileorectal anastomosis (IRA)

D. Total proctocolectomy with Brooke ileostomy

E. Polypectomy and frequent surveillance colonoscopy to remove further polyps

A

ANSWER B

COMMENTS: Cancer develops in approximately 5%–6% of patients with ulcerative colitis. Patients with pancolitis or disease of long-standing duration (> 8 to 10 years) are at the highest risk.

Sigmoid colectomy is not a reasonable option since the finding of high-grade dysplasia is indicative of a high risk for cancer anywhere in the colon, not just where the biopsy was taken. Abdominal colectomy with IRA does not eradicate the disease or remove all of the mucosa at risk for malignant transformation. IRA is now rarely performed for ulcerative colitis because of the presence of persistent disease in the rectum, the risk of neoplasia in the rectum, and poor function because of poor rectal compliance and tenesmus. Historically, the risk for subsequent rectal cancer after an IRA is approximately 20% after 25 years, even with close surveillance. Therefore over 50% of patients will require completion proctectomy because of cancer, dysplastic changes, refractory proctitis, or poor function.

Total proctocolectomy with permanent end ileostomy (TPC) is an acceptable operation, particularly when a one-step approach is preferred or when the patient is a poor candidate for a sphincterpreserving procedure because of age, c-morbidities, or poor sphincter function. Healthy, motivated patients who require surgery for ulcerative colitis generally prefer a proctocolectomy with sphincter preservation and IPAA.

The continent ileostomy procedure (Kock pouch) with the creation of an internal distal ileal reservoir and an invaginated nipple valve was described in the late 1960s for urinary diversion and then adapted as an alternative for the standard end ileostomy. The pouch must be intubated with a rigid plastic tube multiple times ea ch day to stent the valve open and allow the pouch to empty. Even in the best of hands, these require revision surgery in 25%–30% of patients because of slippage of the nipple valve and incontinence or obstruction. This procedure has been largely replaced by the IPAA operation. The combination of proctocolectomy, an ileal reservoir (J pouch), and IPAA offers advantages over proctocolectomy and permanent ileostomy because not only is the diseased mucosa eliminated but so is the need for a permanent abdominal stoma. The straight ileoanal operation was originally described by Mark Ravich, M.D., and David Sabiston, M.D., in 1947 but was not pursued because of copious diarrhea and frequency. The technique has undergone many modifications, most notably the addition of an ileal pouch proximal to the IPAA. The pouch may be S or J shaped, which increases stool storage capacity and decreases frequency. A temporary diverting ileostomy is usually required for 2 to 3 months while the pouch heals. The procedure is currently recommended for the majority of patients with ulcerative colitis and for patients with familial polyposis. It is not indicated for Crohn’s disease because of the high risk for recurrence within the pouch that may lead to complex fistulas and septic complications. Although advanced age is not an absolute contraindication, elderly patients with multiple comorbid conditions may be better served by a TPC with a permanent ileostomy. For appropriately selected patients, the functional results are good, with preservation of the autonomic innervation to the bladder and genitalia. Fecal sensation and continence are retained in most patients. Daytime incontinence occurs in about 5% and nocturnal incontinence or soiling occurs in 20% of patients. Over 95% of patients retain the pouch over the long term.

193
Q

Enteric fistulas may be a complication of diverticulitis.

Which of the following statements is true?

A. Colocutaneous fistulas frequently occur spontaneously.

B. Suspected colovesical fistulas are best confirmed with barium enema.

C. Coloenteric fistulas may be totally asymptomatic.

D. Surgical correction of enteric fistulas is best accomplished in stages.

E. Colonic fistulas occur in about 30% of complicated cases of diverticulitis

A

Answer: C

COMMENTS: Enteric fistulas occur in about 5% of acute cases of colonic diverticulitis. Fistulas usually develop as a consequence of localized abscess formation with perforation into an adjacent hollow organ, typically the bladder, uterus, vagina (after hysterectomy), or small bowel. Most spontaneous diverticular complications occur with the first episode of diverticulitis. Colocutaneous fistulas rarely form spontaneously. They are most commonly seen as a postoperative complication in which they drain through operative incisions or drain tracts.

Colovesical fistulas are most frequently the result of diverticular disease, followed in frequency by cancer, Crohn’s disease, radiation-induced colitis, and foreign bodies. The primary symptoms of fecaluria and pneumaturia are associated with the urinary tract. The patient may relate a history of abdominal pain and fever before the development of the fistula. Although a barium enema may give information regarding the site and extent of involvement of the colon with diverticulosis, the fistula is demonstrated in only one-half of the cases. Cystoscopy is often very useful and may demonstrate the fistula opening into the bladder with stool, gas, or mucous passage or erythema and edema of the wall of the bladder, a finding consistent with a fistula. Computed tomography (CT) may reveal a constellation of findings, including air in the bladder, a thickened loop of bowel lying against the bladder, and enteric contrast in the bladder (before intravenous contrast material has been administered). CT has become the diagnostic test of choice. Coloenteric fistulas may cause no symptoms or may produce diarrhea, depending on which segments of bowel are involved with the fistula.

Most stable fistulas can be corrected with a one-stage operation, which is the preferred treatment. If bowel preparation is inadequate or there is extensive local inflammation or abscess formation beyond the immediate vicinity of the colon or its mesentery, a staged approach may be required. The first stage includes a Hartmann’s resection of the involved bowel with a colostomy and distal closure along with repair of the bladder. The second stage involves colostomy take down and colorectal anastomosis.

194
Q

A 54-year-old man underwent right hemicolectomy for colon
cancer. Pathologic analysis showed invasion of the tumor into the muscularis propria, with 2 of 18 lymph nodes positive for tumor. What is his pathologic staging?

A. Dukes A

B. Astler-Coller A

C. T2N1 (stage IIIA)

D. T2N1 (stage IIB)

E. T3N2 (stage IIIA)

A

ANSWER: C

COMMENTS: The Dukes classification (1932) was the original standardized method for staging CRC.

In subsequent years, however, confusion arose because of numerous modifications.

The American Joint Committee on Cancer (AJCC) standardized the tumor-nodes-metastasis (TNM) classification that is now used globally (Table 23A.1).

The TNM classification stages a tumor according to the depth of tumor invasion into the bowel wall (T), the presence or absence of lymph node involvement (N), and distant metastases (M).

A T1 tumor penetrates only into the submucosa, whereas a T2 tumor partially invades the muscularis propria.

Transmural penetration imparts a T3 designation, a T4a lesion comes through to the serosal surface of the bowel, and a T4b tumor invades into an adjacent structure.

An N0 lesion has not metastasized to regional nodes, an N1 lesion involves three or fewer lymph nodes, and an N2 lesion has metastasized to four or more lymph nodes.

The designations M0 and M1 indicate whether there are distant metastases.

It is recommended that a minimum of 12 lymph nodes should be examined for reliable nodal staging.

Stage I is T1 or T2 only, stage II is T3 or T4 only, stage III is any lymph node involvement (or tumor deposits separate from the primary), and stage IV is any distant metastases (Table 23A.1).

Prognosis is clearly differentiated by these stages, and survival has gradually improved over time with better surgery and better adjuvant therapy.

Stage I colon cancer patients have a 5-year disease-free survival (DFS) rate of about 90%.

Stage II patients who have had appropriate surgical resection have a 5-year DFS of about 75%.

Stage III cancer patients treated by surgery alone have a 5-year survival of about 30%, but with adjuvant therapy the 5-year survival will rise to 60%–70%.

Stage IV colon cancer carries a poor prognosis, with a 5-year survival rate of 5%. Some patients with disease limited to the liver or the lungs may be treated for cure.

195
Q

Cecal diverticula are different from sigmoid diverticula in
that:

A. Sigmoid diverticula are true diverticula while cecal diverticula are false diverticula.

B. Cecal diverticulitis is usually distinguishable from cancer.

C. Cecal diverticula are considered congenital in origin.

D. Asymptomatic cecal diverticula found on barium enema or colonoscopy should be treated operatively because of the high incidence of complications.

E. Feculent peritonitis from perforation of a cecal diverticu- lum may be treated with resection and primary anastomosis in most cases.

A

ANSWER: C

COMMENTS: True diverticula contain all layers of the bowel wall and are generally congenital.

False diverticula lack the muscularis propria and are acquired. Sigmoid diverticula are acquired false diverticula.

Isolated cecal diverticula are usually true or congenital.

However, acquired diverticulosis may affect all parts of the colon (pandiverticulosis).

Cecal diverticulitis is relatively uncommon, and 80% of cases are initially diagnosed as acute appendicitis or, occasionally, perforated cancer.

It is more common in patients of Asian descent.

In patients with repeated attacks, the cecal inflammation and subsequent fibrotic scarring may be indistinguishable from cancer. Such an inflammatory mass of the sigmoid colon may resemble a cancer at laparotomy.

Treatment depends on symptoms, complications, and the number and severity of recurrent episodes. Incidentally found asymptomatic diverticula do not require surgery.

Surgical options depend on the extent of inflammation. If the inflammation is minimal and limited, segmental resection and anastomosis may be performed. If there has been perforation with frank feculent peritonitis, a Hartmann’s resection with a proximal ostomy and distal closure is performed.

A middle of the road option is resection with anastomosis and proximal diverting loop ileostomy.

Closure of the loop ileostomy is a much smaller operation than closure after a Hartmann’s procedure.

196
Q

A 21-year-old woman is noted to have persistent bloody diarrhea, abdominal cramps, and fever. Stool studies are negative for infectious diarrhea. Colonoscopy reveals friable mucosa in a continuous manner from the rectum to the sigmoid colon. No granulomas are found on biopsy. Which of the following statements is true regarding the most likely diagnosis in this patient?

A. Pseudopolyps and cobblestoning are common colono- scopic findings.

B. Surgery is curative for this diagnosis.

C. Rectal sparing is commonly seen on colonoscopy.

D. Perianal fistulas are commonly found on rectal examination.

E. Small bowel involvement is common.

A

ANSWER: B

COMMENTS: This patient’s presentation is most consistent with ulcerative colitis. In ulcerative colitis, the anus is spared, whereas in Crohn’s disease, the anal or perianal disease is the first manifestation in up to 25% of patients.

The anal disease ultimately develops in 50%–70% of patients with Crohn’s colitis.

Rectal involvement can be seen with both of these inflammatory diseases of the colon but is more common in ulcerative colitis (95% vs. 50%).

The small bowel is extensively involved in approximately 50% of patients with Crohn’s disease, whereas “backwash ileitis,” a nonspecific dilation of the terminal ileum, occurs in about 10% of patients with ulcerative colitis and has no prognostic or physiologic implications.

The clinical features of these two entities are similar: chronic diarrhea, cramping, abdominal pain, and fever.

Bloody stools are quite common with ulcerative colitis and are much less frequent with Crohn’s disease.

Removing the entire colon and rectum is curative in ulcerative colitis, whereas there is no curative operation for Crohn’s disease since it may involve any part of the intestinal tract.

Total proctocolectomy for isolated colonic involvement of Crohn’s disease has the lowest recurrence rate of any operation for that disease, yet recurrence in the small bowel is still seen in more than 25% of patients.

Moreover, one-third of patients require additional surgery for such recurrence.

Toxic megacolon can be an emergency, life-threatening complication of either ulcerative colitis or Crohn’s disease, although it is more common in ulcerative colitis. Surgery for ulcerative colitis has two goals: eliminate the disease and provide reasonable GI function.

Common surgical options today are total proctocolectomy with end ileostomy or proctocolectomy with sphincter preservation and IPAA.

Yet, even when clinical and pathological findings confirm ulcerative colitis, recurrence or development of Crohn’s disease in the small bowel and especially in the pouch or anus occurs in about 5% of patients after proctocolectomy.

197
Q

A 27-year-old man after cholecystectomy has recently undergone colonoscopy for recurrent blood per rectum. His colonoscopic findings are seen in Fig. 23A.1. Which of the following is the most likely explanation for the endoscopic findings?

A. Diet high in fiber
B. Diet low in animal fat and protein 
C. Ulcerative colitis
D. Familial polyposis
E. Previous cholecystectomy
A

ANSWER: D
COMMENTS: In the United States, CRC is second only to lung cancer as the leading cause of death from cancer when both genders are considered.

Environmental factors, particularly dietary habits, may explain the wide variation in the geographic distribution of colon cancer.

This patient has familial polyposis.

Genetic factors play a definite role in carcinogenesis, and mutational abnormalities have been identified in patients with familial polyposis and hereditary nonpolyposis colorectal cancer (HNPCC) syndromes.

Cancer develops in almost 100% of patients with familial polyposis, usually by the age of 40 years, if the colon is left untreated.

In HNPCC syndrome, the lifetime risk for the development of colorectal cancer approaches 80%.

Diets low in fiber and high in animal fats and protein are associated with an increased risk for colon cancer.

The mechanisms may include alterations in intestinal transit time and an increase in the formation of carcinogenic compounds as a result of bacterial metabolism of dietary components.

Gallstone disease appears to be more common in areas where colon cancer is prevalent.

Some studies have suggested that cholecystectomy is associated with a higher incidence of subsequent colon cancer, particularly that involving the right colon.

A proposed mechanism for this relationship is related to the carcinogenic potential of secondary bile acids, to which the intestinal mucosa is increasingly exposed after cholecystectomy as a result of increased enterohepatic cycling.

However, evidence supporting this association is conflicting, and any association that may exist is minimal.

Risk factors for the development of cancer in patients with ulcerative colitis include disease of long duration (the incidence increases by 1%–2% per year after 10 years) and total colonic involvement.

An increased risk for cancer has also been seen in patients with Crohn’s disease of both the small and large intestines, particularly in bypassed segments.

The aforementioned notwith- standing, familial polyposis, HNPCC syndrome, and ulcerative colitis account for only a small percentage of the total cases of CRC; most colon cancers occur sporadically without a genetic or inflammatory predisposition.

198
Q

Which disease is correctly matched to the appropriate
treatment?

A. Actinomycosis: penicillin and drainage

B. Lymphogranuloma venereum: penicillin and steroids

C. Tuberculous enteritis: isoniazid and colectomy

D. Yersinia infections: metronidazole and appendectomy

E. Entamoeba histolytica: metronidazole and right hemicolectomy

A

ANSWER: A

COMMENTS: Actinomycosis is a suppurative, granulomatous disease caused by Actinomyces israelii, an anaerobic, gram-posi- tive bacterium that produces chronic inflammatory induration and sinus formation.

Although the causative organism is often part of the normal oral flora, infections may occur in the cervicofacial area, thorax, or abdomen.

The cecal region is the most frequent site of abdominal infection.

A pericecal mass, abscesses, and sinus tracts may be seen. Rectal strictures have been reported as well.

Treatment consists of surgical drainage and penicillin or tetracycline.

Lymphogranuloma venereum is a sexually transmissible disease caused by Chlamydia trachomatis.

It occurs most frequently in men who have sex with men. It starts as proctitis with symptoms of tenesmus, discharge, and bleeding. Perianal and rectovaginal fistulas and rectal strictures may develop.

The diagnosis is made with the Frei intracutaneous test when it is available. Otherwise, the diagnosis may be confirmed by a complement fixation test. Tetracycline is curative, and steroids are occasionally recommended.

Tuberculous enteritis is seen most commonly in the ileocecal region and occasionally leads to stenosis of the distal ileum, cecum, and ascending colon; the endoscopic and radiographic features produced may be indistinguishable from those of Crohn’s disease. Surgery is reserved for patients with obstruction. A triple-drug therapy consisting of isoniazid, p-aminosalicylic acid, and streptomycin usually heals the intestinal lesions.

Yersinia infections are caused by a gram-negative rod that is transmitted through food contaminated by feces or urine. It is often associated with eating undercooked pork. It produces a clinical picture frequently indistinguishable from that of acute appendicitis with fever and right lower quadrant abdominal pain. Yersinia may also cause acute diarrheal gastroenteritis. It primarily affects the ileocecal region. Diagnosis is made by stool cultures or a poly- merase chain reaction (PCR) probe. Scanning often shows inflam- mation and thickening of the terminal ileum. Yersinia enterocolitis may be self-limited. When suspected or diagnosed, it generally responds promptly to treatment with tetracycline, streptomycin, ampicillin, or kanamycin.

Amebic colitis is caused by the protozoan E. histolytica. It primarily affects the colon and rectum and may secondarily infect other organs such as the liver. Up to 10% of the American population are asymptomatic carriers. Transmission of the disease is through food or water contaminated with feces containing Ent­ amoeba cysts. The disease can assume an acute or a chronic form. Treatment is metronidazole, 750 mg three times a day for 10 days.

199
Q

At the time of surgery for left colon obstruction, you find a thickened segment of colon with a narrow lumen and proximal bowel impacted with stool. There are no liver masses palpated. Which of the following is not an appropri- ate operative strategy for this patient?

A. Stricturoplasty

B. Left hemicolectomy and primary anastomosis following intraoperative colonic irrigation

C. Initial decompressive colostomy, bowel washout, and resection within 7 to 10 days

D. Primary left colectomy, colostomy, and either a Hart- mann’s pouch or mucous fistula

E. Primary subtotal colectomy and ileocolic anastomosis

A

ANSWER: A

COMMENTS: CRC is the leading cause of colon obstruction in industrialized countries, and left-sided tumors in particular are susceptible to obstruction because of the more formed con- sistency of the stool.

This scenario may also be seen with other types of primary or metastatic cancer, diverticular disease, and radiation- or ischemia-induced stricturing.

For left-sided tumors producing obstruction, the traditional surgical approach was an initial decompressive transverse colostomy, followed by resection within 7 to 10 days, and often a third-stage operation for the closure of colostomy.

Initial treatment by decompressive colostomy alone is still appropriate for poor-risk or debilitated patients and for those with a marked colon dilation. The most common approach today is the Hartmann’s procedure—primary resection of the obstructing pathologic entity, colostomy, and distal closure.

The distal side may also be brought out as another ostomy (mucous fistula) for decompression, to provide a route for distal washout, and when the viability of the end may be in question.

The colostomy (and mucous fistula, when present) is taken down, and an anastomosis is constructed at a second stage.

Some surgeons advocate primary subtotal or total colectomy with an ileocolic or IRA as a one-stage procedure for left-sided obstruction. However, this may result in diarrhea and incontinence, especially in older patients.

Most right and transverse colon cancers with obstruction can be treated safely by primary resection and reanastomosis as a one-stage procedure as long as the ileum is minimally distended and healthy.

In the absence of peritonitis or perforation, an alternative approach consists of resection followed by intraoperative colonic irrigation and then primary anastomosis if the proximal bowel is not compromised. Colonic irrigation to wash out the lumen is performed by placing either a cecostomy or an appen- dicostomy tube, mobilizing the proximal colon margin site enough to hang it over the lateral abdominal wall, and creating a controlled conduit for the effluent to exit through a large- caliber anesthesia tubing or a long plastic sleeve fixed in the lumen.

Several liters of saline solution are washed through the colon while manually breaking up stool and milking it down the lumen. This process is rather arduous and time consuming. Anastomoses created with this technique have a clinical leak rate of 5%–7% or more. It is not appropriate when the patient is not stable or when the bowel is at all compromised. This technique is not used very often.

Endoscopically placed colon stents are now commonly used to decompress patients with both malignant and benign strictures. Stents may be used for palliation of a patient with an extensive metastatic disease or as a bridge to surgery. They cannot be used in low rectal cancers since they protrude for several centimeters below the tumor and may cause considerable pain if ending in the very low in the rectum or anal canal.

Stricturoplasty has no role in managing colon obstruction. It is used primarily for patients with benign fibrotic small bowel strictures due to Crohn’s disease.

200
Q

Which statement is correct concerning intestinal polyposis
syndromes?

A. Hamartomas are found in patients with both juvenile polyps and Peutz-Jeghers syndrome (PJS).

B. Familial polyposis syndrome often includes extraintesti- nal manifestations.

C. Turcot syndrome often includes small bowel polyps.

D. PJS, Gardner syndrome, and Turcot syndrome are inherited in an autosomal recessive pattern.

E. Familial polyposis and Turcot syndrome are benign conditions without malignant potential.

A

ANSWER: A

COMMENTS: PJS is a disorder characterized by intestinal hamartomatous polyps along with skin and mucosa pigmenta- tion or “cafe-au-lait” spots. It is due to a mutation in the STK11 gene on chromosome 19 and is inherited as an autosomal domi- nant trait.

Hamartomas are lesions in which normal tissue is found in an abnormal structural configuration. Hamartomatous polyps are found primarily in the jejunum and ileum, with involvement of the colon and rectum in one-third and the stomach in one-fourth of patients. The polyps may cause obstruction, intussusception, or bleeding. The pigmented spots are found on the oral mucosa, lips, palms of the hands, and soles of the feet.

There is an increased incidence of many cancers in PJS, including those in the pancreas, liver, lungs, breast, ovaries, uterus, and testicles. However, the polyps themselves do not become cancerous very often. Gastric and colon polyps are usually treated by polypectomy. Small bowel polyps may be seen on capsule endoscopy.

Symptomatic small bowel polyps may be localized with intraoperative total endoscopy, marking of each site, and serial enterotomy with surgical polypectomy when symptomatic.

Sporadic juvenile polyps in children are solitary 70% of the time, and in 60% of cases they are located within 10 cm of the anal verge. They are thought to be a developmental abnormality and may bleed or prolapse.

The syndrome of juvenile polyposis is characterized by the presence of multiple hamartomatous polyps in the stomach, small bowel, or colon, with large “mucous lakes” on pathology.

Mutations in the BMPR1A and SMAD4 are usually found. It has an autosomal dominant inheritance pattern. Signs and symptoms of anemia, anergy, hypoproteinemia, and failure to thrive may be seen.

Solitary juvenile polyps seem to have little propensity for malignancy; however, up to 40% of patients with juvenile polyposis will develop a CRC.
Cronkhite-Canada syndrome (CCS) is a sporadically occurring polyposis syndrome not thought to be inherited. It is diagnosed mostly in patients of Japanese descent, but it can occur in any ethnic group.

The polyps are hamartomas, often have an inflammatory component, and may be anywhere along the GI tract. In addition, there are pigmented skin areas, alopecia, and atrophy of the finger- nails and toenails. The syndrome may be due to an autoimmune disorder since there are elevated levels of immunoglobulin (Ig)G4, and it often responds to treatment with steroids and immunosup- pressives.

There seems to be an increased risk of colorectal neoplasia, but because of its rarity, the level of risk is not clear. Nevertheless, regular screening is recommended.
Familial adenomatous polyposis (FAP) is the best characterized inherited CRC syndrome and is due to an autosomal dominant mutation in the adenomatous polyposis coli (APC) gene.

Patients develop hundreds to thousands of polyps by their early teens. Although most are in the colon, many patients also develop duodenal and jejunal polyps, which can be very difficult to manage.

Periampullary cancer is a significant risk. Most patients with FAP do not have extraintestinal manifestations; however, Turcot and Gardner syndromes are variants that do. Patients with Turcot syn- drome develop central nervous system tumors, and patients with Gardner syndrome develop osteomas, exostoses, and desmoid tumors, which may be quite debilitating.

201
Q

With regard to ischemic colitis, which of the following
statements is true?

A. The most common symptoms are lower abdominal pain and bright red rectal bleeding.

B. Occlusion of the major mesenteric vessels is the cause of ischemia in most cases.

C. The splenic flexure and hepatic flexure are the most vulnerable areas of the colon.

D. Nonoperative management is not justified because of the significant rates of perforation and peritonitis.

E. Griffith’s point at the rectosigmoid junction is the most vulnerable area.

A

ANSWER: A

COMMENTS: Ischemic colitis should be considered in the dif- ferential diagnosis of any elderly patient with lower abdominal pain. It can also be found in individuals of any age in association with hypercoagulable states, periarteritis nodosa, systemic lupus erythematosus, rheumatoid arthritis, polycythemia vera, and sclero- derma.

Ischemic colitis may be manifested as three distinct clinical syndromes, depending on (1) the extent and duration of vascular occlusion, (2) the adequacy of the collateral circulation, and (3) the extent of septic complications.

Approximately 80% of the blood flow to the wall of the colon reaches the mucosa and submucosa, and the remaining 20% supplies the muscularis. Despite the extensive collateral vessels to the colon, it receives only about 50% of the blood flow that the small intestine does. The colon is therefore more sensitive to ischemic injury during acute reductions in blood flow. In contrast to other areas of the body, an increase in functional motor activity of the colon does not result in a parallel increase in absolute colonic blood flow.

Ischemic colitis appears to be a disease of the small arterioles. Although this disease can occur in any segment of the large bowel, it is seen most commonly in the splenic flexure or distal sigmoid colon. A plausible explanation is that these areas may receive sub-optimal blood flow since they are positioned between two vascular systems (“watershed areas”) that rely on an intact but meandering artery for their blood supply. Sudeck’s point is the area between the blood supply from the last sigmoid artery and the superior rectal artery.

However, the clinical significance of Sudeck’s point is questionable because of retrograde flow from the middle and inferior rectal arteries.

Griffith’s point is the vulnerable area at the splenic flexure that is positioned between areas of the colon perfused by the left branch of the middle colic artery and the ascending branch of the left colic artery. The diagnosis is often made by an endoscopic examination that reveals cyanotic, edematous mucosa that may be covered with exudative membranes. Barium enema may show the typical “thumbprinting” of the edematous bowel wall. Injection angiography or CT angiography may be helpful in the questionable patient. However, if gangrenous colitis is suspected on the basis of ominous physical findings, such as involuntary guarding and rebound tenderness and shock, these studies are con- traindicated and prompt laparotomy is mandatory.

Transient ischemic colitis without full-thickness necrosis usually responds to nonoperative management. Parts of the mucosa may slough, but the muscularis remains intact. Late consequences include ischemic strictures that may be resected electively with primary anastomosis after the initial ischemic episode has subsided. If surgery is needed for peritonitis and gangrenous colitis, resection with end colostomy is the preferred operation (Hartmann’s proce- dure or double-barreled colostomy).

202
Q

Which of the following colonoscopic screening recommendations is correct?

A. A patient whose father had CRC at age 64—start screening at age 40, repeat every 10 years

B. A patient whose father had colorectal polyps at age 58—start screening at age 50, repeat every 10 years

C. A patient whose mother had colorectal polyps at age 58—start screening at age 40, repeat every 5 years

D. A patient with HNPCC—start screening at age 30, repeat every 5 years

E. A patient with ulcerative colitis—annual screening after initial diagnosis

A

ANSWER: A

COMMENTS: Although screening recommendations initially involve utilizing stool guaiac and flexible sigmoidoscopy, most patients are still screened by colonoscopy.

Determining the initiation and timing of screening colonoscopies requires physicians to take a thorough personal and family history of colorectal and other diseases.

Briefly, the timing of screening initiation varies by the patient’s history of IBD or genetic predisposition to the disease and by the family history of disease.

The interval is determined by the age of onset of disease in the patient’s family history as a surrogate for the aggressiveness of disease.

All recommendations are a guideline and may be adjusted to more frequent intervals if a colonoscopy reveals positive findings.

It is important to note that these recommendations exist only for screening colonoscopy; should a patient present with changes in bowel habits, rectal bleeding, or other concerning symptoms, the colonoscopy is considered diagnostic and is obtained immediately without regard to the screening age.

The screening recommendations for colonoscopy are as follows:
• Patients with no family history of CRC or polyps—begin at age 50 and repeat at 10-year intervals

  • Patients with a first-degree relative with CRC or polyps at age 60 or older (or two first- or second-degree relatives of any age)—begin at age 40, or 10 years before the earliest diag- nosed age, and repeat at 10-year intervals
  • Patients with a first-degree relative with CRC or polyps at age 60 or younger (or two first-degree relatives of any age)—begin at age 40, or 10 years before the earliest diagnosed age, and repeat at 5-year intervals
  • Patients with known HNPCC—begin at age 20 to 25 years and repeat at 1- to 2-year intervals
  • Patients with a known family history of FAP—begin flexible sigmoidoscopy at puberty and perform annually until 25 or until the colon and rectum are removed. If negative at 25, perform colonoscopy every 10 years
  • Patients with known ulcerative colitis—begin at 8 to 10 years after the onset of the disease and repeat annually
  • Patients with known Crohn’s colitis—similar to ulcerative colitis
203
Q

A 54-year-old man is referred by his physician for the evaluation of rectal bleeding. He admits to a recent history of constipation and rectal fullness. Colonoscopy showed a 3-cm mass starting 2 cm above the dentate line. Biopsies are consistent with a neuroendocrine cancer that contains a large amount of amine precursor (5-hydroxytryptophan). Which statement is correct regarding this tumor?

A. This tumor occurs at equal frequency in the colon and rectum.

B. The incidence of invasive malignancy and metastases correlates with the location of the tumor.

C. This tumor in the rectum frequently causes flushing, diarrhea, and heat intolerance.

D. These lesions are treated by enucleation.

E. Rectal lesions larger than 2 cm are best treated by radical
resection.

A

ANSWER: E
COMMENTS: This patient has a carcinoid tumor of the rectum. This is a neuroendocrine tumor. This group of tumors includes a wide and diverse set of neoplasms that range from small, completely benign lesions to large, poorly differentiated cancers with an extremely dismal prognosis.

These neoplasms have the potential of storing large amounts of an amine precursor (5-hydroxytrypto- phan), and through the amine precursor uptake and decarboxylation (APUD) system, these lesions may produce several biologically active amines including serotonin.

Carcinoids may be found anywhere along the GI tract and in the pulmonary tree and may occasionally be found elsewhere. They are generally divided into foregut, midgut, and hindgut.

The most common locations are the appendix and ileum (midgut), rectum and colon (hindgut), and stomach (foregut).

Rectal carcinoids are fairly common and account for 12%–15% of lesions, while colon carci- noids are relatively rare and account for only 2.5% of all GI lesions.

The incidence of invasive malignancy and metastases to regional lymph nodes correlates well with the size of the carcinoid for both colonic and rectal lesions.

Carcinoids larger than 2 cm are malig- nant 90%–95% of the time, while lesions smaller than 2 cm are malignant only 5% of the time.

GI carcinoids typically arise in the submucosa. Rectal lesions smaller than 2 cm rarely invade the muscularis or spread to the lymph nodes and very rarely secrete metabolically active sub- stances. Therefore they are not associated with carcinoid syndrome and symptoms of flushing, diarrhea, and heat intolerance. They may be locally excised transanally.

Enucleation is not appropriate. Recurrence is rare if margins are clear. Rectal lesions larger than 2 cm or those that have penetrated into the rectal muscularis are usually aggressive neuroendocrine cancers.

Metastatic evaluation is necessary. If widespread metastases are not present, these malignancies are best treated by APR and colostomy or LAR with anas- tomosis if higher in the rectum.

These tumors are not particularly radiosensitive, and no specific chemotherapy regimens have been shown to be particularly effective. However, large series are lacking because of the relative rarity of these lesions, and radiation and chemotherapy are used selectively for palliation.

Malignant colon carcinoids are treated by formal oncologic resection with wide lymphatic clearance. Up to two-thirds of patients with neuroendocrine cancers of the colon are found to have either local spread or systemic metastases at the time of diagnosis.

If disseminated disease is present, resection of the primary lesion is still recommended to alleviate symptoms and avoid bleeding and obstruction.

Carcinoids of the colon infrequently produce carcinoid syndrome, although occasional lesions that have metastasized to the liver will do so.

204
Q

Which of the following findings warrants APR?

A. A fixed circumferential adenocarcinoma just above the dentate line

B. An ulcerating adenocarcinoma whose lower edge is 7 cm from the dentate line, with infiltration and expansion of the second hypoechoic layer seen on ultrasound imaging

C. A 2-cm mobile adenocarcinoma arising in a villous adenoma 3 cm from the dentate line, with an intact second hypoechoic band seen on ultrasound imaging

D. A circumferential adenocarcinoma 12 cm from the anal verge

E. A 1.5-cm submucosal carcinoid tumor 5 cm from the dentate line

A

ANSWER: A

COMMENTS: The most important determinant of the type of operation to perform for a rectal cancer is the location of the lesion within the rectum.

Tumors located 0 to 5 cm from the anal verge, especially those that involve the sphincter muscle and are produc- ing pain, are best treated by APR.

Approximately 10%–15% of tumors within this region can be considered for local excision if they are minimally invasive (Tis or T1—no deeper than the submucosa), are not poorly differentiated, and do not have a lymphovascular or perineural invasion.

A coloanal anastomosis can be performed when the distal margin is 2 cm after mobilization, although functional results are often poor.

Lesions in the upper part of the rectum (10 to 15 cm) are amenable to anterior resection, with the restoration of intestinal continuity by colorectal anastomosis.

Lesions located in the mid-rectum (5 to 10 cm) are treated by a variety of operations, depending on the skill of the surgeon and the patient’s body habitus. Most cancers in this region can be treated by LAR with colorectal anastomosis or coloanal anastomosis. A temporary loop ileostomy is often used to protect the low colorectal anastomosis, especially in radiated patients because of the higher risk of anastomotic leak. Impaired fecal continence has been noted in 10%–35% of patients after coloanal anastomosis.

Part of the degradation in the function is due to the loss of the rectum’s capacity and storage function. Techniques such as the construction of a colonic J pouch or performance of a coloplasty to widen and increase the compliance of the distal colon may improve function but have not been adopted widely.

The decision to perform a sphincter-saving operation must be individualized, and safety is a primary concern.

If the patient is obese or debilitated; if the pelvis is long, narrow, and angulated; if a satisfactory anastomosis cannot be performed; or if sphincter function is poor, an APR may be the best choice.

Low rectal carcinoids larger than 2 cm should be treated with radical resection (APR or LAR) and lymphadenectomy.

Smaller lesions may be treated with local excision.

205
Q

Which of the following is the appropriate operation for a
sigmoid cancer that has not metastasized?

A. Segmental resection of the sigmoid with 5-cm margins and high ligation of the sigmoid branch of the IMA

B. Resection of the entire sigmoid from the distal descending colon to the rectosigmoid junction, sparing the main left colic artery

C. Resection of the sigmoid and the descending colon, with high ligation of the IMA at its origin

D. Resection of the entire colon proximal to the lesion with IRA

E. Resection of the entire sigmoid with wide iliac lymph node dissection and prophylactic oophorectomy

A

ANSWER: B

COMMENTS: The amount of colon resected when operating with the intent to cure colon cancer is based more on the need for removal of the draining mesenteric lymph nodes that parallel the vascular supply to that area of the colon.

The IMA arises from the aorta 3 to 4 cm above the aortic bifurcation.

The left colic artery splits off about 3 cm below the origin of the IMA and ascends in the mesentery to meet the marginal artery of Drummond that communicates with the left branch of the middle colic artery and the superior mesenteric artery distribution.

Below the left colic artery, the IMA gives off several sigmoidal branches and finally terminates as the left and right superior rectal arteries.

Sigmoid cancers should be resected with at least a 5-cm margin and usually much more since this is not the determining factor.

The need for wide mesenteric clearance is important since up to one-third of patients with involved lymph nodes are cured by proper resection alone.

Adequate lymph node clearance also provides a large number of nodes for sampling to determine the stage. At least 12 lymph nodes must be evaluated, but a comprehensive pathologic examination will often identify 30 nodes or more. The more nodes evaluated, the more accurate the staging.

High ligation of the IMA at its takeoff from the aorta used to be advocated to get the highest lymph nodes; however, subsequent studies have shown that the division of the IMA just below the left colic artery results in equivalent outcomes and decreases the risk of left colon ischemia.

For a sigmoid cancer without the evidence of distal spread, the resection should include the entire sigmoid colon to just above the descending colon–sigmoid junction proximally and to the true rectum where the taeniae coli spread out and become a full longitudinal coat of the rectum distally, plus at least 5 cm on either side of the tumor. If the tumor is at or adjacent to this distal level, a portion of the upper rectum should be removed as well (LAR).

It is important to note that the point where the sigmoid becomes the rectum is defined by a complete longitudinal muscle layer and not by any extrinsic landmark such as the sacral promontory or the “peritoneal reflection,” which are meaningless in this context.

Resection of the entire intraabdominal colon may be considered for patients with an obstructing cancer because resection of a dilated stool-laden colon may safely permit an ileorectostomy rather than a colostomy.

Other indications for a total colectomy include synchronous cancers in separate segments of the colon cancer or in patients with known familial polyposis or Lynch syndrome because of the high risk of metachronous cancers.

Oophorectomy may be considered in postmenopausal women because approximately 6% of these patients have simultaneous drop metastases to the ovaries, and it removes the risk of ovarian cancer.

Routine prophylactic oophorectomy does not improve survival in general.

Isolated recurrence in an ovary is fairly uncommon, and less than 2% of women with CRC subsequently undergo ovarian resection.

Additionally, spread to the ovaries is a sign of disseminated disease, and oophorectomy is not associated with increased survival.

206
Q

Which of the following is true about HNPCC (Lynch
syndrome)?

A. It is inherited as an autosomal recessive trait.

B. Most cancers in patients with HNPCC involve the right colon.

C. The average age at diagnosis is 62 years.

D. A segmental colectomy is frequently curative for these patients.

E. There is a high frequency of associated breast, brain, and lung cancers.

A

ANSWER: B

COMMENTS: HNPCC syndrome is categorized into two forms depending on whether it is isolated (Lynch syndrome I—colon cancer alone) or associated with extracolonic tumors (Lynch syndrome II—endometrial, ovarian, breast, urothelial, biliary, and gastric).

HNPCC accounts for approximately 5% of all CRCs and is caused by mutations in one of the several mismatch repair genes, allowing the propagation of errors in DNA replication.

These genes include MSH2, 60%; MLH1, 30%; MSH6, 7%–10%; and PMS2, <1%. Inheritance is autosomal dominant.

Colon cancers are more common on the right colon (72%), are frequently synchronous (more than one at a time—18%), and are frequently metachronous after segmental colectomy (have a second colon cancer develop later—40% in 10 years). Therefore patients undergoing only segmental colectomy have a high rate of developing another colon cancer.

In HNPCC, the risk of developing colon cancer is over 80%, endometrial cancer 60%, stomach cancer 15%, ovarian cancer 10%, hepatobiliary cancer 5%, urinary tract cancer 5%, small bowel cancer 3%, and brain cancer 2%.

Although patients with HNPCC have a high rate of developing colon cancer, the overall survival is actually better in them than in patients with sporadic cancers.

Identifying patients and families with HNPCC requires vigilance but is quite important.

The Amsterdam Criteria, developed in 1990, is the original method and uses the 3-2-1 rule:
• 3 successive generations affected by CRC,
• 2 first-degree relatives affected by CRC, and
• 1 affected person younger than 50 years.
This was fairly reliable in that people satisfying the criteria were highly likely to have HNPCC.

However, there are many patients with HNPCC who do not quite satisfy these criteria, for example, small families with few siblings or cousins or spontaneous mutations. In addition, the presence of extraintestinal cancers was not included.

Therefore, in 1998, the revised Amsterdam II Clinical Criteria were released:
• 3 or more relatives with an associated cancer (colorectal, endometrial, small intestine, ureter, or renal pelvis),
• 2 or more successive generations affected,
• 1 or more relatives diagnosed before the age of 50 years,
• 1 should be a first-degree relative of the other two,
• FAP should be excluded, and
• Tumors should be verified by pathologic examination.

The Amsterdam Criteria were developed before the genes responsible for HNPCC were known and before genetic testing was available.

They are exclusionary with a high degree of specificity but low sensitivity (62%). Now, testing by immunohistochemical staining of tumor specimens and direct genomic characterization is possible and reasonably inexpensive. A new approach was needed.

The National Cancer Institute developed the Bethesda Guidelines to identify patients who are candidates for genetic testing. These guidelines are much more inclusive and cast a wide net. Alone they are not terribly specific, but with appropriate testing, they are estimated to be 94% sensitive.

The current Revised Bethesda Guidelines from 2004 state that anyone who satisfies any one of the following criteria is a candidate for further testing:
• CRC diagnosed in a patient who is less than 50 years old
• Presence of synchronous or metachronous CRC or other Lynch
syndrome–associated tumors, regardless of age
• CRC with high microsatellite instability histology diagnosed in
a patient less than 60 years old
• CRC diagnosed in one or more first-degree relatives with a
Lynch syndrome–associated tumor, with one of the cancers
being diagnosed at less than 50 years of age
• CRC diagnosed in two or more first-degree or second-degree
relatives with Lynch syndrome–associated tumors, regardless of age

Family members at risk should undergo biannual colonoscopy beginning at age 25 or 10 years younger than the age of the youngest affected family member.

Women should have an annual pelvic examination and pelvic ultrasound to examine the ovaries and the thickness of the endometrial stripe. Serum markers for ovarian cancer should be checked, and mammograms should be obtained earlier than usually advised. Family members of an afflicted individual should be counseled and offered genetic screening.

Total colectomy with IRA should be recommended for HNPCC patients with a new colon cancer because of the high risk for metachronous tumors. If a woman has completed childbearing, hysterectomy and bilateral salpingo-oophorectomy should also be recommended at the time of colectomy.

207
Q

A 54-year-old patient presents with rectal bleeding and tenesmus. She has a history of cervical cancer and underwent a hysterectomy and radiation therapy 5 months ago. She has no personal or family history of cancer, abdominal problems, or rectal bleeding and has not traveled recently. A surveil- lance CT 1 month ago was negative. Which of the following is the most likely diagnosis?

A. Ulcerative colitis
B. Ischemic colitis
C. Radiation proctitis
D. Recurrent cervical cancer 
E. Rectal cancer
A

ANSWER: C

COMMENTS: Radiation therapy is used in the treatment of cervical, uterine, anal, rectal, and prostate cancer.

It is a very effective treatment but may result in severe side effects due to dose-depen- dent cell death in normal surrounding structures.

Injury from radiation therapy occurs via damage to DNA and production of oxygen free radicals. The maximum safe therapy is considered to be 6000 to 8000 cGy, depending on the tissue, but side effects can occur at much lower doses.

The effects of acute radiation toxicity tend to manifest in the first 6 months as villous atrophy and degeneration of the mucosal lining. At 6 to 12 months following radiation, patients develop a progressive microvascular fibrosis (obliterative arteritis), causing thrombosis and ischemia.

Acute radiation proctitis typically manifests as diarrhea, mucous discharge, tenesmus, and bleeding. Chronic changes frequently present as nonhealing ulcerations and telangiectasia fistulas, thickening, and stricture. Symptoms differ between those induced with external beam radiation therapy and those induced with brachytherapy.

Most patients can be adequately treated with symptomatic control, antidiarrheals (after infectious causes are ruled out), and hydration.

Sulfasalazine and sucralfate enemas can provide relief in some patients. Other modalities to control bleeding may include argon beam plasma (APC) coagulation, hydrocortisone enemas, and topical application of formaldehyde (4%–10%) under direct vision.

Although ulcerative colitis and new rectal primaries can present with tenesmus and bleeding, they are unlikely causes of new symptoms in this patient with prior radiation.

Ischemic colitis must be ruled out but again is inconsistent with the clinical history.

Recurrent cervical cancer is unlikely to cause these symptoms with normal imaging.

208
Q

A 28-year-old patient with severe ulcerative colitis undergoes a total proctocolectomy with IPAA. Four days after the operation, she develops fever, anal pain, and tenesmus. Which of the following is the most likely diagnosis?

A. Small bowel obstruction
B. Pelvic sepsis
C. Ileoanal anastomotic stricture D. Pouch–vaginal fistula
E. Pouchitis

A

ANSWER: B

COMMENTS: Creation of an IPAA is associated with low mortal- ity but considerable morbidity. Small bowel obstruction can occur in up to 20% of patients; this high figure is expected given the extent of underlying inflammation as well as the colectomy and extensive mobilization of the small bowel.

Pelvic sepsis occurs in up to 5% of patients. It may present early or late, usually manifesting as an abscess or perineal fistula. The main symptoms are anal pain, tenesmus, fevers, and purulent discharge from the anus.

The diagnosis is confirmed with CT imaging, which may show a discrete abscess or only soft tissue thickening. Antibiotics should be started immediately, and any fluid collections should be drained percutaneously or unroofed trans- anally if inaccessible. A return to the operating room for laparotomy is rarely necessary.

In the literature anastomotic stricture occurrence has been reported in a variable percentage of patients and is best avoided by a tension-free anastomosis and rapid control of any developing pelvic sepsis.

The symptoms are generally more gradual, and the patient may have anal pain but generally no infectious symptoms.

Prior to the closure of the temporary loop ileostomy, mild stenosis is expected, but it may be easily dilated at the closure operation. A pouch–vaginal fistula generally presents with isolated vaginal dis- charge or urinary tract infections.

Pouchitis is the most common long-term complication of IPAA. The symptoms are similar to pelvic sepsis and may include fevers, abdominal cramping, and diarrhea; however, this will occur much later in the clinical course.

Pouchitis occurs in up to 25% of patients with an IPAA. It is usually treated with oral antibiotics and resolves quickly.

Some patients have many recurrences or chronic inflammations. Crohn’s disease in the pouch occurs about 4% of the time and should be ruled out as a cause of abscesses, fistulas, and pouchitis.

209
Q

Which of the following neoplastic polyps is most likely to require surgical excision after a complete colonoscopic resection?

A. A 3-cm pedunculated carcinoma–containing adenomatous polyp with low-grade differentiation, no lymphovascular invasion, and no mucosal penetration that is resected piecemeal in five sessions

B. A 2-cm pedunculated carcinoma–containing adenomatous polyp with low-grade differentiation, positive lymphovas- cular invasion, and no mucosal penetration resected completely in one session

C. A 3-cm pedunculated hyperplastic polyp with no evidence of neoplasia

D. A 3-cm adenomatous noncancerous polyp with a 3-mm resection margin

E. A 1-cm hyperplastic polyp with a 1-mm resection margin

A

ANSWER: B

COMMENTS: Endoscopic polypectomy is an effective treatment that spares selected patients the morbidity of surgical resection in the treatment of advanced polyps. Most pedunculated polyps can be resected endoscopically. Large sessile polyps are more difficult to remove. Most can be safely removed in several pieces with no impact on recurrence, as long as the margins are clear.

All polyps undergo a complete pathologic examination after excision to determine not only whether the polyp is completely resected but also if high-risk factors are present that may indicate a higher likelihood of residual disease, lymphatic spread, or risk of recurrence.
Adenomas harbor a higher risk of malignancy than nonadeno- matous polyps.

Hyperplastic polyps harbor no risk, and serrated polyps are in between. Factors that predict focal carcinoma in an adenomatous polyp include increased size, high-grade dysplasia, and villous versus tubular structure.

Factors that are associated with an increased risk of spread when there is a focus of cancer are high tumor grade (poorly dif- ferentiated), penetration into the submucosa or deeper (T1 or greater vs. in situ location in the mucosa), lymphovascular inva- sion, and resection margin of less than 2 mm.

Many of these patients will undergo surgical resection. In the absence of these factors, the metastatic spread would be exceedingly rare.

Piecemeal removal of large polyps presents a problem since it may be impossible to determine if the resection margins are clear. Polyps of the rectum are considered more aggressive and require a more complete excision.

A full-thickness transanal excision, with clear margins using either standard operating anal retractor for low lesions or TEM is recommended.

210
Q

Which of the following is the best initial management for
acute colonic pseudo-obstruction (Ogilvie’s syndrome)?

A. Colonoscopy
B. Rectal tube decompression
C. Nasogastric tube decompression and correction of electrolytes
D. Neostigmine
E. Lower GI and gastrografin enema
A

ANSWER: C

COMMENTS: Ogilvie’s syndrome was first described by Sir William Heneage Ogilvie in 1948. This is a syndrome of obstipa- tion, acute distention, and dysmotility or ileus of the colon without evidence of a mechanical obstruction. It is associated with the use of opiates and neuroleptic medications, diabetes, myxedema, sclero- derma, uremia, hyperparathyroidism, lupus, Parkinson’s disease, retroperitoneal hematomas, and severe metabolic illnesses.

It is commonly encountered in immobile hospitalized patients after orthopedic or neurosurgical procedures.

Its pathophysiology is not clear but may involve an imbalance in autonomic neural function of the colon distal to the splenic flexure that results in a contraction of the distal part of the colon and functional obstruction.

Frequently, the right and transverse sections of the colon are dilated compared with the distal colon. The risk of ischemia rises when the cecal diameter reaches 12 cm or more.

If the patient is hemodynamically stable, without peritonitis, and without a known mechanical obstruc- tion, initial management includes hydration, mobilization, correction of any electrolyte abnormalities, avoidance of offending drugs such as opiates, placement of a nasogastric tube if the stomach or small bowel is distended, tap water enemas, and serial abdominal examinations.

Mechanical obstruction should be ruled out with a contrast enema or colonoscopy.

Colonoscopy may be both diagnostic and therapeutic; it will rule out a mechanical problem such as carcinoma or volvulus, and decompression of the colon or reduction of a volvulus may be performed.

Seventy percent of patients will improve with conservative treatment in the first 48 h.

Neostigmine or colonoscopy should be considered if conservative treatment fails. Neostigmine is a cholinesterase inhibitor that produces colonic contraction; however, it can also cause bradycardia.

All patients receiving neostigmine must be placed on a cardiac monitor, and atropine must be readily available to treat significant bradycardia.

Placement of a rectal tube is rarely effective because the dilation is primarily in the proximal colon.

Colonoscopy to the transverse colon is adequate for suction decompression.

Colonoscopy may need to be repeated. Patients who fail nonoperative measures should be considered for surgical treatment such as cecostomy or resection if the cecum or right colon is compromised, ischemic, or perforated.

211
Q

Which of the following screening tests provides the greatest
effectiveness (reduction in mortality) in detecting CRC?

A. Annual fecal occult blood test (FOBT)

B. Flexible sigmoidoscopy every 5 years

C. Colonoscopy every 10 years

D. Barium enema every 5 years

E. CT colonography

A

ANSWER: C

COMMENTS: Screening asymptomatic, average-risk patients for CRC with adenomatous polyp removal has been clearly shown to reduce the incidence of CRC and to increase the number of early cancers detected versus advanced cancers.

However, screening must be accomplished in a cost-effective manner that encourages patient compliance.

Screening tests mainly take the form of FOBT, flexible sigmoidoscopy, and colonoscopy.

The test that most easily accomplishes the goals of compliance and cost-efficacy is an annual examination of the stool for occult blood (FOBT). This test uses the peroxidase-like activity of hemo- globin. Three separate stool samples are collected on a guaiac paper, and hydrogen peroxide is added; if hemoglobin is present to catalyze the reaction, the colorless guaiac is oxidized to a blue-colored quinone.

Normal blood loss in the stool is 2 mg of hemoglobin/g of stool; FOBT requires fecal blood loss of 10 mg of hemoglobin/g of stool to obtain a positive result. However, the sensitivity of FOBT is as low as 50%–85% in patients with known cancers. Mass screening programs with FOBT will yield positive results in 1%–8% of patients. The positive predictive value of a positive test result is 10% for cancer and 30% for adenoma. These programs diagnose a higher percentage of early localized cancers than may be expected otherwise, indicating the usefulness of such an approach.

The risk reduction in CRC mortality ranges widely depending on the study and the type of FOBT used. Estimates range from 18% to 55%.

Combining annual FOBT with periodic flexible sigmoidoscopy every 5 years provides a greater risk reduction. Flexible sigmoidoscopy provides an effectiveness of approximately 34%–66%; effectiveness is increased in combination with annual FOBT.

The benefits of this regimen above colonoscopy lie in a shorter, less costly procedure that does not require oral bowel preparation.

Flexible sigmoidoscopy is estimated to detect 70% of lesions present. When combined with FOBT, it provides reasonable sensitivity for the screening of a large population.

Double-contrast barium enema provides an alternate means of screening with an effectiveness of 33%–47%. It may provide improved visualization in colons with multiple strictures than flex- ible sigmoidoscopy or colonoscopy. It is generally used in conjunction with flexible sigmoidoscopy.

CT colonography (formerly called “virtual colonoscopy”) was introduced in 1994. Like colonoscopy, it requires a bowel preparation. Air is insufflated into the colon followed by thin-cut images in a CT scanner in the prone and supine state. Three-dimensional reconstruction of the entire colon and the use of a “fly-through” view result in kinetic images that may allow better visualization behind the haustral folds than colonoscopy.

However, only lesions greater than 1 cm are reliably seen, and residual stool may not be well differentiated from significant findings.

Although CT colonography is effective as a screening tool versus no screening, it is not as effective as colonoscopy, it costs substantially more to achieve reasonable effectiveness, and it is not therapeutic since polypectomy and biopsies cannot be performed. Therefore it is primarily used in patients who cannot tolerate a colonoscopy or in patients with an incomplete colonoscopy because of difficult passage or obstruction.

Colonoscopy remains the screening test of choice since it is highly sensitive and specific as well as diagnostic and therapeutic.

Colonoscopic screening has been shown to be the most effective method of risk reduction. Studies have shown 60%–90% reduction in the incidence of CRC in patients who are screened appropriately. Although complication rates are higher than other methods, the rate of perforation and significant bleeding is only 4 of 10,000 or 0.04%.

Current screening practices for asymptomatic patients endorsed by the American Cancer Society consist of the following:
• High-sensitivity FOBT with three consecutive stool samples yearly
• Flexible sigmoidoscopy every 5 years with FOBT every 3 years
• Colonoscopy every 10 years
Colonoscopy is also used as a diagnostic test when a person has symptoms, and as a follow-up test when the results of another screening test are unclear or abnormal.

212
Q

Which of the following is true regarding rectal prolapse?

A. The extruded mucosa has radially orientated folds.

B. Rectal prolapse occurs mostly in men, with a male-to- female ratio of 6:1.

C. The Altemeier procedure involves a full-thickness resection of the prolapsed rectum through a perineal approach.

D. Fecal incontinence is not a predominant symptom in rectal prolapse.

E. Rectal prolapse is commonly attributed to intussusception of the rectum due to a neoplastic lead point.

A

ANSWER: C

COMMENTS: Rectal prolapse is a pelvic floor disorder that is most commonly found in women, with a 6:1 female-to-male ratio. This disorder has a bimodal distribution of incidence, with peak onsets within the first 3 years and after the seventh decade of life.

There are varying degrees of prolapse—internal intussusception or occult rectal prolapse (or prolapse of the rectal wall without protrusion through the anus), procidentia (or complete protrusion of all layers of the rectum), and mucosal prolapse.

Rectal prolapse is differentiated from incarcerated internal hemorrhoids by a close examination of the mucosal folds.

Incarcerated internal hemor- rhoids have a radially invaginated tissue, which distinguishes the hemorrhoidal cushion beds.

Rectal prolapse has concentric (target) folds.
The pathophysiology of rectal prolapse is not clear, but several typical abnormalities have been identified.

The normal rectal mes- enteric attachments to the sacrum are lost, and the rectum sags down into the deep pelvis.

Defecography studies have demonstrated that weakness of the pelvic floor allows full-thickness intussusception of the rectum through the anal canal and levator hiatus.

Pudendal nerve injury due to stretching of the nerve in the ischiorectal fossa between its fixed exit from Alcock’s canal and insertion into the sphincter muscles contributes to pelvic floor weakness. Direct trauma, obstetric injury, neuropathic diseases, such as diabetes, and neoplasms involving the sacral nerve root can all lead to pudendal nerve damage.

Although neoplasm is a common cause of adult small bowel intussusception, it is not usually the cause of intussusception in rectal prolapse.

The most common symptom is the sensation of an anal “mass” that reduces with manual pressure. Protrusion usually occurs with increased abdominal pressure such as during coughing or defecation.

Fecal incontinence is a predominant symptom that is seen in 50%–75% of patients with rectal prolapse. Other symptoms include tenesmus and rectal pressure. Internal intussusception often presents with rectal pressure and obstructed defecation. It is best demonstrated by cine defecography.

Operative repairs for rectal prolapse can be done through either an abdominal or a perineal approach.

The abdominal approach usually involves the resection of redundant sigmoid colon and rectopexy. This approach is generally reserved for healthier patients who can tolerate abdominal surgery. Either an open or minimally invasive approach can be used. The recurrence rate is low.

The Altemeier procedure is a perineal approach that includes proctosigmoidectomy with a full-thickness resection of the redundant rectum and a variable amount of sigmoid while prolapsed.

A levatorplasty is also often performed with this procedure to tighten the anal hiatus.

Other repairs include the Delorme procedure (mucosal resection with plication of the rectal wall), rectopexy alone, rectopexy with mesh, and a Thiersch loop.

Procedure selection is highly dependent on the extent of prolapse and the physiologic condition of the patient.

213
Q

Which of the following is the most common cause of
massive colonic bleeding?

A. CRC
B. Ulcerative colitis
C. Diverticulosis
D. Ischemic colitis 
E. Infectious colitis
A

ANSWER: C

COMMENTS: The causes of lower GI bleeding are many; it is accounted for predominantly by diverticular disease or angiodysplasia (60%), IBD (13%), anorectal disease such as hemorrhoids and fissures (11%), and neoplasia (9%).

Few of these lead to massive bleeding.

Frequently found to coexist, diverticulosis and angiodysplasia are responsible for most cases of massive colonic bleeding.

The cause of angiodysplasia, or vascular ectasia, is unknown, but it may be related to degenerative vascular changes associated with aging and to colon wall muscular hypertrophy that obstructs the submucosal veins and leads to dilation and a propensity of these veins to bleed.

Most cases of colonic angiodysplasia are located in the cecum and right colon. They may also be found in the small bowel, especially the terminal ileum.

In contrast to diverticular disease, bleeding from angiodysplasia is venous and not as severe.

Diverticulosis can also cause massive bleeding due to ruptured vasa recta in the neck of a diverticulum. Superficial mucosal ulceration in diverticula usually causes mild and self- limited bleeding. Ulcerative colitis is more likely to cause mild-to- moderate bleeding and is frequently associated with diarrhea and systemic signs of chronic illness such as weight loss and failure to thrive.

Cancer of the colon generally causes occult rather than massive GI bleeding, with slow blood loss in stools rather than marked bright red blood or melena.

Ischemic and infectious colitis may present with mild bleeding usually associated with abdominal pain and fevers, if infectious, or diffuse central or peripheral vascular disease, if ischemic.

214
Q

An 18-year-old man with a 2-year history of ulcerative colitis is admitted for an acute exacerbation of his disease. He is febrile and tachycardic with a heart rate of 135 beats/min. His blood pressure is stable. Laboratories show a leukocytosis of 12.3 × 103/L, and plain radiographs show moderate colonic distention to 7 cm in the transverse colon. What is the next step in the treatment of toxic megacolon in this patient?

A. Nasoenteric decompression, broad-spectrum antibiotics, and intravenous steroids

B. Colonoscopic decompression

C. Emergency total abdominal colectomy with ileostomy

D. Nasoenteric decompression, broad-spectrum antibiotics, and infliximab

E. Diverting colostomy

A

ANSWER: A

COMMENTS: Toxic megacolon is most commonly seen in patients with IBD. Classically, it presents acutely in patients with ulcerative colitis (∼5%) and less frequently in those with Crohn’s colitis. Although the incidence of toxic megacolon is decreasing in IBD because of better medical management, it may also occur with severe Clostridium difficile colitis that is increasing.

Toxic megacolon is a subset of toxic colitis. Both cause critical illness and can lead to perforation, peritonitis, and death.

The diagnosis of toxic megacolon is based on the clinical findings of fever, tachycardia, and abdominal distention, combined with radiographs showing colonic distention to 6 cm or greater and leukocytosis greater than 10.5 × 103/L. Successful management is predicated by early recognition and aggressive treatment.

Immediate surgery is required for perforation and peritonitis. Otherwise, the patient is admitted to the intensive care unit for rapid fluid resuscitation, transfusion of blood products if needed, and nasogastric suction to minimize the accumulation of swallowed air in the colon.

Air usually accumulates preferentially in the transverse colon as opposed to the cecum with distal colon obstruction, and this may be promoted by patients lying supine.

Prompt administration of steroids is still felt to be an important component of therapy, and broad-spectrum antibiotics are routinely administered.

Response to medical management is assessed with serial vital signs and abdominal radiographs.

Colonoscopy is not necessary in patients with known IBD since attempts at decompression are not helpful and intubation of the already tenuous and friable colon may cause perforation.

In patients with no previous history of IBD, a careful proctoscopy to 10 to 15 cm with little insufflation is generally adequate to confirm
IBD.

Infliximab is an anti-TNF-α antibody that blocks the TNF-α receptor, which in turns decreases inflammation. It is used to reduce the signs and symptoms of IBD and maintain remission.

Infliximab is not used in the setting of acute toxic megacolon related to IBD.

Worsening colonic distention, fever, and leukocytosis are indications for urgent surgery.

The operative procedure of choice is abdominal colectomy and ileostomy without proctectomy. The rectum or distal sigmoid may be closed as in a Hartmann’s procedure. If the sigmoid is weak or thinned out, it may be exteriorized through the lower end of the midline incision, wrapped in wet gauze, and matured several days later at the bedside as a mucous fistula (Jones procedure).

Total colostomy with preservation of the rectum allows a sphincter-preserving operation, usually an IPAA procedure, to take place once health has been restored.

Even when medical therapy of toxic colitis and megacolon is successful, most patients do not have a satisfactory long-term outcome and will require colectomy within the next 6 to 12 months for ongoing symptoms and even recurrent toxic colitis.

215
Q

In which of the following patients is colonoscopy indicated?

A. A patient with Crohn’s colitis to monitor the efficacy of treatment

B. A patient currently admitted for his second attack of diverticulitis

C. A patient with multiple prior failed screening colonosco- pies due to a tortuous colon

D. A 30-year-old patient (below screening age) with known hemorrhoids and rectal bleeding

E. A patient with a cecal volvulus and no peritoneal signs

A

ANSWER: D

COMMENTS: Different colonic diseases have different indica- tions for and contraindications to colonoscopy.

Rectal bleeding: Patients with melena, hematochezia, other blood loss in the stool, and iron deficiency anemia of unknown etiology should always undergo evaluation of the entire colon, even if the patient is thought to have an anorectal source. Although bleeding from hemorrhoids, fissures, and other anorectal causes is quite common, it does not usually result in anemia, and a malignant source should always be ruled out.

IBD: Endoscopy aids in the diagnosis of IBD; however, it is not indicated for assessment of response to medical therapy, which can be done based on clinical findings alone. Patients with a history of ulcerative colitis for more than 8 to 10 years are at a higher risk for developing adenocarcinoma of the colon; their risk increases 1%–2% per year after 10 years.

Therefore they should undergo regular surveillance colonoscopy with multiple biopsies to monitor for dysplasia, a known marker for increased cancer risk in IBD.

This should be done even if the disease is in remission.

The risk of cancer in patients with Crohn’s disease is less well understood. It is likely that patients with pancolitis due to Crohn’s disease probably have similar rates of cancer as do chronic ulcerative colitis patients. However, most Crohn’s patients have segmental rather than diffuse colonic disease and so the risk may be relatively lower.

Colonoscopy should not be performed during acute manifestations of IBD because of the potential for colonic perforation.

Ischemic colitis/diverticulitis: Colonoscopy is contraindicated in patients with acute peritoneal inflammation such as acute diverticulitis, peritonitis, or perforation.

Colonoscopy may be per- formed after the acute inflammation has resolved to evaluate for cancer, polyps, or other causes of symptoms.

Volvulus/pseudo-obstruction: Colonoscopy is indicated for patients with sigmoid volvulus or pseudo-obstruction of the colon for decompression and to rule out other causes of symptoms, provided that there are no signs of peritoneal inflammation. Colonoscopic decompression of a cecal volvulus is usually not successful.

Routine screening: Screening begins at age 50 in the average- risk population and continues at 10-year intervals until age 75. Screening after this age is controversial.

Colonoscopy is the gold standard since it is both diagnostic and therapeutic.
Polyps may be removed and larger lesions may be biopsied. If polyps are found and removed, surveillance colonoscopy should be performed at more frequent intervals because of increased risk.

CT colonography (formerly known as virtual colonoscopy) is useful in patients in whom colonoscopy was incomplete because of a tortuous sigmoid colon or pain. If there is any suspected lesion, a colonoscopy must be performed.

If an adenomatous polyp or cancer is discovered during screening sigmoidoscopy, colonoscopy is indicated to exclude the possibility of proximal synchronous polyps (30%) or cancer (4%–8%).
Increased risk screening: Colonoscopy should be performed earlier and more frequently in patients with a known family history of cancer, such as HNPCC, FAP, and those having first-degree rela- tives with colon cancer.

In general, patients with first-degree relatives with polyps or colon cancer should initiate screening at age 40 or 10 years younger than their relative’s age of diagnosis; screening interval is dictated by the age of the affected relative, with younger age signifying increased family risk.

216
Q

Which of the following statements is correct?

A. Backwash ileitis is associated with ulcerative colitis.

B. Diversion colitis is associated with ulcerative colitis and
Crohn’s colitis.

C. Microscopic colitis is associated with Campylobacter infection.

D. Metronidazole is used to treat acute ileitis caused by Yersinia infection.

E. Pseudomembranous colitis is associated with amebiasis.

A

ANSWER: A

COMMENTS: Backwash ileitis consists of nonspecific inflamma- tion and dilation of the ileum in patients with ulcerative colitis involving the entire colon. There is no thickening or narrowing as seen in Crohn’s disease. Its presence does not imply a pre-Crohn’s disease condition, nor does it imply a poor outcome after the IPAA procedure. It will resolve once the ulcerative colitis is treated.

Diversion colitis is found in segments of defunctionalized bowel, usually distal to a diverting ostomy or after a Hartmann’s procedure. Colon epithelial cells seem to require nutrients from the luminal contents passing by, including short-chain fatty acids.

Diversion colitis is thought to result when colonocytes are deprived of these compounds. This is supported by the observation that instillation of short-chain fatty acids ameliorates this condition.

Preliminary trials on idiopathic ulcerative proctocolitis have shown a response to short-chain fatty acid enemas.

Following reversal of the fecal diversion, the endoscopic findings of diversion colitis usually resolve.

Microscopic colitis (also known as lymphocytic colitis) is characterized by a history of watery diarrhea and microscopic inflammation of the colonic mucosa.

The colitis often responds favorably to sulfasalazine.

Collagenous colitis (which exhibits a collagenous band under the surface epithelium of the colon on a microscopic examination) may be a variant of this condition because patients have similar symptoms and also respond to sulfasalazine.

Spontaneous remission of these two conditions is common. Most of these patients have been incorrectly labeled for years as having irritable bowel syndrome.

Colonoscopy with biopsy may yield the correct diagnosis.
Campylobacter causes a diarrheal gastroenteritis that is often self-limited. However, it can cause severe dysentery, especially in a weakened host. It responds well to macrolide antibiotics including erythromycin, clarithromycin, or azithromycin.

Acute inflammatory ileitis causes right lower quadrant pain and is commonly confused with appendicitis or Crohn’s disease. Acute ileitis may be due to infection with Y. enterocolitica. It is often self-limited but may be treated with antibiotics. It is often resistant to penicillin but is sensitive to aminoglycosides, sulfa- methoxazole and trimethoprim (TMP-SMZ, Bactrim), ciprofloxacin, and doxycycline.

Antibiotic-induced colitis (also known as pseudomembranous colitis) is characterized by watery diarrhea, which is rarely bloody and is caused by the proliferation of C. difficile. The diagnosis is best made by detecting C. difficile toxin in the stool.

Either oral vancomycin or metronidazole is used to treat this condition. The latter is less expensive and is therefore used more often.

217
Q

With regard to the APC syndromes, which of the following statements is true?

A. Annual colonoscopic screening of family members should begin at the age of 25.

B. Twenty-five percent of the offspring of an afflicted individual will have the disease.

C. The risk for the development of colon cancer is approxi- mately 50%.

D. Abdominal colectomy and ileoproctostomy eliminate the risk for carcinoma.

E. Periampullary tumors are an important cause of death.

A

ANSWER: E

COMMENTS: Most reports of FAP syndromes reflect experience in American and European populations, but these diseases have been identified in Africans and Asians as well.

There is probably no race or geographic area that is exempt. These polyposis syndromes occur in approximately 1 in every 10,000 births.

The disease is transmitted as an autosomal dominant trait, and therefore 50% of the offspring of an afflicted individual will have the disease.

About 30%–40% of familial polyposis patients do not have a family history of polyposis, and these cases represent spontaneous mutations in the APC gene.

FAP is the causative factor in about 1%–2% of all CRCs.

The polyps are not present at birth but usually first appear at puberty and gradually increase in number so that by the age of 21, the colon and rectum are carpeted by thousands of polyps.

If left untreated, virtually all patients will develop CRC by 40 years of age. Some patients will also develop duodenal and jejunal polyps, which can be very difficult to manage.

Periampullary cancer develops in about 5% of patients, primarily when they already have extensive duodenal polyps.

Surgical management is primarily aimed at reducing the risk of CRC. Since most patients will develop cancer by age 40, prophylactic colectomy is recommended in the teens to early 20s.

The most common approach is proctocolectomy with anal mucosectomy to the dentate line and reconstruction with an IPAA. This procedure removes all of the at-risk colorectal mucosa while avoiding a permanent ileostomy.

Abdominal colectomy with IRA is still recommended occa- sionally since it is a relatively straightforward procedure and pelvic dissection is avoided. It is only appropriate if there are less than 20 polyps in the rectum and they can be cleared preoperatively.

However, close surveillance of the rectal remnant by proctoscopy every 6 months is mandatory. The incidence of rectal cancer after IRA varies widely among series, probably because of variations in surveillance compliance.

Rates of 10%–60% have been published from reputable institutions. In fact, patients are less likely to die of rectal cancer than of periampullary tumors or desmoids.

Nevertheless, the importance of surveillance proctoscopy cannot be over- emphasized. IRA should not be considered if there are large or dysplastic rectal polyps, or if the patient already has a colon cancer.

Attenuated FAP (aFAP), defined as between 20 and 100 polyps, is relatively uncommon. It seems to be due to mutations in a different part of the APC genome that is typical of FAP. Although there is a somewhat lower rate of cancer, it is still a high-risk mutation, and surgical management is usually recommended. An IRA procedure may be appropriate in many patients with aFAP.

Patients with a family history of FAP should be screened with flexible sigmoidoscopy starting at puberty.

Annual sigmoidoscopy is recommended until age 20. Genetic screening is also available but is not 100% accurate, and therefore does not replace sigmoidoscopy.

Colonoscopy is not necessary since all affected patients will have rectosigmoid polyps. Upper GI endoscopy screening should be performed to look for involvement of the stomach and duodenum every 1 to 3 years beginning at the age of 20 to 25. All polyps should be removed, if possible.

218
Q

Match the gene in the left column with the applicable statement in the right column:

A. FAP 
B. p53
C. hMSH2
D. DCC (deleted in
colorectal cancer) 
E. K­ras

a. Tumor-suppressor gene that regulates DNA transcription, common in many cancers, located on chromosome 1?
b. Late-occurring alteration resulting in the loss of cell-to-cell contact, thereby enhancing metastases
c. Tumor-suppressor gene (APC) located on chromosome 5
d. Most common mutation found in patients with HNPCC
e. Oncogene that produces a plasma membrane–based protein involved in the transduction of growth and differentiation signals. When mutated, the protein cannot regulate cell growth and differentiation

A

ANSWERS: A-c, B-a, C-d, D-b, E-e

COMMENTS: The APC gene is located on chromosome 5, is large (consisting of approximately 15 exons), and encodes for a cytoplasmic protein of 2843 amino acids. APC mutations occur in patients with both sporadic CRCs and familial polyposis, are frequent, are comparable in incidence with adenomas and carcinomas, and occur early in the development of cancer. The protein product of the APC gene is normally involved in maintaining cellular adhesion and sup- pressing neoplastic growth, but the mutant protein may not be capable of serving this function. The APC gene thereby acts as a tumor-sup- pressor gene. Approximately 35% of patients with sporadic cancers and up to 75% of those with polyposis cancers have APC mutations that can occur at variable points within the gene. This may explain the various phenotypes associated with the polyposis syndromes.

The p53 gene is a tumor-suppressor gene located on chromosome 17. Mutations of this gene are the most common genetic abnormality found in various human cancers.

The gene encodes for a nuclear phosphoprotein that regulates transcription and nega- tively influences cellular proliferation by binding at specific DNA sites.

For example, cells damaged by ultraviolet light or radiation are kept from replicating by the wild-type (natural) p53 protein. Mutant p53 binds to wild-type p53, thereby preventing specific binding to DNA and permitting tumor growth.

Mismatch repair genes correct errors of DNA replication. Alterations in these genes have been implicated in the pathogenesis of HNPCC.

The genetic mutations identified are (1) MSH2 on chromosome 2 (mutation of this gene may account for up to 60% of the genetic alterations seen in families with HNPCC); (2) MLH1 on chromosome 3, which may act as a tumor-suppressor gene; (3) MSH6 on chromosome 2; (4) PMS1 on chromosome 2; and (5) PMS2 on chromosome 7.

Mutations of the latter two genes account for only 10% of the mutations seen in families with HNPCC. Germline mutations of the hMSH2 and hMLH1 genes by themselves are not enough to produce the HNPCC phenotype. A somatic mutation of the remain- ing wild-type allele is also necessary.

The DCC gene is located on chromosome 18 and is one of the first mutations found to be associated with colon cancer. It encodes for a protein involved in cell-to-cell contact.

Deletions of this gene have been found in 73% of patients with CRCs but in only 11% of those with adenomas, thus suggesting that gene loss occurred late during tumorigenesis. Cancers with the loss of the DCC gene are more likely to initially be seen as an advanced disease (in compari- son with tumors maintaining this gene), and patient survival is consequently compromised.

The K­ras gene, an oncogene found on chromosome 12, encodes for a plasma membrane–based protein involved in the transduction of growth and differentiation signals. Approximately 50% of patients with CRC have K­ras mutations. Large adenomas and adenomas with small areas of invasive cancer have nearly the same incidence of K­ras mutations, thus suggesting that genetic alterations in the K­ras gene occur early (but not as early as APC mutations) during tumorigenesis. It has yet to be proved whether K­ras mutations have any prognostic significance.

219
Q

A pedunculated 1.5-cm tubular adenoma is removed endoscopically from the sigmoid colon and found to contain well-differentiated adenocarcinoma extending to but not beyond the muscularis mucosae. The margin of resection is free of tumor. Select the best therapeutic option.

A. Observation only
B. Endoscopic fulguration of the polypectomy site
C. Operative colotomy and excision of the polypectomy site D. Sigmoid colectomy
E. Laparoscopic segmental colectomy

A

ANSWER: A

COMMENTS: By definition, this lesion is classified as carci- noma in situ or intramucosal adenocarcinoma and is treated adequately by endoscopic polypectomy.

Because a lymphatic plexus exists just below the muscularis mucosae, lymphatic dissemination is possible only when invasion deep to this level has occurred.

The muscularis mucosae of the colon wall may extend for a vari- able distance into the stalk of the polyp and may not even reach the head.

Pedunculated polyps consist of four anatomic levels: Haggitt’s level 1 is the head itself, level 2 is the interface between the head and the stalk, level 3 is the stalk, and level 4 is the junction between the stalk and the colonic wall (Fig. 23A.2).

Endoscopic polypectomy should be considered adequate treatment for a polyp containing invasive cancer at level 1, 2, or 3, if the carcinoma is well differentiated and does not exhibit invasion of the veins or lymphatics and the resection margins are free of cancer.

For example, endoscopic polypectomy would be sufficient for a tubular adenoma with well-differentiated cancer extending to level 3 as long as there was no evidence of venous or lymphatic invasion and the margin of resection was free of disease.

However, a poorly differentiated cancer extending to level 2 would require formal segmental resection either minimally invasive or by open means.

Similarly, any polyp with cancer extending to level 4 requires segmental resection, regardless of differentiation or vascular invasion.

Laparoscopic colectomy is now widely accepted as equivalent to open operation for the cure of colon cancer when performed by experienced surgeons.

The Clinical Outcomes of Surgical Therapy (COST) Study Group performed a randomized controlled trial com- paring open colectomy with laparoscopically assisted colectomy and found no difference in intraoperative complications, reopera- tions, survival, and tumor recurrence.

However, this patient does not need any further treatment other than observation and subsequent endoscopic surveillance.

220
Q

Which of the following is the most important prognostic determinant of survival after treatment of CRC?

A. Lymph node involvement

B. Transmural extension

C. Tumor size

D. Histologic differentiation

E. DNA content

A

ANSWER: A

COMMENTS: Of the many variables that affect the cure of patients with colon cancer, the status of the lymph nodes has con- sistently remained the most important. Even with modern aggressive therapy, the long-term survival of node-positive patients is approximately one-half that of node-negative patients.

The extent of nodal disease also has an impact on the prognosis. Patients with four or more positive lymph nodes have a significantly lower 5-year survival rate than do patients with three or fewer positive nodes.

Tumor size in and of itself has no bearing on metastatic potential or prognosis. The DNA content of colorectal tumors has been studied extensively, and aneuploidy seems to correlate well with histologic differentiation, transmural penetration, and the presence of nodal metastases.

However, the DNA content in general has not been shown conclusively to be an important independent prognostic indicator.

Microsatellite instability is associated with inherited cancers but has also not been shown conclusively to be an independent prognostic indicator.

A meta-analysis of 32 eligible studies involving 7642 patients noted that only about 15% of the CRC population had microsatellite instability reflecting inactivation of mismatch repair genes.

In the remainder of the colorectal population (85%), colon cancer developed from other pathways and included aneuploidy, allelic losses, amplifications, and transloca- tions.

In this study and other studies, microsatellite instability was actually associated with a better prognosis in stage II cancers.

221
Q

Which of the following is true regarding EUS imaging?

A. Sedation is required.

B. The bowel must be prepared as for colonoscopy or colectomy.

C. Scanning is best performed with a 3.0-MHz crystal.

D. Imaging of lesions more than 10 cm from the anus is not possible.

E. Image-guided needle biopsy of extraluminal nodules is safe.

A

ANSWER: E

COMMENTS: Staging of rectal cancer has evolved over the past few years. Accurate staging helps determine appropriate treatment and predicts outcomes. EUS has been used for several decades and is generally performed as an office procedure without the need for sedation or formal bowel preparation.

Both rigid probes and endoscopic mounted devices are available. A single enema is given 1 to 2 h before the examination to remove any stool from the rectal vault. Because only a minimal penetration of the rectal wall and perirectal tissues is required, a high-frequency ultrasound crystal is used (i.e., 7 or 10 MHz) to obtain a high resolution of the superficial structures.

It is possible to image lesions in the mid and upper portions of the rectum, but to be certain that the ultrasound probe is in contact with neoplasms at this level, it is necessary to insert the probe under direct vision through a 2-cm-wide proctoscope.

EUS is used to stage rectal cancer. It is superior to CT for locoregional staging and has an overall accuracy for predicting tumor stage of up to 80%–95% as compared with CT, which is 65%–75% accurate. EUS also provides the benefit of guiding fine- needle aspiration biopsies to improve the accuracy of nodal staging. EUS demonstrates the depth of invasion of cancer (T stage) and nodal involvement (N stage).

For endoscopic ultrasound to be accurate on staging lymph nodes, the nodes must be at least 5 mm. Interobserver variability in performing EUS can be a limitation.

In addition, EUS is limited in restaging patients treated with neoadjuvant chemotherapy and radiation therapy because of the treatment-induced fibrosis. Therefore a complete staging with CT of the chest, abdomen, and pelvis; magnetic resonance imaging (MRI) of the pelvis; and examination under anes- thesia (EUA) in earlier cases should be performed before initiating treatment.

Ultrasound staging is as follows: uT1 is confined to the mucosa and submucosa, uT2 invades but does not penetrate through the muscularis propria, uT3 invades into the perirectal fat, uT4 invades into adjacent organs, uN0 has no lymph node enlarge- ment, and uN1 has lymph node enlargement. Image-guided needle biopsy of extraluminal nodules is a safe procedure that can be performed under ultrasound guidance; suspicious perirectal nodules can also undergo a biopsy in this fashion.

Only if the biopsy specimen contains benign lymphoid tissue can it be assumed that the nodule in question is truly free of cancer.

Pelvic MRI is now part of the recommended staging protocol. It is better at staging larger lesions, looking for extrarectal extension and venous, sacral, and pelvic side wall involvement.

The likelihood of obtaining clear circumferential margins may also be predicted. EUS is more accurate for the staging of early lesions— T1 versus T2—and therefore it is very useful for deciding on local treatment for T0 and T1 lesions versus radical treatment for T2 and greater tumors.

222
Q

A patient presents with a recently diagnosed 3-cm rectal cancer with its lower edge 4 cm from the anorectal ring. EUS shows a tumor extending through the muscularis propria with three adjacent lymph nodes measuring 1 cm each. CT scan does not show any evidence of distant spread to the liver or lungs. Which of the following is the most appropriate treatment for this patient?

A. APR with colostomy
B. LAR with total mesocolon excision and anastomosis
C. Chemotherapy and radiation followed by resection
D. Preoperative external beam radiation therapy
E. Preoperative chemotherapy

A

ANSWER: C

COMMENTS: Multiple studies have shown that radiation therapy to the pelvis decreases the rate of local and regional recurrence and that systemic chemotherapy reduces the rate of systemic progres- sion. Preoperative combined chemotherapy as opposed to postoperative treatment is associated with lower complication rates and better clinical function.

A variety of protocols have been studied globally. In the United States, a full-course radiation therapy con- sisting of about 5000 cGy to the pelvis and regional lymph node basins for over 6 weeks with a final boost to the primary tumor region with synchronous 5-fluorouracil (5FU)-based chemotherapy is standard. Radical resection of the rectum and mesentery with the lymph nodes by either APR with a permanent colostomy for low rectal lesions or LAR with anastomosis and temporary diverting loop ileostomy is performed 6 to 10 weeks following completion of the neoadjuvant therapy. After recovery from the operation, 6 to 12 months of chemotherapy are given, depending on the final pathology. Recent data show that up to 20% of patients will have a complete pathologic response to neoadjuvant treatment, meaning that no residual tumor is found in the resected specimen.

However, a patient with a complete response cannot be reliably predicted based on current clinical evaluation or imaging. Therefore all patients proceed to resection unless they are medically unfit or refuse.

Short-course neoadjuvant therapy with 2500 cGy given for over 1 week followed by surgery 1 week later is used in some centers in Europe but is not standard in the United States. No trials have yet shown that the short-course therapy is as good as the long- course treatment.

This patient has stage III rectal cancer based on pretreatment clinical staging, which shows T3 invasion (through the muscularis propria), three enlarged perirectal lymph nodes (N1), and no evidence of distant spread (M0).

Full neoadjuvant treatment with subsequent surgery is appropriate. LAR with a coloanal anastomosis is likely to be technically possible since a 2-cm margin from the anorectal ring should be achievable.

Therefore APR is not necessary. In stage III patients, neoadjuvant therapy is associated with a significant increase in survival from 30% with surgery alone to 60%–70% with combined therapy.

223
Q

A 62-year-old woman complains to her doctor of bright red blood per rectum, mixed with stool.

On rectal examination, she is found to have a palpable mass at the tip of the finger.

Proctoscopy shows the presence of a posterior 4-cm ulcerated mass that occupies 35% of the circumference beginning at 4 cm from the anorectal ring.

A biopsy shows rectal adenocarcinoma, and a complete staging shows a T3 lesion and several enlarged lymph nodes, but no distant metastasis.

Her carcinoembryonic antigen (CEA) level is normal. Select the most appropriate treatment for this patient.

A. Local excision
B. TEM excision
C. APR
D. Preoperative chemotherapy and radiation followed by resection
E. Fulguration
A

ANSWER: D

COMMENTS: The most common symptom of rectal cancer is hematochezia. Other symptoms include mucus discharge, tenes- mus, and changes in bowel habits, often with a thin, pencil- or ribbon-like appearance. The workup for rectal cancer includes complete colonoscopy to exclude synchronous colon tumors, CT of the chest, abdomen, and pelvis, MRI of the pelvis, and EUA in selected patients who may be candidates for local treatment.

The precise location is best determined by rigid proctosigmoidoscopy, which should be done even if the tumor has been diagnosed with colonoscopy to accurately assess orientation of the tumor and the level above the anal canal.

The upper end of the anal canal is the anorectal ring that is typically 1 to 2 cm above the dentate line. This is the lowest level that resection may be performed without sacrificing the anal sphincters.

A 2-cm margin is recommended since studies have shown that intramu- ral spread of cancer below the visible and palpable margin is rarely beyond 1 cm.

Measurement of CEA levels is standard. Normal CEA levels are up to 5 μg/L. In smokers, they may be up to 7 μg/L.

Although CEA levels are not useful for screening since only 70% of cancers express this antigen and levels are often normal in localized lesions, they are useful for follow-up.

A baseline level is always drawn prior to initiating treatment. Very high levels, in the hundreds to thousands, are associated with metastatic disease. If the CEA is elevated and it normalizes after treatment, it may be useful for detecting recurrence. CEA levels are routinely checked every 3 months for several years.

Transanal excision is not part of the initial evaluation of rectal cancer and should be considered only after the evaluation is complete so that the appropriate treatment approach can be selected.
Preoperative clinical TNM staging (Table 23A.1) as well as the level of the lesion in the rectum guide therapeutic decisions. Stage I tumors do not require neoadjuvant therapy (preoperative radiation and chemotherapy).

Ultrasound-staged T1 tumors can be treated with transanal local excision: either standard transanal excision with conventional operating anoscopes, TEM, or TAMIS using laparoscopic instruments. TEM uses carbon dioxide insufflation through a 40-mm operating rectoscope and a binocular mag- nifying visual system to create a better endoscopic visualization of the operative field. TEM is more likely to yield negative margins, an intact specimen, and lower recurrence than operations using conventional transanal instruments. The pitfall with any transanal approach is an inability to assess lymph node status. In T1 cancers, the incidence of lymphatic metastases is about 5%, and the outcomes of local excision versus radical excision are the same. Therefore they are considered safe for transanal excision.

Ultrasound-staged T2 rectal cancers should undergo radical surgery because of the significant risk of lymph node metastases (25%–30%).

The ultrasound image of this patient showed penetration of the muscularis propria and possibly the mesorectal fat. Suspicious lymph nodes (spherical, hypoechoic) are seen as well. Therefore she is clinical stage III and should undergo neoadjuvant chemora- diation therapy before radical surgery.

APR involves a complete excision of the rectum and anus through a combined abdominal and perineal approach. APR leaves the patient with a permanent colostomy. Thus APR is indicated when the patient has poor preoperative sphincter control, the patient’s body habitus is unfavorable, the tumor involves the anal sphincters, or part of the sphincter needs to be resected to obtain negative margins (as in a tumor less than 2 cm from the anorectal ring).

LAR involves excision of the rectum and mesorectum through an abdominal approach and colorectal or coloanal anastomosis. A temporary loop ileostomy is often used to protect the low colorectal anastomosis, especially in radiated patients because of the higher risk of anastomotic leak.

In the United States, stages II and III rectal cancers are com- monly treated with full-course neoadjuvant (preoperative) radiation therapy consisting of 5040 cGy and 5-FU-based chemotherapy.

Fulguration is rarely used in current surgical practice because of the advances in transanal excision that provides a full-thickness specimen for pathology staging. It may be used in patients with small lesions below the peritoneal reflection that are poor surgical candidates or for small recurrences.

224
Q

A 73-year-old man undergoes an exploratory laparotomy and small bowel resection for small bowel obstruction. During his postoperative course, the patient develops diarrhea, abdominal tenderness, and distention. He is diagnosed with a postopera- tive ileus. A stool specimen is sent that is positive for C. difficile toxin. He is developing a significant leukocytosis, fever, and elevated creatinine. What is the recommended treatment?

A. Metronidazole oral
B. IV vancomycin
C. Piperacillin-tazobactam
D. Vancomycin oral
E. IV metronidazole and vancomycin via a rectal tube
A

ANSWER: E
COMMENTS: Evidence shows that the incidence and virulence of C. difficile are increasing. Metronidazole and oral vancomycin have been known to be effective therapies for C. difficile since the late 1970s. IV vancomycin is ineffective in treating C. diffi­ cile colitis.

A review conducted before 2000 showed that the failure rate of metronidazole for treatment of C. difficile colitis was 2.5% and the failure rate of treatment with oral vancomycin was 3.5%. Since 2000, there has been an increase in this failure rate, with recent data showing a metronidazole failure rate of 18.2%. Newer data also show that both agents still have similar efficacy in mild infection, but that vancomycin has superior efficacy in severe infection.

Rectal administration of vancomycin may be employed as a treatment modality in severe infections and in patients who cannot tolerate oral administration as in the case of this patient with a postoperative ileus.

Dual therapy of IV metronidazole and vancomycin may be employed in severe infections as well.

225
Q

Which of the following statistics are FALSE regarding CRC in the United States?

A. CRC rates are highest in black men and women.
B. CRC rates are lowest in Asian/Pacific Islander men and women.
C. Mortality rates are 30%–40% higher in women than in men.
D. Approximately 5% or 1 in 20 Americans will be diagnosed with a cancer of the colon or rectum in their lifetimes.
E. CRC is the third leading cause of cancer death in women.

A

ANSWER: C

COMMENTS: CRC is the third most commonly diagnosed cancer and the second leading cause of cancer death in men and women combined in the United States.

Approximately 5% or 1 in 20 Amer- icans will be diagnosed with a cancer of the colon or rectum in their lifetimes. There are approximately 150,000 new cases of CRC in the United States per year. CRC rates are highest in black men and women and lowest in Asian/Pacific Islander men and women.

The incidence and death rates of CRC increase with age. Overall, 90% of new cases and 93% of deaths occur in people aged 50 years or older. Mortality rates are 30%–40% higher in men than in women. The median age of colon cancer diagnosis is 69 in men and 73 in women. The median age of rectal cancer diagnosis is 63 in men and 65 in women.

CRC death rates have been declining since 1980 in men and since 1947 in women, and this can be attributable to improved screening, treatment, and decreases in exposure to risk factors.

226
Q

A 32-year-old woman presents to the emergency department with abdominal pain that has gradually increased over the past week. The patient has a history of FAP and underwent a total proctocolectomy with IPAA at age 20. She has 5 to 6 soft to loose bowel movements per day, which is normal for her. A CT scan reveals a 10-cm soft tissue mass in the middle abdomen, displacing the loop of small bowel to either side. Upon review of a CT scan from several years prior, the mass was present but has grown from 7 cm. What is the next step in initial management?

A. External beam radiation 
B. Surgical debulking
C. Chemotherapy
D. Daily tamoxifen
E. Core needle biopsy
A

ANSWER: D

COMMENTS: Extracolonic manifestations of FAP include adenomas of the upper GI tract, desmoid tumors, osteomas, pigmented lesions of the retina, and epidermoid cysts, as well as gastric, thyroid, suprarenal, and central nervous system cancers.

The patient in this scenario has a desmoid tumor of the small bowel mesentery.

This is a typical presentation of a patient with a history of FAP and the finding of an intraabdominal soft tissue mass.

Desmoid tumors are benign growths that occur in 10%–20% of patients with FAP.

They are a form of fibromatosis and are more common in patients with a family history of desmoids.

Histologically, they consist of proliferating myofibroblasts. They have no malignant potential but often progress locally, infiltrating tissues and causing obstruction, vascular compromise, and pain. They are most commonly found in the abdomen, particularly in the small bowel mesentery, in the retroperitoneum, and in the abdominal wall.

Occasionally they will develop in other sites. Desmoids are the second most common cause of death in patients with FAP, after GI malignancies.

Most desmoids grow slowly and may cause symptoms over time; some will grow faster. Some will grow, plateau, and then regress. There are no specific inciting factors, but the observation has been made that desmoids may grow more rapidly after abdomi- nal surgery. They also have a high rate of recurrence after resection.

Asymptomatic desmoid tumors demonstrating little or no growth can be observed. They should be followed with imaging to assess progression.

Initial therapeutic measures are aimed at slowing growth. Several medications have been shown to be of benefit in some patients. Nonsteroidal antiinflammatory drugs (NSAIDs) such as sulindac and indomethacin have been used both to treat desmoids and to slow growth of polyps. Alone, they are not particularly effective. Antiestrogen agents such as tamoxifen and pro- gesterone seem to have more effect. Tamoxifen is currently the mainstay of medical treatment. Combined therapy with an NSAID and tamoxifen may prove to be more effective.

Overall, about 50% of patients have stabilization or reduction in tumor mass with medical therapy.

Cytotoxic chemotherapy is rarely used but is an option when the tumor is growing rapidly and causing symptoms, does not respond to other medical therapy, is not resectable, or recurs rapidly after resection. Typical agents include doxorubicin and carboplatin.

Radiotherapy is rarely used for intraabdominal tumors because of the risks and side effects. These tumors are not particularly radiosensitive.

Radiotherapy may be used for abdominal wall or extremity lesions. It also may be used as an adjuvant to surgery.

Surgical excision is the most effective means of eradicating these tumors; however, recurrence rates are over 60%, and clear margins may be difficult to achieve. There also may be significant morbidity with resection, including ischemia, short-bowel syndrome, and the loss of an ileoanal pouch, if present.

The most problematic lesions are those in the base of the small bowel mesentery that surround major vessels such as the superior mesenteric artery and vein.

If a large abdominal desmoid is causing obstruction, then a bypass is an alternative to resection.

227
Q

Which of the following is NOT a principle of Enhanced Recovery After Surgery (ERAS) protocols in colorectal surgery?

A. Multimodal analgesia
B. Hypothermia
C. Minimizing perioperative starvation
D. Early postoperative ambulation
E. Avoiding aggressive perioperative fluid loading
A

ANSWER: B

COMMENTS: ERAS programs focus on many aspects of care of the surgical patient with the intent of decreasing pain and narcotic use, decreasing the length of stay, decreasing complication rates, reducing costs, and improving the overall experience.

Many ERAS programs have risen in the setting of colorectal surgery. Due to the success of these programs, the concepts are being applied to other surgical specialties.

Although there are variations in the details of the programs among institutions, there are well- established principles.

The major elements within these enhanced recovery programs include preoperative education, multimodal analgesia (the use of epidural regional anesthesia, maximizing usage of nonopioid analgesic options, minimizing narcotic usage), limiting perioperative starvation (preoperative complex carbohydrate drinks, early refeeding), limiting intravenous fluids, maintaining normothermia, and early postoperative ambulation.

Setting expectations and psychologically and emotionally supporting the patients by dedicated personnel helps.

The average hospital stay after colectomy may be reduced from 5.5 to 3.5 days for laparoscopic surgery without an increase in readmission rates.

228
Q

A 58-year-old man had a left colectomy for a T2 N2 M0 colon cancer 4 years ago, followed by adjuvant chemotherapy.

During a surveillance visit, his CEA is found to be elevated, and a CT scan revealed a large, 6-cm lesion in the liver. A subsequent biopsy is consistent with a metastatic recurrence.

The positron emission tomography (PET) scan reveals no other extrahepatic lesions. What treatment should be offered to this patient with metastatic disease?

A. Systemic chemotherapy
B. Radiofrequency ablation plus systemic chemotherapy
C. Orthotopic liver transplant
D. Palliative care consult to discuss goals of care
E. Hepatic resection plus systemic chemotherapy

A

ANSWER: E

COMMENTS: Twenty percent of patients have metastasis at the time of diagnosis, with many more developing metastatic spread throughout the course of their disease.

The most common site of metastasis is the liver; 80% of stage IV patients have hepatic spread, and 40% of these do not have the extrahepatic disease.

Hepatic resection for isolated liver metastases has been clearly shown to be safe and efficacious in experienced hands.

The traditional guidelines for recommending liver resection (metastasectomy) for patients with stage IV CRC are:

1) three or fewer lesions
2) disease limited to one side of the liver
3) no extrahepatic disease, and
4) a willing patient who is a reasonable surgical candidate.

The median survival for isolated liver metastases treated with only chemotherapy is 20 months.

With hepatic resection, median survival is as much as 88% at 1 year and 43% at 5 years. The long-term survival is about 30%.

Hepatic resections for colorectal hepatic metastases have become a much safer surgery over the years, with perioperative mortality of 1%-5%.

The key to safety and efficacy in hepatic resections is the careful patient selection.

Factors associated with poor survival include the presence of extrahepatic disease, inability to achieve a negative margin, short interval to recurrence, number and size of tumors, lymph node–positive primary, high preoperative CEA, advanced age, and significant co-morbid disease.

Other modalities available for treatment of hepatic metastases include radiofrequency ablation, staged resections for bilobar disease, and preoperative portal vein embolization or ligation to induce liver hypertrophy.

An evolving approach to bilobar disease is the two-step ALPPS (associating liver partition and portal vein ligation for staged hepatectomy) procedure.

In the first operation, the liver is divided into two segments.

The side to be retained (FLR, future liver remnant) is cleared of disease by limited resections with clear margins. The portal vein branch to the side to be removed is ligated.

During the weeks following this, the FLR parenchyma undergoes remarkable hypertrophy.

In the second operation a few weeks later, the planned resection is performed, leaving the hypertrophied side.

This approach allows a much larger proportion of the liver to be resected, yet leaves enough for the patient to survive.

Initial results are impressive, but a larger experience with longer follow-up is needed to assess its usefulness.

229
Q

A 60-year-old otherwise healthy woman presents to the emergency department for acute onset of lower abdominal pain with associated nausea and emesis. Her previous surgical history includes a total hysterectomy. On examination, she is distended and peritonitic. Her vitals show tachycardia and hypotension, and lab results show bicarb of 14 and lactic acid of 4.8. The emergency department orders a plain film (Fig. 23A.3). What is the optimal therapy for this patient?

A. Subtotal colectomy with end ileostomy

B. Endoscopic decompression, placement of a rectal tube, and sigmoid colectomy with primary anastomosis

C. Intraoperative decompression followed by subtotal colectomy

D. Laparotomy, detorsion, and cecostomy tube

E. Right hemicolectomy and primary anastomosis with or without diverting ileostomy

A

ANSWER: E

COMMENTS: Colonic volvulus refers to torsion of the bowel around its own mesentery.

The word volvulus originates from the Latin word volvere, meaning “to twist.”

Colonic volvulus is a rare cause of bowel obstruction, collectively accounting for only 2%–3% of cases of colonic obstruction in the United States.

Sigmoid volvulus is the most common form and accounts for about two-thirds of cases.

Cecal volvulus is present in almost one-third of cases, while volvulus of the transverse colon is extremely rare.

During intrauterine development of the fetus, the intestinal tract rotates counterclockwise and is drawn into the peritoneum and out of the yolk sac.

The cecum normally fixes to the right retroperitoneum and posterolateral abdominal wall. Incomplete fixation with a mobile cecum puts patients at a risk for cecal volvulus.

Other risk factors include previous surgery that may result in adhesions that create a point of fixation for twisting, chronic constipation with colon dilation, intestinal malformations including malrotation, excessive exercise, distal colon obstruction, very high fiber diets, and late pregnancy.

The patient in this scenario has a cecal volvulus as demonstrated by her radiograph.

She has signs of ischemic bowel based on her physical examination, vitals, and acidotic state. She needs prompt surgical treatment.

Mortality increases with both the presence of ischemic bowel and a delay in surgical treatment. Nonsurgical treatment has no role in a patient with evidence of bowel ischemia.

Even if this patient was stable without any signs of bowel compromise, endoscopic decompression has been found to be ineffective in the majority of cases of cecal volvulus.

A right hemicolectomy should be performed with a complete resection of all ischemic/gangrenous bowel and primary anastomosis if the margins appear healthy and well-perfused bowel.

The decision to perform an end ileostomy is based on multiple factors including nutritional status, anemia, use of chronic steroids, other comorbidities, and the appearance of the tissues.

In cases where there is no ischemic or gangrenous bowel, detorsion and cecopexy can be considered in the frail or elderly population where minimal surgery is preferred.

This is reasonable although recurrence rates are higher.

Detorsion and cecostomy tube are no longer performed because the complication and mortality rates are much higher than with cecopexy.

Resection should always be performed in the case of ischemic bowel and should be attempted whenever possible, even in the cases of non- ischemic bowel, when the patient can tolerate the operation because it is the definitive treatment and eliminates the risk of recurrence.

230
Q

Which statement is FALSE regarding sigmoid volvulus?

A. Sigmoid volvulus is commonly seen in patients who are elderly, debilitated, chronically constipated, or on psychotropic medications.

B. Mortality rates are lower when emergent decompression is performed before sigmoid resection.

C. Nonoperative reduction of sigmoid volvulus is successful in 70%–80% of cases.

D. If the gangrenous bowel is seen during decompression, a rectal tube should be placed to prevent recurrence.

E. Sigmoid resection within 2 to 3 days of decompression is the preferred definitive treatment.

A

ANSWER: D

COMMENTS: The prerequisite for the development of sigmoid or cecal volvulus is a mobile segment of bowel that can rotate around a mesentery whose points of fixation are in proximity.

Typically, the cecum rotates around the ileocolic artery, while the sigmoid rotates around the IMA. Otherwise, there are surprisingly few similarities between sigmoid and cecal volvulus.

Volvulus of the cecum is found most frequently in persons 25 to 35 years of age, whereas it is unusual for sigmoid volvulus to occur in an active, otherwise healthy individual.

Usually, it occurs in the elderly, debilitated persons, or those with psychiatric or neurologic disorders in which immobility, medications that impair bowel motility, and loss of accessory defecatory muscles may lead to constipation and elongation of the colon.

Both types of volvulus typically cause abdominal distention and pain.

With cecal volvulus, plain abdominal films may show evidence of small bowel obstruction and a dilated colon segment extending into the left upper quadrant, whereas with sigmoid volvulus, the distended twisted loop has a fairly characteristic appearance of a “bent inner tube” extending into the right upper quadrant.

For sigmoid volvulus, endoscopic detorsion plus insertion of a rectal tube to evacuate the voluminous fecal contents is the preferred initial therapeutic approach but should be attempted only if the mucosa does not appear gangrenous. It should not be attempted if the patient has rebound abdominal tenderness or other signs of peritoneal inflammation.

Evidence of gangrenous bowel mandates surgical management.

Although nonoperative detorsion is successful approximately 70%–80% of the time, recurrence rates have been reported from 33% to as high as 90%, and therefore an elective resection of the elongated colon is recommended if the patient has an acceptable operative risk.

Data from the Veterans Affairs (VA) Hospitals have reported mortality rates as high as 24% for patients undergoing initial emergency operations for sigmoid volvulus versus only 6% for patients initially treated with endoscopic decompression followed by scheduled sigmoid resection soon afterward.

Nonoperative colonoscopic reduction of cecal volvulus is difficult and usually ineffective. Prompt surgery is the mainstay of treatment for cecal volvulus. If gangrenous, the cecum must be resected. In the absence of vascular compromise, either resection or sutured cecopexy is a reasonable option.

The most important determinant of a patient’s outcome is whether bowel gangrene is present, with mortality being highest if surgery is performed for intestinal infarction or perforation. Mortality is also higher if operating for recurrent volvulus.

231
Q

Which of the following is not a type of cecal volvulus?

A. An axial volvulus from clockwise twisting where the torsed cecum remains in the right lower quadrant

B. Upward folding of the cecum

C. Downward folding of the cecum

D. A torsion of the cecum and a portion of the terminal ileum resulting in a displacement of the cecum out of the right lower quadrant

E. Bascule

A

ANSWER: C

COMMENTS: Sigmoid volvulus is the most common type of volvulus. Cecal volvulus accounts for approximately one-third of cases of colonic volvulus.

The causes of cecal volvulus can be congenital or acquired.

During embryological development, the cecum normally rotates into the right lower quadrant and becomes fixed to the retroperitoneal structures and right gutter.

Incomplete fixation can put patients at a risk for cecal volvulus.

Other risk factors include previous surgery (allowing adhesions to create a point of fixation for twisting), chronic constipation, intestinal malformation, excessive exercise, distal colon obstruction, very high fiber diets, and late pregnancy.

There are three types of cecal volvulus:

Type I: Axial torsion—An axial volvulus from clockwise twisting of the cecum around the axis of the ileocolic artery. The cecum remains in the right lower quadrant.

  • Type II: Loop torsion—A torsion of the cecum and a portion of the terminal ileum resulting in a displacement of the cecum out of the right lower quadrant into the left upper quadrant. This is the most common type.
  • Type III: Cecal bascule type—The cecum flows upward into the midepigastrium or left upper quadrant on a tangential fold from the hepatic flexure in the right upper quadrant down toward the left lower quadrant.

Surgery is the mainstay of treatment for cecal volvulus. Right hemicolectomy is the procedure of choice since it eliminates the problem.

If resection is not desired or the simplest procedure is preferred in a compromised patient, cecopexy by raising a flap of lateral peritoneum and fixing the cecum to the right gutter under this flap may be considered.

However, recurrence rates are fairly high. If there is any compromise of the bowel such as gangrene or ischemia, resection is mandatory.

In most circumstances, the primary ileocolic anastomosis is safe, although ileostomy and either distal closure or mucous fistula is an option.

232
Q

A 56-year-old man is scheduled to undergo a laparoscopic segmental colectomy for a carcinoma of the descending colon. Which of the following statements is true concerning bowel preparation for colorectal operations?

A. Preoperative nonabsorbable oral antibiotics alone are effective in preventing postoperative wound infections.

B. Preoperative mechanical bowel cleansing alone is most effective in preventing postoperative wound infections.

C. A broad-spectrum antibiotic should be administered only once and just before surgery.

D. Mechanical cleansing with sodium phosphate is preferred in patients with renal insufficiency, cirrhosis, ascites, and congestive heart failure.

E. Mechanical cleansing should be used selectively in patients with a complete bowel obstruction or perforation.

A

ANSWER: C

COMMENTS: The colon contains a higher concentration of bacteria, both aerobic and anaerobic, than any other part of the body.

Infectious complications constitute the major morbidity of colorectal operations and occur at higher rates than most other general surgery procedures.

Escherichia coli is the most common aerobic organism and Bacteroides the most common anaerobic organism found in the colon.

Mechanical cleansing of the colon has been a time-honored practice that can be achieved by the administration of a cathartic in combination with enemas or by peroral lavage with a nonabsorbable polyethylene glycol–electrolyte (PEG) solution administered the afternoon before surgery.

Although there was a period when mechanical bowel preparation prior to colectomy was felt to be unnecessary, recent data have shown that the combination of mechanical preparation, oral antibiotics, and one preoperative dose of intravenous antibiotics within 30 min of the skin incision is the best approach to reduce colorectal surgical site infections.

Oral and rectal Fleet Phospho-Soda (sodium phosphate) was commonly used in the past because of its efficacy in mechanically cleansing the bowel with less volume.

However, its use has been associated with significant electrolyte derangements causing compromise of patients with renal insufficiency, cirrhosis, ascites, and congestive heart failure. Therefore it was removed from the market.

Polyethylene glycol is not contraindicated in patients with renal failure, cirrhosis, ascites, or congestive heart failure since it generally preserves electrolyte balance.

Complete bowel obstruction and bowel perforation are absolute contraindications to mechanical bowel preparation.

233
Q

Which of the following is the least appropriate operative strategy for a patient presenting with sigmoid diverticulitis and localized peritonitis?

A. Sigmoid colectomy with primary anastomosis

B. Sigmoid colectomy with primary anastomosis and diverting ileostomy

C. Sigmoid colectomy with end colostomy and rectal stump

D. Drainage and ileostomy without resection

E. Total colectomy

A

ANSWER: E

COMMENTS: Diverticulitis is the inflammation caused by perfo- ration of a colonic diverticulum. In many cases, the perforation is microscopic and results in local infection and inflammation in the bowel wall and adjacent mesentery. In complicated diverticulitis, a larger perforation may cause a phlegmon, abscess, fistula, or peritonitis.

The severity of complicated acute diverticulitis is graded by the Hinchey classification:

  • Stage I: Diverticulitis with a localized pericolic or mesocolic abscess
  • Stage II: Diverticulitis with a pelvic abscess
  • Stage III: Perforation with purulent peritonitis
  • Stage IV: Perforation with fecal peritonitis

The Hinchey classification does not describe uncomplicated diverticulitis or late complications such as fistula or stenosis.

It provides a loose guideline to the management of acute complicated disease.

Historically, sigmoid colectomy with diverting colostomy and rectal stump—the Hartmann’s procedure—was the primary treatment for acute diverticulitis.

It is still the most common procedure performed in the acute setting. In recent years, primary anastomosis has been found to be reasonable for selected patients presenting with a walled off abscess or localized peritonitis.

The anastomosis must be tension free, inflammation free, away from the inflammatory site or abscess cavity, and well vascularized.

Tissues to be anastomosed must be healthy and not involved in the inflammatory process. The patient must be stable, and a complete washout should be performed.

A number of studies have been performed looking at the option of laparoscopic washout and drainage without resection.

Although some surgeons have been enthusiastic, the studies are low powered and general consensus recommendations have not yet shifted to this approach.

A recent multicenter, randomized trial from the Scandinavian countries showed that primary resection was superior to laparoscopic lavage. Mortality, quality of life, and hospital stay were not different; however, the reoperation rate was higher and several sigmoid cancers were missed.

Still, most cases of diverticulitis are managed nonoperatively with antibiotics. Radiologic drainage of localized abscesses is usually well tolerated.

The general plan is for observation of uncomplicated patients. Interval resection is usually reserved for patients with a drained abscess, a fistula, or multiple episodes of diverticulitis.

The need for resection after radiologic drain is now being questioned; this is currently being studied.

234
Q

Which of the following is a known risk factor for the anastomotic leak?

A. Long rectal stump 
B. Stapled anastomosis 
C. Female gender
D. IRA
E. Prior operation
A

ANSWER: D
COMMENTS: Anastomotic leaks after a colorectal operation can present in a variety of ways, from isolated postoperative tachycardia to fulminant sepsis and peritonitis.

Multiple factors have been implicated in the development of anastomotic leak, few of which are definitive. Implicated, or associated but not necessarily causal, are the use of drains, advanced malignancy, shock, malnutrition, emergent surgery, smoking, steroid use, male gender (related to a narrow pelvis), and technical reasons.

The gut microbiome has recently been increasingly implicated in the development of leaks, independent of other predictors.

Studies are in progress to evaluate this mechanism. Factors that have been definitively related are poor blood supply, tension on the suture line, use of radiated tissue, emergency operations, and contaminated fields.

Anastomoses in the low rectum are more technically challenging and therefore at higher risk of a leak. Ileorectal anastomoses have the highest leak rate of any colonic anastomosis, although it is not clear why.

Patients with Crohn’s disease have a slightly higher incidence of anastomotic leaks. Extensive research has shown that there is no overall differ- ence in the anastomotic leak rates of hand-sewn and stapled anastomoses.

However, a deep pelvic colorectal anastomosis is much easier to construct with a stapler, while a hand-sewn anastomosis should be considered when the bowel is thick, edematous, or otherwise not suitable for stapling.

235
Q

Which of the following is true regarding ileostomy construction and management?

A. Approximately 95% of patients with an ostomy tolerate it well with the improved quality of life.

B. One-third of patients require revision of their ileostomy for parastomal hernias, prolapse, or wound care issues.

C. New-onset necrotic skin changes around an ostomy can always be attributed to poor wound care.

D. Ostomy retraction helps facilitate pouching.

E. The Brooke method of maturing an ileostomy prevents
mucosal sloughing.

A

ANSWER: B

COMMENTS: When performed in low-risk patients with attention to good technique, the creation of a Brooke ileostomy is a safe and well-tolerated procedure with few postoperative complications.

Originally described by Bryan Brooke in 1952, the everted and primarily matured ileostomy surmounted the problems of serositis, stenosis, and retraction, known as ileostomy dysfunction.

The crucial elements of successful ostomy creation include preoperative site marking by a knowledgeable surgeon or enterostomal nurse, adequate mobilization of the mesentery to allow a tension-free ostomy, and primary maturation with eversion using tripartite sutures.

Obese patients with a thick abdominal wall and deep skin folds present a challenge since site selection may be difficult and the mesentery may be thick and short.

Inadequate maturation and excessive tension from the mesentery can cause retraction, which interferes with effective pouching, leading to wound care difficulties, skin rashes and breakdown, leakage, and occasional soft tissue infections.

Finally, a poorly sized fascial aperture can lead to obstruction or ischemia if too small, or a parastomal or prolapse if too large.

For these reasons, up to one- third of patients eventually require operative revision or reversal of their ostomy.

Peristomal rashes and skin breakdown are usually due to ineffective pouching because of poor patient education or compliance, or due to the ostomy problems such as retraction, prolapse, poor siting, or hernia. The services of a trained enterostomal nurse (WOCN, wound ostomy care nurse) are invaluable.

Ideally, patients scheduled for surgery that will include an ostomy will have a preoperative WOCN consultation and organized postoperative training on ostomy care.

WOCNs will also allow nonoperative management of many problems. Patients with Crohn’s disease and, rarely, ulcerative colitis may develop pyoderma gangrenosum with serpiginous skin ulcers. Although pyoderma is typically developing in the pretibial skin, it can also develop around a stoma or in a skin incision.

If pyoderma is suspected, the patient should be evaluated for Crohn’s disease, if not already diagnosed.

Treatment includes medical control of the IBD and local care with various immunosuppressive topical agents and ostomy appliances.

Tacrolimus cream has been quite effective in many patients.

The Brooke method of maturation refers to folding over the mucosa to hide the serosa, preventing serositis and ileitis. This method does not prevent mucosal sloughing, a complication related to ischemia.

236
Q

A 37-year-old man presents with a recurrent perianal abscess. The abscess was drained in the emergency department, and he is now being seen in the clinic for follow-up care. Which of the following is true about perirectal suppuration?

A. The pathophysiology of perirectal abscesses is related to infection of the perianal skin.

B. A horseshoe abscess is best drained at the bedside with the use of local anesthesia.

C. An intersphincteric abscess causes pain higher in the rectum, frequently without external manifestations.

D. Ischiorectal abscesses should be drained immediately under general anesthesia so that the fistula can be identified.

E. Perianal Crohn’s disease is not a risk factor for recurrent disease.

A

ANSWER: C

COMMENTS: Most perirectal abscesses are the result of obstruction of the anal ducts and glands lying in the intersphincteric space.

Horseshoe abscesses include bilateral ischiorectal, supra- levator, or perianal abscesses that communicate.

Horseshoe abscesses usually arise from infection of the posterior midline glands.

A horseshoe ischiorectal abscess starts in the deep postanal space and extends bilaterally into each ischiorectal space.

The patient is best treated in the operating room under regional or general anesthesia by incising the skin from the external sphincter to the coccyx.

This exposes the superficial external sphincter, which is split longitudinally but not transected. The incision provides access to the deep postanal space.

A probe is inserted into the posterior midline crypt and then into the deep postanal space. A seton is placed in this space and wrapped around the internal sphincter and superficial external sphincter. Counterincisions are made laterally along extensions of the abscess.

An intersphincteric abscess is usually accompanied by pain and bulging inside the rectum but no external swelling. Treatment consists of transanally laying open the internal sphincter, beginning at the lower edge of the abscess and extending cephalad to the top of the abscess cavity.

Most perianal abscesses can be drained with the use of local anesthesia, but if the patient has a high fever, significant leukocytosis, or extreme pain, treatment in the operating room under general anesthesia is preferable.

Identification of a fistula may be deferred until there are clinical signs that a fistula is present, specifically, nonhealing of an abscess wound or recurrence of the abscess at the same location.

Patients with anorectal Crohn’s disease may experience recurrent abscesses despite maximal therapy.

Anorectal abscesses may be the first manifestation of Crohn’s disease in over 5% of patients, and up to 50% of patients with Crohn’s disease will develop anorectal issues including abscesses, fistulas, anal ulcers, and large swollen skin tags at some point in their disease process.

237
Q

All of the following are accepted applications of endorectal ultrasound except:

A. Assessing sphincter integrity in patients complaining of fecal incontinence

B. Determining whether a rectal cancer is suitable for local excision

C. Ruling out recurrent rectal cancer

D. Evaluating a transsphincteric anal fistula

E. Routine screening for rectal cancer

A

ANSWER: E

COMMENTS: Endorectal ultrasound imaging has had a signifi- cant impact on the diagnosis and management of a variety of anorectal diseases.

The initial use of endoluminal ultrasound was for the staging of rectal cancers.

The depth of penetration and the presence of abnormal lymph nodes were used to determine the stage of the cancer and its suitability for local excision.

Generally, tumors that demonstrate deep penetration of the rectal wall have an increased likelihood of lymph node metastases and are not suitable candidates for transanal excision because of the unacceptably high recurrence rates associated with local excision of these advanced neoplasms.

Most surgeons would restrict transanal excision to T1 tumors for curative intent. Recently, the use of ultra- sound imaging for staging rectal cancers has been expanded to determine whether a lesion is advanced enough to warrant preop- erative radiation therapy and chemotherapy.

Ultrasound imaging can be used to assess the rectal wall and the extraluminal tissue for any sign of recurrent cancer following surgery. In this respect, it has distinct advantages over other imaging modalities, such as computed tomography, in that the probe is placed in direct contact with the operative site.

Ultrasound has never been demonstrated to be beneficial for routine screening for rectal cancer. Ultrasound requires the application of the probe directly to the tissue of interest.

Endoscopic ultrasound (EUS) may be useful in several benign diseases of the anus and rectum. It may be used to image the sphincter mechanism in patients complaining of fecal incontinence. In fact, before a diagnosis of idiopathic or neurogenic incontinence is made, an ultrasound scan should be done to inspect the integrity of the sphincter.

Ultrasound provides excellent diagnostic accuracy in determining disruptions of the internal and/or external sphincters.

Although most anorectal abscesses and fistulas can be managed without imaging studies, ultrasound has proved useful for deter- mining the extent of abscess collections laterally and in a cephalad direction.

Furthermore, the relation of the tract of the fistula to the sphincter muscle can be assessed with ultrasound imaging.

The internal opening can be identified as a hypoechoic disruption of the internal sphincter muscle. Hydrogen peroxide may be injected into the fistula tract during ultrasound scanning to further delineate the fistula tract.

238
Q

A 36-year-old woman has prolapsing tissue and bleeding with bowel movements both in the toilet following forceful defecation and on the toilet paper. On examination, she has several large vascular lesions that are reducible with direct pressure. Which of the following statements regarding her most likely condition is true?

A. Internal hemorrhoids are vascular cushions arising proximal to the dentate line and are covered by anoderm.

B. Prolapsing hemorrhoids are external hemorrhoids covered by anoderm.

C. Bleeding internal hemorrhoids should be initially managed with surgical excision.

D. Thrombosed external hemorrhoids should be treated by hemorrhoidectomy with the patient under general anesthesia.

E. Recurrence is uncommon after surgical hemorrhoidectomy.

A

ANSWER: E

COMMENTS: Internal hemorrhoids are submucosal cushions normally located above the dentate line and are therefore covered by the transitional mucosa of the anal canal and not by anoderm.

External hemorrhoids are the dilated veins of the inferior hemor- rhoidal plexus located below the dentate line and are covered by anoderm.

Prolapsing hemorrhoids are internal hemorrhoids that prolapse below the dentate line. Bleeding is the main manifestation of smaller internal hemorrhoids and is managed initially by rubber banding, infrared coagulation, or injection sclerotherapy.

Surgery is reserved for internal hemorrhoids that do not respond to dietary measures and office procedures. Surgery may be the best initial therapy for large prolapsing internal hemorrhoids or for those that are acutely thrombosed and incarcerated.

Thrombosed external hemorrhoids are most often treated by excision using local anesthesia in the office setting.

General anesthesia is only required in the uncommon situation when they are quite large and circumferential. Internal hemorrhoidal recurrence is rare after formal surgical hemorrhoidectomy.

When it does occur, it is usually related to inadequate removal of the rectal mucosa and hemorrhoidal tissue or inadequate management of chronic constipation.

239
Q

A 40-year-old woman is evaluated for pain with defecation. The pain started acutely after a large bowel movement, is extremely intense following the passage of any stool, and persists for several hours. She is barely able to tolerate examination, which demonstrates a small defect in the anoderm located at the posterior midline. Which of the following statements regarding her condition is true?

A. Most lesions are located above the dentate line.

B. The lesions are always located at the posterior midline.

C. The operation of choice for this lesion is excision and posterior internal sphincterotomy.

D. Lateral partial subcutaneous sphincterotomy for lesions not in the midline is considered a definitive treatment.

E. Pharmacologic therapy with nitroglycerin, calcium channel blockers, or botulinum toxin may prove beneficial.

A

ANSWER: E

COMMENTS: An anal fissure is a tear of the skin-lined portion of the anal canal located below the dentate line. It is most commonly caused by hard bowel movements and straining.

Occasionally, high internal anal sphincter tone is the cause. Gentle spreading of the buttocks is frequently all that is needed to reveal the fissure. Once the acute fissure is identified, digital examination and anoscopy may be deferred in the acute setting.

About 90% of fissures (acute or chronic) are located in the posterior midline, an area where the anoderm is least supported by the sphincter and where blood flow is the poorest.

The anterior midline is the second most common location and is involved in 10% of women.

Fissures located laterally or multiple fissures should arouse suspicion for Crohn’s disease, syphilis, tuberculosis, leukemia, or other causes, and therapy is directed toward the underlying disease.

The initial treatment of a midline fissure is conservative and starts with a bowel management program including lubricants, fluids and fiber supplements, and warm baths.

Over 90% of acute fissures will heal with this approach. Topical nitroglycerin or calcium channel blocking agents may help resolve chronic fissures in approximately 50% of patients; however, several weeks or months of treatment may be required.

Botox injected directly into the internal sphincter on either side of the fissure may also produce healing in approximately 60% of patients.

However, recurrence is common. Persistent chronic fissures are often used to high internal anal sphincter tension, which can be demonstrated on digital rectal examination or by anorectal manometry.

Operative treatment consists of lateral partial internal sphincterotomy up to the dentate line to relax the internal sphincter.

Posterior fissurectomy and sphincterotomy can lead to a keyhole defect and soiling. For this reason, lateral internal sphincterotomy has become the procedure of choice.

Although mild incontinence to gas or diarrhea may be seen, it is infrequent enough to justify sphincterotomy in the small number of patients who do not respond to conservative therapies.

External sphincterotomy should not be performed for anal fissure because it is not necessary and leads to unacceptably high rates of incontinence.

240
Q

A 41-year-old patient with persistent pruritus ani is seen for further treatment after having undergone a biopsy. Which of the following statements is correct regarding possible causes of this problem?

A. Bowen’s disease progresses rapidly to invasive cancer and requires urgent wide local excision.

B. Paget’s disease of the perianal skin is often associated with breast cancer.

C. Basal cell carcinomas of the anal margin have an excellent prognosis.

D. Buschke-Löwenstein tumors are primarily treated by chemotherapy.

E. Surgical excision of Bowen’s disease and Paget’s disease requires 5-cm margins.

A

ANSWER: C

COMMENTS: Precursors to malignant disease are often found in the anal margin.

Anal intraepithelial neoplasias, such as Bowen’s or Paget’s disease, can cause symptoms such as itching.

Skin changes are usually present, and biopsy is indicated whenever the pruritus does not resolve with conservative measures.

Bowen’s disease represents high-grade dysplasia or squamous cell carcinoma in situ of the perianal skin. It is often associated with the human papillomavirus (HPV) (especially HPV types 16 and 18). It is usually indolent, and less than 5% progress to invasive cancer. Treatment consists of wide local excision with clear margins.

Paget’s disease is adenocarcinoma in situ of the perianal skin, usually from skin appendages such as ducts and glands. A lower gastrointestinal malignancy, usually a rectal or anal carcinoma, is found in 50%–70% of patients, but it is not associated with breast cancer.

After evaluating the colon and anorectum for synchronous tumors, treatment is wide local excision. This disease may be multicentric, so circumferential mapping biopsies around the anal canal and perineum are part of the procedure.

With HPV-related neoplasia, high-resolution anoscopy may be useful for targeting biopsies with excision or destruction of abnormal tissue, but this is primarily used in human immunodeficiency virus (HIV)–positive patients and men who have anoreceptive intercourse.

Basal cell carcinoma of the perianal skin is rare and is associated with a 5-year survival rate of almost 100% following local surgical excision.

Buschke- Löwenstein tumors, also called giant condylomas, are large verrucous squamous cell tumors of the anal canal and perineum due to HPV infection.

Pathologically, they resemble condyloma and may have areas of invasive squamous cell carcinoma. They are rarely metastatic but may invade local structures around the anal canal or margin.

Treatment consists of wide local excision or, rarely, abdominoperineal resection (APR) if the sphincter is invaded. Radiation therapy is occasionally used preoperatively to shrink large cancers or for patients with recurrent tumors.

241
Q

A 27-year-old man who engages in anal receptive intercourse has white, cauliflower-shaped masses throughout his perianal region and in the anal canal. Which of the following statements regarding his most likely condition is true?

A. The causative agent appears to be HPV.

B. Podophyllin, administered in a 25% solution, results in the resolution of the warts in 80% of patients, and recurrence rates are less than 10%.

C. Immunotherapy with vaccination is used as the initial treatment of small lesions.

D. Carcinoma frequently develops if the lesions are left untreated.

E. Surgical treatment involves excision and is not associated with any risk of transmission to health care providers.

A

ANSWER: A

COMMENTS: Anal infection with HPV is responsible for condyloma acuminatum, which appears as a group of cauliflower-shaped masses on the perianal skin and in the anal canal.

The disease is transmitted by close contact and is seen in both genders regardless of whether anal intercourse is practiced.

It is especially prevalent in anoreceptive men who have sex with men, and in this population, it is seen more often than genital warts.

Another high-risk population consists of those receiving immunosuppression after organ transplantation.

Podophyllin, a cytotoxic agent available in 10% and 25% solutions, must be applied by a physician. However, the results have been disappointing. Clearance of the warts has been noted in 22%–77% of patients, with recurrence rates being as high as 65%. Podophyllin may cause skin burns and cannot be used within the anal canal. Multiple treatments are usually necessary. Failure to treat intraanal lesions results in persistent disease.

An autologous vaccine prepared from sampled condyloma can be injected weekly for 6 weeks. No adverse reactions have been seen, and resolution of lesions has been noted in up to 95% of patients. However, the process is not standardized, and so at present, such therapy is only considered for extensive, persistent, or recurrent cases of condyloma.

Malignant transformation is a concern but occurs only rarely. Surgical treatment includes excision by one of three techniques (sharp excision with scissors, laser vaporization, or electrocautery excision and fulguration).

Electrocautery is the preferred therapy and is associated with a recurrence rate of 10%– 25%. With close follow-up and treatment of any recurrence, 95% of patients are rendered disease free.

There is a risk of surgeon infection within the respiratory tract from the inhalation of vaporized viral particles. This risk can be minimized by using smoke evacuators and special masks during procedures.

242
Q

A 60-year-old woman complains of air and stool coming from her vagina. Digital rectal examination reveals an area of induration in the rectovaginal septum, although contrast barium enema does not demonstrate any abnormality. Which of the following statements regarding her probable condition is true?

A. Eighty-five percent of fistulas caused by obstetric trauma heal spontaneously.

B. Low rectovaginal fistulas may be treated effectively by primary fistulotomy.

C. Rectovaginal fistulas from Crohn’s disease usually require proctectomy.

D. Repairs of radiation-induced rectovaginal fistulas generally include a stoma.

E. High rectovaginal fistulas respond well to fibrin glue.

A

ANSWER: D

COMMENTS: Rectovaginal fistulas are classified according to location and cause, which influences the type of corrective surgery required. High fistulas require an abdominal approach, whereas low fistulas can be repaired through a transanal, transperineal, or transvaginal approach. The common causes of these fistulas include obstetrical injuries, primary or recurrent cancer, radiation treatment of pelvic cancers, inflammatory bowel disease, trauma, or infection (e.g., cryptoglandular infection, tuberculosis, or lymphogranuloma venereum).

Five percent of all vaginal deliveries are accompanied by third- or fourth-degree perineal lacerations.

Approximately 10% of these repairs become disrupted and result in incontinence and/or a rectovaginal fistula.

Approximately 50% of these obstetric fistulas heal spontaneously (not 85%); therefore if the patient’s symptoms are not disabling, a 3- to 6-month waiting period is recommended. This waiting period also allows the tissue inflammation and edema to subside before surgical intervention.

Repair of a fistula secondary to an obstetric injury can be performed transvaginally or transrectally.

With the former, the tract is excised and the rectovaginal septum is inverted with serial purse-string sutures.

With the latter, a flap consisting of mucosa, submucosa, and muscularis is advanced to cover the rectal side of the fistula. A diverting colostomy is not required unless previous surgical attempts have failed.

A low ano-vaginal fistula may be treated by fistulotomy, but rectovaginal fistulas (even distal ones) should not be treated by this method.

Partial or total incontinence may result if the fistula tract is divided.

High rectovaginal fistulas are best treated through a transabdominal approach so that coexisting pathologic conditions such as diverticulitis, cancer, or inflammatory bowel disease can be addressed.

The rectovaginal septum is mobilized, the fistula is divided, the vagina is closed, and normal tissue (such as omentum or a muscle flap) is used to buttress the repair.

Because the colon is not usually normal (inflammation or radiation injury), bowel resection is generally necessary.

Fistulas secondary to Crohn’s disease do not always require proctectomy if the symptoms are minimal, the rectum is relatively healthy, and continence is normal.

In such cases, an advancement flap can lead to healing. When there is refractory rectal Crohn’s disease (especially the stricturing form) or incontinence, proctectomy with a colostomy is often ultimately needed.

Radiation- induced fistulas may be very difficult to treat since the radiated tissue does not heal well. Repair requires use of nonradiated tissue, and so may include partial proctectomy with left colon-to- rectum anastomosis and/or use of an omental or gracilis muscle flap.

A colostomy or ileostomy as sole therapy (e.g., a poor-risk patient with recurrent, unresectable cancer or multiple failed attempts at repair) or to divert stool from an anastomosis after resection of diseased bowel is often necessary.

243
Q

A 65-year-old woman presents to your office reporting 1 year of fecal incontinence. She states that she experiences the loss of loose stools twice per week. She typically passes five loose or watery stools daily. She has a history of four vaginal deliveries including at least one episiotomy. She had a normal screening colonoscopy 1 year ago. The most appropriate initial treatment for these symptoms is:

A. Colostomy

B. Bulk-forming agents and loperamide

C. Sacral nerve stimulation

D. Artificial bowel sphincter

E. Overlapping sphincteroplasty

A

ANSWER: B

COMMENTS: Fecal incontinence is commonly related to obstetric injuries, which result in a sphincter defect. However, the quality of the stool is also a common contributing cause of incontinence.

A patient with marginal sphincter function may experience incontinence when the stool is loose or watery.

Treatment with bulk- forming agents, such as psyllium husk, and/or loperamide will bulk the stool.

Treatment of the diarrhea will cure the symptoms of incontinence in many patients. If unsuccessful, then physical therapy including biofeedback may be employed.

Sphincter reconstruction and sacral nerve stimulation are well documented to improve symptoms; however, they should be reserved for patients who do not improve with medical therapy and bowel management.

244
Q

A 47-year-old man presents with a 2.5-cm anal mass arising just proximal to the dentate line, involving > 1/2 of the circumference of the anal canal, and fixed to the internal sphincter muscle. Biopsies show squamous cell carcinoma. Metastatic evaluation is negative. What treatment would you recommend for this patient?

A. Low anterior resection with coloanal anastomosis

B. High-resolution anoscopy

C. 5-Flourouracil, mitomycin C, and radiation therapy

D. APR with colostomy

E. Wide local excision with clear margins and close surveillance

A

ANSWER: C

COMMENTS: This is a localized anal canal squamous cell carcinoma.

TNM staging of anal carcinoma includes the size of the primary lesion.

T1 lesions are 2 cm or less in diameter;

T2 are > 2 cm to 5 cm,

T3 are > 5 cm, and

T4 are invading into an adjacent structure beyond the sphincter muscles.

The lesion is staged as T2, N0, M0.

Endorectal ultrasound or magnetic resonance imaging (MRI) may further define the depth of penetration, extension to other organs, and lymph node status.

It is important to examine the inguinal lymph nodes since this is a common site of spread.

The primary treatment of localized squamous cell carcinoma of the anal canal was defined by Norman Nigro, M.D. (the Nigro protocol), who demonstrated that combined chemoradiation therapy was much more effective than radical surgery (APR), with high rates of anal preservation.

The modified Nigro protocol in use today includes a course of chemotherapy using 5-fluorouracil (5-FU) and mitomycin C and 6 weeks of radiation therapy to a dose of about 5400 cGy.

Anal canal carcinoma must be differentiated from anal margin carcinoma.

Any squamous cell carcinoma that has any part within the anal canal (from the anal verge to the anorectal ring) is considered an anal canal carcinoma, while anal margin lesions are entirely outside of the anal canal in the perineal skin.

Anal margin carcinomas are treated by local excision with clear margins, which results in a high rate of cure.

APR is reserved for persistent or recurrent cancer or if the patient has previously received radiation therapy for other indications.

The modified Nigro protocol results in an initial complete response in 90% of patients.

Salvage APR for persistence or recurrence results in a cure rate of 25%–50%.

So, the overall 5-year disease-free survival for anal canal carcinoma is about 80%.

245
Q

A 32-year-old male with third-degree internal hemorrhoids underwent an operative hemorrhoidectomy in the surgical center and was discharged home. What is the most common early postoperative complication after hemorrhoidectomy?

A. Urinary retention 
B. Bleeding
C. Fecal Incontinence 
D. Perirectal abscess 
E. Anal fissure
A

ANSWER: A

COMMENTS: The most common early postoperative complication after hemorrhoid surgery is urinary retention. This typically occurs in 10% (2%–36%) of patients.

The incidence of postoperative urinary retention is more common in men and is directly related to the volume of intravenous fluids administered.

In addition, it may be related to narcotic administration and regional anesthesia.

Other complications include bleeding (0.03%–6%), anal stenosis (0%–6%), infection (0.5%–5%), and incontinence (2%–12%).

Bleeding is the next most common complication of stapled hemorrhoidopexy.

Other complications unique to stapled hemorrhoidopexy include rectovaginal fistula, rectal perforation, rectal occlusion, and chronic pain related to an inappropriately distal staple line.

246
Q

A 47-year-old man with poorly controlled diabetes mellitus presents to your clinic complaining of anal bulging and bleeding with bowel movements. Anoscopy shows a single left lateral, second degree internal hemorrhoid. You perform rubber band ligation without difficulty, and the patient is sent home. The following evening your answering service receives a call from the patient who is complaining of fever, anal pain, and difficulty urinating. What is your recommendation?

A. Encourage increased fluid intake, stool softeners, Sitz baths, and oral analgesics.

B. Tell the patient to call his primary care physician.

C. Ask the patient to return to the clinic in the morning.

D. Perform an examination under anesthesia urgently.

E. Remove the band in the office.

A

ANSWER: D

COMMENTS: Rubber band ligation is commonly performed for first- and second-degree hemorrhoids.

The band is placed on the redundant mucosa a minimum of 2 cm proximal to the dentate line.

This will slough off in 5-7 days and can be associated with light bleeding.

Complications of banding include pain, thrombosis, bleeding, and perineal or pelvic sepsis. Placing the band too close to the dentate line will cause immediate pain and require removal.

Thrombosis of the internal, external, or both complexes may be managed by observation and analgesics if mild or by operative hemorrhoidectomy if extensive.

Significant bleeding is abnormal and may require control of the bleeding site in the office or the operating room (OR).

Life-threatening sepsis due to necrotizing infection is rare but mandates immediate evaluation and treatment (fluid, antibiotics, and examination in the OR).

This sepsis is associated with severe diabetes and states of immune suppression. Early recognition of this problem is critical, and treatment must be rapid.

247
Q

A 45-year-old male with a 4-cm anal mass above the dentate line is found to have squamous cell carcinoma on biopsy. In this patient, which nodal basin is likely to be the initial site of spread?

A. Inferior mesenteric artery nodes 
B. Internal iliac nodes
C. Inguinal nodes
D. Para-aortic nodes
E. Sister Mary Joseph node
A

ANSWER: B

COMMENTS: Lymphatic drainage of the upper and middle rectum includes the inferior mesenteric and para-aortic nodes.

Drainage from the lower one-third of the rectum occurs partially in a cephalad direction toward the inferior mesenteric artery, but also laterally along the middle hemorrhoidal vessels to the internal iliac nodes.

Anal canal lymphatic drainage is defined by the relationship of the lesion to the dentate line.

Superior to the dentate line, drainage occurs to the inferior mesenteric and internal iliac nodes.

Distal to or below the dentate line, drainage occurs along the inferior lymphatics to the superficial inguinal nodes.

The Sister Mary Joseph node is a periumbilical nodule and often due to metastatic gastric or pancreatic cancer.

248
Q

A 28-year-old Caucasian man presents with a 9-month history of anal pain during and after defecation with a trace of red blood on the toilet tissue. On anoscopy, there is an irregular fibrotic tear in the anoderm in the right lateral position. Further evaluation of this problem should include:

A. Colonoscopy

B. QuantiFERON gold test

C. Complete blood count

D. HIV testing

E. All of the above

A

ANSWER: E

COMMENTS: Chronic anal fissures in the lateral position are rare and should prompt consideration of alternative diagnoses.

These include Crohn’s disease, leukemia, tuberculosis, syphilis, HIV, or anal carcinoma.

The QuantiFERON gold test is useful for diagnosing both active and latent Mycobacterium tuberculosis infection.

Medical treatments such as bowel management, Sitz baths, topical lidocaine, topical nitroglycerin, or nifedipine may be initiated for pain relief and to promote healing.

However, sphincterotomy should not be recommended until other causative disorders are identified and treated.

Sphincterotomy in the setting of untreated Crohn’s disease or HIV may result in nonhealing wounds and/or fecal incontinence.

249
Q

An 89-year-old multiparous female with chronic obstructive pulmonary disease (COPD) presents with a 12-month history of a worsening anal bulge and fecal incontinence. On examination, she has a 5-cm full-thickness rectal prolapse. It is easily reduced but spontaneously reprolapses. What is the best surgical treatment for this patient?

A. Low anterior resection
B. Perineal rectosigmoidectomy
C. Transabdominal mesh rectopexy
D. Laparoscopic sigmoid resection
E. Robotic sigmoid resection and mesh rectopexy
A

ANSWER: B

COMMENTS: Full-thickness rectal prolapse involves all layers of the rectal wall.

It is often associated with fecal incontinence, particularly in elderly multiparous patients.

Initial treatment includes high-fiber diet and stool softeners to prevent straining.

This may reduce the severity of the prolapse but will not cure it.

In healthy patients with constipation, sigmoid resection with sutured rectopexy (Frykman procedure) can result in low rates of recurrence and improvement in constipation.

Patients with fecal incontinence ideally should not undergo sigmoid resection, as this may worsen this symptom.

Abdominal procedures should be avoided in older frail patients because of the risks.

These patients are better suited for a perineal approach such as a perineal rectosigmoidectomy (Altemeier procedure).

This is a low-risk operation that may be performed under regional anesthesia or even sedation with local anesthesia.

The redundant rectum and colon are resected from the perineal approach with a coloanal anastomosis.

A levateroplasty may be added to tighten up the levator hiatus and possibly improve incontinence.

Although the loss of the rectal vault may worsen incontinence and frequency and the recurrence rate is higher than with abdominal procedures, the very low risk and rapid recovery make this the best operation in this group of patients.

250
Q

A 42-year-old previously healthy man has had a 4-day history of increasing rectal pain and difficulty sitting.

Examination shows a 3-cm area of erythema and fluctuance consistent with a perianal abscess. Which of the following is true regarding the anatomic spaces in which this process may occur?

A. The supralevator space is situated above the levator ani muscles and is connected with the contralateral side anteriorly.

B. The retrorectal space lies between the rectum and the sacrum but below the rectosacral fascia.

C. The deep postanal space lies below the levator ani muscles and posterior to the external sphincter muscles.

D. The perianal space and the superficial postanal space lie deep to the superficial portion of the external anal sphincter.

E. The intersphincteric space lies just outside the conjoined longitudinal muscle.

A

ANSWER: C

COMMENTS: The supralevator space is bounded superiorly by the peritoneum, laterally by the pelvic walls, medially by the rectum, and inferiorly by the levator ani muscles (Figs. 23B.1 and 23B.2).

It communicates with the contralateral side posteriorly. Infection in this space can arise from an abdominal or pelvic source (e.g., diverticulitis or pelvic inflammatory disease) or as an upward extension from an anorectal source.

The retrorectal space lies above the rectosacral fascia between the upper two-thirds of the rectum and sacrum.

The fascia runs downward and forward from the sacrum to the anorectal junction.

The retrorectal space contains loose connective tissue and is a site for the formation of tumors arising from embryologic remnants (i.e., dermoids, teratomas, and chordomas).

The retrorectal space is bounded anteriorly by the rectum, posteriorly by the presacral fascia, laterally by the pelvic side wall, superiorly by the peritoneal reflection, and inferiorly by the rectosacral fascia, below which is the supralevator space.

The ischiorectal space lies below the levator muscle, above the transverse septum of the ischiorectal fossa, and between the external sphincter and the lateral pelvic wall.

This space communicates posteriorly through the deep postanal space, which lies between the levator ani and the superficial external sphincter.

The lower border of the deep postanal space is the anococcygeal ligament, which originates from the superficial portion of the external sphincter in the posterior midline.

This communication allows an abscess in the deep postanal space to extend to both ischiorectal spaces (horse- shoe abscess).

The perianal space (the most common space involved in abscesses) lies superficial to the superficial external anal sphincter.

The intersphincteric space lies within the conjoined longitudinal muscle, where the anal glands are also located.

The perianal, ischiorectal, and supralevator spaces may connect posteriorly with their counterparts on the contralateral side to form a horseshoe connection in any of these spaces.

251
Q

A 55-year-old man presents to the clinic with a long-standing anal fissure with pain on defecation and red streaking after bowel movements. He has failed conservative measures. On anoscopy you note a small posterior midline fissure with surrounding fibrosis, a skin tag, and a hypertrophied
papilla. You plan a lateral internal sphincterotomy. In performing this procedure, you will divide a continuation of which muscle?

A. Puborectalis muscle

B. Muscularis propria of the rectum

C. Levator ani muscle

D. Conjoined longitudinal muscle

E. Subcutaneous external anal sphincter muscle

A

ANSWER: B

COMMENTS: The internal anal sphincter represents the distal 2.5 to 4.0 cm of the circular muscle layer (muscularis propria) of the rectum.

This sphincter is therefore smooth muscle and under autonomic control.

In its basal state, it is contracted, creating a natural barrier to the loss of stool and gas.

The external anal sphincter is a continuation of the puborectalis muscle. This sphincter is a striated muscle, and like the other pelvic floor muscles, it is maintained by an unconscious resting tone at the level of the cauda equine but can be voluntarily relaxed and contracted.

The conjoined longitudinal muscle combines with fibers of the levator ani and then descends and crosses the external anal sphincter to insert into the perianal skin.

252
Q

A 33-year-old woman with a history of multiple pregnancies and irritable bowel syndrome (IBS) presents to the clinic with an anal bulge. She has a history of bulging with bowel movements, fecal leakage, and occasional bright red bleeding on the toilet paper. On anoscopy, you note two enlarged hemorrhoid plexuses: one left lateral and one right posterior. Both prolapse with Valsalva and reduce spontaneously. What degree do these represent?

A. First-degree hemorrhoids
B. Second-degree hemorrhoids 
C. Third-degree hemorrhoids 
D. Fourth-degree hemorrhoids 
E. Rectal prolapse
A

ANSWER: B

COMMENTS: The internal hemorrhoidal tissues originate proximal to the dentate line and are vascular structures that are covered by mucosa (transitional and columnar epithelium).

The anal canal from 1 to 2 cm above the dentate line is viscerally innervated and is insensitive to pain, touch, or temperature, and therefore amenable to office procedures without the need for anesthetics.

Internal hemorrhoids are classified by 4 degrees depending on the amount of prolapse.

Table 23B.1 reviews the differences in internal hemorrhoids by degree.

253
Q

An 18-year-old otherwise healthy college student presents to your office complaining of chronic, recurrent pain, and drainage from a wound at the superior aspect of his gluteal cleft. On examination, he has a single midline pit with minimal serous drainage and no surrounding erythema. There are no surrounding sinus tracts. He has never
received medical attention for this issue. The most appropriate management is:

A. A cleft lift procedure
B. Observation
C. Topical clindamycin
D. Excision with off-midline closure 
E. Incision and drainage
A

ANSWER: D
COMMENTS: This patient suffers from pilonidal disease.

This is mostly likely due to hair follicles that become distended with keratin and then subsequently infected, rupturing into the subcutaneous tissue.

This is most likely due to micro-trauma related to the weight of the buttocks having its greatest support (and effect) at the midline skin of the gluteal cleft.

The patient does not have an abscess or cellulitis. Therefore drainage or antibiotics are not indicated.

Pilonidal disease will cause recurrent symptoms unless definitively treated with excision closure of the midline pit(s) and elimination of the associated epithelialized sinus tract extending from the pit to the abscess cavity.

This may be accomplished by excision removing the midline pit, epithelialized tract, and abscess cavity.

Excision may be accompanied by leaving the wound open. This is an acceptable treatment option; however, it requires dressing changes that are difficult for patients to perform themselves.

The cleft lift procedure has been demonstrated to achieve high rates of cure, with few wound complications; however, as the initial procedure for uncomplicated disease is not indicated, it may be overly aggressive.

As the pilonidal is not infected, incision and drainage is not indicated.

Simple excision and closure of the wound is indicated.

Off-midline closure has been established as the preferred technique compared with midline closure, as this reduces the tension on the wound caused by the weight of the buttocks.

254
Q

A 74-year-old man with biopsy-proven rectal adenocarcinoma is undergoing a low anterior resection. Which layers must be stapled through when resecting the distal portion of resection specimen?

A. Mucosa, submucosa, circular muscle layer, longitudinal muscle layer, and serosa

B. Mucosa, submucosa, longitudinal muscle layer, circular muscle layer, and serosa

C. Mucosa, submucosa, longitudinal muscle layer, and circular muscle layer

D. Mucosa, submucosa, circular muscle layer, and serosa

A

Answer: C

The wall of the colon and rectum are made of five separate layers: mucosa, submucosa, circular muscle layer, longitudinal muscle layer, and serosa.

The mid and lower rectum lack serosa so this layer would not be stapled through if the surgeon were stapling through the mid or lower rectum. (See Schwartz 10th ed., p. 1176.)

255
Q

Which layer of muscle joins together to form the internal anal sphincter?

A. Circumferential muscle layer
B. Longitudinal muscle layer
C. Puborectalis muscle
D. Circular muscle layer

A

Answer:A

The inner circular muscle joins to form the internal anal sphincter.

The subcutaneous, superficial, and deep external sphincter surrounds it.

The deep external anal sphincter is an extension of the puborectalis muscle. (See Schwartz 10th ed., p. 1176.)

256
Q

A 24-year-old woman with medically refractory ulcerative colitis decides to undergo a total colectomy. During this procedure, where would it be most appropriate to look for the inferior mesenteric vein in order to ligate it?

A. Look for the inferior mesenteric artery; the veins of the colon usually parallel with the corresponding arteries.

B. The inferior mesenteric artery can be ligated within the peritoneum, where it joins with the superior mesenteric artery.

C. The inferior mesenteric vein is often ligated at the inferior edge of the pancreas, just below where it joins with the splenic vein.

D. The inferior mesenteric vein will not be ligated for this procedure.

A

Answer: C
The inferior mesenteric vein does not run with the inferior mesenteric artery.

Instead, it travels cranially in the retroperitoneum over the psoas and then posterior to the pancreas to join the splenic vein.

The vein is often ligated at the inferior edge of the pancreas during a colectomy. (See Schwartz 10th ed., p. 1177.)

257
Q

An anatomy class is dissecting out the rectum. What are the correct fascial arrangements that they will encounter during this dissection?

A. The presacral fascia separates the rectum from the pre-sacral venous plexus and the pelvic nerves; Waldeyer fascia extends forward and downward and attaches to the fascia propria at the anorectal junction. Denonvilliers fascia separates the rectum from the prostate and seminal vesicles in men and from the vagina in women.

B. The presacral fascia extends forward and downward and attaches to the fascia propria at the anorectal junction; Waldeyer fascia separates the rectum from the prostate and seminal vesicles in men and from the vagina in women; Denonvilliers fascia separates the rectum from the presacral venous plexus and the pelvic nerves.

C. The presacral fascia separates the rectum from the prostate and seminal vesicles in men and from the vagina in women; Waldeyer fascia extends forward and downward and attaches to the fascia propria at the anorectal junction; Denonvilliers fascia separates the rectum from the presacral venous plexus and the pelvic nerves.

D. The presacral fascia separates the rectum from the prostate and seminal vesicles in men and rom the vagina in women; Waldeyer fascia extends backward and downward and attaches to the fascia propria at the anorectal junction; Denonvilliers fascia separates the rectum from the presacral venous plexus and the pelvic nerves.

A

Answer: A

The presacral fascia separates the rectum from the presacral venous plexus and the pelvic nerves.

The rectosacral fascia (Waldeyer fascia) extends forward and downward and attaches to the fascia propria at the anorectal junction.

Anteriorly, Denonvilliers fascia separates the rectum from the prostate and seminal vesicles in men and from the vagina in women. (See Schwartz 10th ed., p. 1177.)

258
Q

Choose the correct definition of intestinal malrotation:

A. At the 4th week of gestation the midgut herniates through the abdominal cavity, rotates 270° clockwise around the superior mesenteric artery and then travels to its resting place in the abdomen during the 10th week.

B. At the 4th week of gestation the midgut herniates through the abdominal cavity, rotates 270° counter-clockwise around the superior mesenteric artery and then travels to its resting place in the abdomen during the 12th week.

C. At the 6th week of gestation the midgut herniates through the abdominal cavity, rotates 270° clockwise around the superior mesenteric artery and then travels to its resting place in the abdomen during the 12th week.

D. At the 6th week of gestation the midgut herniates through the abdominal cavity, rotates 270° counter-clockwise around the superior mesenteric artery and then travels to its resting place in the abdomen during the 10th week.

A

Answer: D

During the 6th week of gestation, the midgut herniates out of the abdominal cavity and rotates 270° counterclockwise around the superior mesenteric artery and travels to its resting position in the 10th week of gestation.

Failure of the mid-gut to rotate and return to the abdominal cavity during the 10th week of gestation results in intestinal malrotation. (See Schwartz 10th ed., pp. 1175, 1179.)

259
Q

A 62-year-old man has perforated diverticulitis and undergoes an emergent left hemicolectomy with a diverting loop ileostomy. If he has a high output ileostomy and is at risk of diversion colitis, which fatty acids are not being absorbed?

A. Butyric acid and propionic acid

B. Propionic acid and palmitic acid

C. Tricosic acid and butyric acid

D. Lauric acid and palmitic acid

A

Answer: A

Short-chain fatty acids (acetic acid, butyric acid, and propionic acid) are produced by bacterial fermentation of dietary carbohydrates and are an important source of energy for the colonic mucosa, and metabolism by colonocytes provides energy for processes such as active transport of sodium.

Diversion of feces by an ileostomy or colostomy can result in “diversion colitis” which is associated with mucosal atrophy and inflammation. (See Schwartz 10th ed., p. 1179.)

260
Q

A 58-year-old mother of 10 suffers from fecal incontinence. Usually, defecation occurs by increased intraabdominal pressure via the Valsalva maneuver, increased rectal contraction, and relaxation of the puborectalis muscle, which forms a “sling” around the distal rectum, forming a relatively acute angle that distributes intra-abdominal forces onto the pelvic floor.

With defecation, this angle straightens, allowing downward force to be applied along the axis of the rectum and anal, and opening of the anal canal. A dysfunction at which point of this pathway can lead to fecal incontinence?

A. Injury to the puborectalis.

B. Decreased rectal contraction.

C. Repair of the internal or external sphincter during delivery.

D. Hypertrophic internal and external anal sphincters.

A

Answer: A

Defecation proceeds by coordination of increasing intra-abdominal pressure via the Valsalva maneuver, increased rectal contraction, relaxation of the puborectalis muscle, and opening of the anal canal.

Impaired continence may result from poor rectal compliance, injury to the internal and/or external sphincter or puborectalis, or nerve damage or neuropathy. (See Schwartz 10th ed., p. 1180.)

261
Q

A healthy 48-year-old physician with no family history of cancer and who strictly adheres to a high protein, high fiber diet, exercises five times per week or 50 minutes, and takes vitamin C supplements daily performs a fecal occult blood test (FOBT) on herself and tests positive. Should she have any further colon screening?

A. No, vitamin C can produce a false-positive result.

B. Yes, all positive FOB requires further investigation with a colonoscopy.

C. Yes, all positive FOB requires further investigation with FOB in 1 year.

D. No, she has no risk factors or colon cancer and should
follow the USPSTF screening guidelines for colorectal cancer.

A

Answer: B

Fecal occult blood test (FOBT) has been a nonspecific test for peroxidase contained in hemoglobin; consequently, occult bleeding from any gastrointestinal source will produce a positive result.

Similarly, many foods (red meat, some fruits and vegetables, and vitamin C) will produce a false-positive result.

Any positive FOB mandates further investigation, usually by colonoscopy. (See Schwartz 10th ed., p. 1182.)

262
Q

A 22-year-old college student presents to clinic with a history of intermittent diarrhea for the past 5 days after returning from Mexico. On further questioning, she has had previous episodes of diarrhea or the past 2 years, unrelated to travel. After a physical examination, what are appropriate tests that should be ordered to appropriately work up this patient?

A. Stool wet-mount and stool culture.

B. Sigmoidoscopy and colonoscopy, but only if no peritoneal signs on physical examination.

C. Add Sudan red to stool sample.

D. All of the above.

A

Answer: D

Stool wet-mount and culture can often diagnose infection. Sigmoidoscopy or colonoscopy can be helpful in diagnosing inflammatory bowel disease for ischemia.

However, if the patient has abdominal tenderness, particularly with peritoneal signs, or any other evidence of perforation, endoscopy is contraindicated.

For chronic diarrhea, tests for malabsorption and metabolic investigations should be conducted along with colonoscopy.

Biopsies should be taken even if the colonic mucosa appears grossly normal. (See Schwartz 10th ed., p. 1184.)

263
Q

A 76-year-old man undergoes an emergent sigmoidectomy for a perforated colon mass. The surgeon performs a Hartmann procedure and brings up a colostomy. In an emergency setting, where is the most appropriate location to seat a colostomy?

A. Above the beltline, within the rectus abdominus muscle, away from the costal margin

B. Below the beltline, within the rectus abdominus muscle, near the iliac crest

C. Above the beltline, within the rectus abdominus muscle, near the costal margin

D. Below the beltline, within the rectus abdominus muscle, away from the iliac crest

A

Answer: A

In an emergency operation, like this one, where the stoma site has not been marked, an attempt should be made to place a stoma within the rectus muscle and away from both the costal margin and iliac crest.

In emergencies, placement high on the abdominal wall is preferred to a low-lying site. (See Schwartz 10th ed., p. 1192.)

264
Q

A previously healthy 46-year-old woman with a history of rectal adenocarcinoma, first discovered on colonoscopy 1 year ago who is status post low anterior resection with a diverting loop ileostomy returns to clinic 3 months after her low anterior resection for a preoperative appointment for her ileostomy reversal. Over the past 3 months she has had good ileostomy output as well as occasional loose stools per rectum. What workup does she need to have prior to ileostomy reversal?

A. A digital rectal examination to palpate the anastomosis and check for patency.

B. No examination is needed as this was a simple diversion and she has continued to pass stool per rectum.

C. A flexible sigmoidoscopy or contrast enema to check for patency.

D. A colonoscopy to evaluate for polyps not previously seen on previous colonoscopy.

A

Answer: C
A flexible endoscopic examination and a contrast enema (Gastrografin) are recommended prior to closure to ensure that the anastomosis has not leaked and is patent. (See Schwartz 10th ed., p. 1193.)

265
Q

A 75-year-old woman undergoes a right hemicolectomy and end ileostomy for right-sided perforated diverticulitis. What is the most concerning adverse outcome in the short term of this procedure and will require revision?

A. Skin breakdown caused by succus entericus.

B. Stoma necrosis above the level of the fascia.

C. Stoma necrosis below the level of the fascia.

D. Stomal retraction below the level of the fascia.

A

Answer: C

Stoma necrosis may occur in the early postoperative period and is usually caused by skeletonizing the distal small bowel and/or creating an overly tight fascial defect.

Limited mucosal necrosis above the fascia may be treated expectantly, but necrosis below the level of the fascia requires surgical revision.

Stoma retraction may occur early or late and may be exacerbated by obesity. Local revision may be necessary. (See Schwartz 10th ed., p. 1193.)

266
Q

A 19-year-old man with medically refractory ulcerative colitis undergoes a total colectomy with J-pouch creation. What are some of the late complications of ileal pouch- anal reconstruction?

A. More than eight bowel movements per day 
B. Nocturnal incontinence
C. Pouchitis
D. Small bowel obstruction
E. All of the above
A

Answer: E

The functional outcome of ileal pouch-anal reconstruction is not always perfect.

Patients should be counseled to expect 8 to 10 bowel movements per day.

Up to 50% have some degree of nocturnal incontinence.

Pouchitis occurs in nearly 50% of patients who undergo the operation or chronic ulcerative colitis, and small bowel obstruction is common.

Pouches fail in 5 to 10% of patients.

267
Q

A 50-year-old woman who underwent a total colectomy with ileal pouch-anal reconstruction 5 years ago presents to the emergency room with diarrhea, fever, 2 weeks of malaise, and severe abdominal pain. What is the most appropriate differential diagnosis?

A. Parasitic infection, ulcerative colitis of the remaining rectal cuff, undiagnosed Crohn disease.

B. Bacterial or viral infection, undiagnosed Crohn disease, and pouchitis.

C. Rectal cancer of remaining rectal cuff, bacterial or viral infection, and undiagnosed Crohn disease.

D. Parasitic infection, bacterial or viral infection, and pouchitis.

A

Answer: B

This patient is likely presenting with pouchitis.

Pouchitis is an inflammatory condition that affects both ileoanal pouches and continent ileostomy reservoirs.

The incidence of pouchitis ranges from 30 to 55%.

Symptoms include increased diarrhea, hematochezia, abdominal pain, fever, and malaise.

Diagnosis is made endoscopically with biopsies.

Differential diagnosis includes infection and undiagnosed Crohn disease.

(See Schwartz 10th ed., p. 1194.)

268
Q

A 68-year-old man is undergoing a right hemicolectomy for a cecal mass. He asks what the current research has shown about decreasing postoperative infection after this procedure. When should antibiotics always be used for this procedure?

A. Oral antibiotics should be used in combination with bowel preparation.

B. Parenteral antibiotic prophylaxis at the time of surgery after the skin incision is made and redosed as needed during the procedure.

C. Parenteral antibiotic prophylaxis at the time of surgery before the skin incision is made.

D. Oral antibiotics should be used postoperatively to decrease risk of anastomosis leak.

A

Answer: C

Prospective randomized trials are needed to better understand the role of oral antibiotic prophylaxis in colorectal surgery.

In contrast, long-standing, convincing data support the efficacy of parenteral antibiotic prophylaxis at the time of surgery.

Broad-spectrum parenteral antibiotic(s) with activity against aerobic and anaerobic enteric pathogens should be administered just prior to the skin incision and redosed as needed depending on the length of the operation.

There is no proven benefit to using antibiotics postoperatively after an uncomplicated colectomy.

269
Q

A 22-year-old woman presents to the clinic with a 3-year history of bloody diarrhea, abdominal pain, and anorectal fistulas. Her father had similar symptoms during his 20’s and has had multiple abdominal surgeries. What is the percentage of patients with this disease who have family members with the same disease?

A. 5–10%
B. 10–20%
C. 10–30%
D. 20–40%

A

Answer: C

Family history may play a role in inflammatory bowel disease as 10 to 30% of patients within inflammatory bowel disease report a family member with the same disease. (See Schwartz 10th., p. 1195.)

270
Q

A 25-year-old man is undergoing workup to determine if he has ulcerative colitis, Crohn disease, or indeterminate colitis. What diagnostic findings would indicate that he has Crohn disease.

A. Atrophic mucosa, crypt abscesses, inflammatory pseudopolyps, scarred and shortened colon, continuous involvement of rectum and colon.

B. Mucosal ulcerations, noncaseating granulomas, fibrosis, strictures, and fistulas in the colon with deep serpiginous ulcers.

C. Atrophic mucosa, noncaseating granulomas, strictures, “cobblestone” appearance on endoscopy.

D. Mucosal ulcerations, crypt abscesses, inflammatory pseudopolyps, continuous involvement of colon and rectum

A

Answer: B

Ulcerative colitis is a mucosal process in which the colonic mucosa and submucosa are infiltrated with inflammatory cells.

The mucosa may be atrophic, and crypt abscesses are common.

Endoscopically, the mucosa is requently friable and may possess multiple inflammatory pseudopolyps.

In long-standing ulcerative colitis, the colon may be foreshortened and the mucosa replaced by scar.

A key feature of ulcerative colitis is the continuous involvement of the rectum and colon; rectal sparing or skip lesions suggest a diagnosis of Crohn disease.

Crohn disease is a transmural inflammatory process that can affect any part of the gastrointestinal tract from mouth to anus.

Mucosal ulcerations, an inflammatory cell infiltrate, and noncaseating granulomas are characteristic pathologic findings. Chronic inflammation may ultimately result in fibrosis, strictures, and fistulas in either the colon or small intestine.

The endoscopic appearance of Crohn colitis is characterized by deep serpiginous ulcers and a “cobblestone” appearance. (See Schwartz 10th ed., p. 1195.)

271
Q

What structures are most likely to be site of extracolonic disease in inflammatory bowel disease?

A. Liver, biliary tree, joints, skin, eyes

B. Biliary tree, lungs, heart, spleen

C. Joints, skin, biliary tree, bladder

D. Skin, liver, pancreas, joints, eyes

A

Answer: A

The liver is a common site of extracolonic disease in inflammatory bowel disease.

Fatty infiltration of the liver is present in 40 to 50% of patients, and cirrhosis is found in 2 to 5%.

Primary sclerosing cholangitis is a progressive disease characterized by intra- and extrahepatic bile duct strictures.

Forty to 60% of patients with primary sclerosing cholangitis have ulcerative colitis.

Pericholangitis is also associated with inflammatory bowel disease and may be diagnosed with a liver biopsy.

Bile duct carcinoma is a rare complication of long-standing inflammatory bowel disease.

Arthritis also is a common extracolonic manifestation of inflammatory bowel disease, and the incidence is 20 times greater than in the general population.

Erythema nodosum is seen in 5 to 15% of patients with inflammatory bowel disease and usually coincides with clinical disease activity.

Pyoderma gangrenosum is an uncommon but serious condition that occurs almost exclusively in patients with inflammatory bowel disease.

Up to 10% of patients with inflammatory bowel disease will develop ocular lesions.

These include uveitis, iritis, episcleritis, and conjunctivitis. (See Schwartz 10th ed., p. 1196.)

272
Q

An 18-year-old woman is undergoing workup to determine if she has ulcerative colitis, Crohn disease, or indeterminate colitis. What diagnostic findings would indicate that she has ulcerative colitis?

A. Atrophic mucosa, crypt abscesses, inflammatory pseudopolyps, scarred and shortened colon, continuous involvement of rectum and colon.

B. Mucosal ulcerations, noncaseating granulomas, fibrosis, strictures, and fistulas in the colon with deep serpiginous ulcers.

C. Atrophic mucosa, noncaseating granulomas, strictures, “cobblestone” appearance on endoscopy.

D. Mucosal ulcerations, crypt abscesses, inflammatory pseudopolyps, continuous involvement of colon and rectum.

A

Answer:A

Ulcerative colitis is a mucosal process in which the colonic mucosa and submucosa are infiltrated with inflammatory cells.

The mucosa may be atrophic, and crypt abscesses are common.

Endoscopically, the mucosa is frequently friable and may possess multiple inflammatory pseudopolyps.

In long- standing ulcerative colitis, the colon may be foreshortened and the mucosa replaced by scar.

A key feature of ulcerative colitis is the continuous involvement of the rectum and colon; rectal sparing or skip lesions suggest a diagnosis of Crohn disease.

Crohn disease is a transmural inflammatory process that can affect any part of the gastrointestinal tract rom mouth to anus.

Mucosal ulcerations, an inflammatory cell infiltrate, and noncaseating granulomas are characteristic pathologic findings.

Chronic inflammation may ultimately result in fibrosis, strictures, and fistulas in either the colon or small intestine.

The endoscopic appearance of Crohn colitis is characterized by deep serpiginous ulcers and a “cobblestone” appearance.

(See Schwartz 10th ed., p. 1195.)

273
Q

The goals of medical therapy for inflammatory bowel disease are to decrease inflammation and alleviate symptoms.

Mild to moderate flares are treated in the clinic and more severe symptoms may require hospitalization.

What is the first-line therapy for inflammatory bowel disease in the outpatient setting?

A. Salicylates, such as sulfasalazine and 5-acetyl salicylic acid

B. Antibiotics, such as metronidazole and fluoroquinolones

C. Corticosteroids

D. Azathioprine and 6-mercaptopurine

A

Answer: A
Sulfasalazine (Azulfidine), 5-acetyl salicylic acid (5-ASA), and related compounds are first-line agents in the medical treatment of mild to moderate inflammatory bowel disease.

These compounds decrease inflammation by inhibition of cyclooxygenase and 5-lipoxygenase in the gut mucosa.

They require direct contact with affected mucosa for efficacy. Multiple preparations are available for administration to different sites in the small intestine and colon.

Antibiotics are often used to decrease the intraluminal bacterial load in Crohn disease.

Metronidazole has been reported to improve Crohn colitis and perianal disease, but the evidence is weak. Fluoroquinolones may also be effective in some cases.

In the absence of fulminant colitis or toxic megacolon, antibiotics are not used to treat ulcerative colitis.

Corticosteroids (either oral or parenteral) are a key component of treatment for an acute exacerbation of either ulcerative colitis or Crohn disease.

Corticosteroids are nonspecific inhibitors of the immune system, and 75 to 90% of patients will improve with the administration of these drugs.

Azathioprine and 6-mercatopurine (6-MP) are antimetabolite drugs that interfere with nucleic acid synthesis and thus decrease proliferation of inflammatory cells.

These agents are useful for treating ulcerative colitis and Crohn disease in patients who have failed salicylate therapy or who are dependent on, or refractory to, corticosteroids. (See Schwartz 10th ed., p. 1196.)

274
Q

A thin and ill appearing 26-year-old man presents to the emergency department (ED) with fevers, chills, severe abdominal pain, and a rigid abdomen. While doing a history and physical, it is noted that he has a history of ulcerative colitis. What would be indications that stoma creation would be more appropriate than a primary anastomosis in this patient?

A. Long-standing history of ulcerative colitis with multiple colon polyps.

B. A prealbumin of 6.0 in a patient who has been on corticosteroids.

C. A blood glucose level of 300 in a patient who finished a corticosteroid course 3 weeks ago.

D. Diarrhea more than 10 times per day for months with albumin of 3.6.

A

Answer: B

Patients with inflammatory bowel disease are often malnourished.

Abdominal pain and obstructive symptoms may decrease oral intake.

Diarrhea can cause significant protein loss.

Ongoing inflammation produces a catabolic physiologic state.

Parenteral nutrition should be strongly considered early in the course of therapy or either Crohn disease or ulcerative colitis.

The nutritional status of the patient also should be considered when planning operative intervention, and nutritional parameters such as serum albumin, prealbumin, and transferrin should be assessed.

In extremely malnourished patients, especially those who are also being treated with corticosteroids, creation of a stoma is often safer than a primary anastomosis. (See Schwartz 10th ed., p. 1197.)

275
Q

A 24-year-old woman presents to the ED with fever, severe abdominal pain with guarding on palpation, and a history of 5 days of bloody stools. She has a history of ulcerative colitis. What are the indications for emergency surgery or ulcerative colitis?

A. Hemorrhage with continued decrease in hematocrit levels in spite of blood transfusion.

B. Hemodynamic instability requiring transfer to the ICU with decline in status over a 48-hour period after admission.

C. Severe abdominal pain and diarrhea that does not respond to bowel rest, hydration, and parenteral corticosteroids.

D. Cecum measured at 9 cm in diameter on computed tomography (CT) scan.

E. All of the above.

A

Answer: E

Emergency surgery is required for patients with massive life- threatening hemorrhage, toxic megacolon, or fulminant colitis who fail to respond rapidly to medical therapy.

Patients with signs and symptoms of fulminant colitis should be treated aggressively with bowel rest, hydration, broad-spectrum antibiotics, and parenteral corticosteroids.

Colonoscopy and barium enema are contraindicated, and antidiarrheal agents should be avoided.

Deterioration in clinical condition or failure to improve within 24 to 48 hours mandates surgery. (See Schwartz 10th ed., p. 1197.)

276
Q

A 55-year-old woman with a history of long-standing Crohn disease presents to the clinic with a 1-month history of abdominal pain and a new area of induration, fluctuance, and foul-smelling drainage from a former midline incision. What are the most common indications for surgery for Crohn disease?

A. Internal fistula or abscess
B. Obstruction
C. Toxic megacolon
D. Strictures

A

Answer: A

The most common indications for surgery are internal fistula or abscess (30–38% of patients) and obstruction (35–37% of patients).

Crohn disease of the large intestine may present as fulminant colitis or toxic megacolon. In this setting, treatment is identical to treatment of fulminant colitis and toxic megacolon secondary to ulcerative colitis.

Resuscitation and medical therapy with bowel rest, broad-spectrum antibiotics, and parenteral corticosteroids should be instituted.

If the patient’s condition worsens or fails to rapidly improve, total abdominal colectomy with end ileostomy is recommended. (See Schwartz 10th ed., p. 1199.)

277
Q

A 23-year-old man presents to the clinic with severe pain on defecation that began 2 months ago. He has tried conservative management at home with sitz baths but his pain has become so severe that he has started to restrict how much he eats to prevent having bowel movements. On rectal examination, a fissure is found. What would indicate that this fissure from Crohn disease?

A. Deep and broad ulcer located in the lateral position.

B. Shallow and broad ulcer located in the anterior position.

C. Deep and narrow ulcer located in the posterior midline position.

D. Shallow and narrow ulcer located in the lateral position.

A

Answer: A

The most common perianal lesions in Crohn disease are skin tags that are minimally symptomatic.

Fissures are also common.

Typically, a fissure from Crohn disease is particularly deep or broad and perhaps better described as an anal ulcer.

These fissures are often multiple and located in a lateral position rather than anterior or posterior midline as seen in an idiopathic fissure-in-ano.

A classic-appearing fissure in ano located laterally should raise the suspicion of Crohn disease.

(See Schwartz 10th ed., p. 1200.)

278
Q

A 65-year-old man presents to the ED with fevers, abdominal pain, and bloody stools for the past 2 days. On CT scan he is found to have diverticulitis with scant free air and a small fluid collection associated with the sigmoid colon. What is the etiology of diverticulosis?

A. Lack of dietary fiber causes smaller stools volume requiring higher intraluminal pressure.

B. Chronic contraction causes muscular hypertrophy and causes colon to act as segments rather than a continuous tube.

C. The mucosa and muscularis mucosa herniate through the colon wall.

D. All of the above.

A

Answer: D
The majority of colonic diverticula are false diverticula in which the mucosa and muscularis mucosa have herniated through the colonic wall.

These diverticula occur between the teniae coli, at points where the main blood vessels penetrate the colonic wall (presumably creating an area of relative weakness in the colonic muscle).

They are thought to be pulsion diverticula resulting from high intraluminal pressure.

The most accepted theory is that a lack of dietary fiber results in smaller stool volume, requiring high intraluminal pressure and high colonic wall tension or propulsion.

Chronic contraction then results in muscular hypertrophy and develop- ment of the process of segmentation in which the colon acts like separate segments instead of functioning as a continuous tube.

As segmentation progresses, the high pressures are directed radially toward the colon wall rather than to development of propulsive waves that move stool distally.

The high radial pressures directed against the bowel wall create pulsion diverticula. (See Schwartz 10th ed., p. 1201.)

279
Q

A 72-year-old woman presents to the clinic to discuss surgical management of her long-standing diverticulosis. What would be an indication for a colectomy in this patient?

A. Three episodes of diverticulitis requiring hospitalization in an otherwise asymptomatic patient.

B. A single episode of diverticulitis in an immunosuppressed patient.

C. A current episode of complicated diverticulitis resulting in feculent peritonitis.

D. Inability to exclude malignancy in a patient who was recently hospitalized for her first episode of complicated diverticulitis.

A

Answer: B

Many surgeons now will not advise colectomy even after two documented episodes of diverticulitis assuming the patient is completely asymptomatic and that carcinoma has been excluded by colonoscopy.

Immunosuppressed patients are generally still advised to undergo colectomy after a single episode of documented diverticulitis.

Medical comorbidities should be considered when evaluating a patient for elective resection, and the risks of recurrent disease should be weighed against the risks o the operation.

Because colon carcinoma may present in an identical fashion to diverticulitis (either complicated or uncomplicated), all patients must be evaluated
for malignancy after resolution of the acute episode.

Colonoscopy is recommended 4 to 6 weeks after recovery.

Inability to exclude malignancy is another indication for resection.

(See Schwartz 10th ed., p. 1202.)

280
Q

A 63-year-old woman presents to the ED with a 2-day history of left lower quadrant abdominal pain and is found to be febrile to 38.6°C. Her white blood cell (WBC) count is 15,000. On CT scan she is found to have colonic inflammation with an associated pericolic abscess. What is her Hinchey stage?

A. Stage I
B. Stage II
C. Stage III
D. Stage IV

A

Answer: A*

The Hinchey staging system is often used to describe the severity of complicated diverticulitis:

Stage I includes colonic inflammation with an associated pericolic abscess;

stage II includes colonic inflammation with a retroperitoneal or pelvic abscess;

stage III is associated with purulent peritonitis;

and stage IV is associated with fecal peritonitis. (See Schwartz 10th ed., p. 1202.)

281
Q

A 58-year-old man presents to the clinic with a 2-month history of the sensation of urinating air. He has a history of diverticulitis, with his last episode requiring hospitalization being 6 months ago. What are the most common fistulas that develop in complicated diverticulitis?

A. Colovaginal fistulas
B. Coloenteric fistulas
C. Colocutaneous fistulas
D. Colovesical fistulas

A

Answer: D

Approximately 5% of patients with complicated diverticulitis develop fistulas between the colon and an adjacent organ.

Colovesical fistulas are most common, followed by colovaginal and coloenteric fistulas.

Colocutaneous fistulas are a rare complication of diverticulitis. (See Schwartz 10th ed., p. 1203.)

282
Q

A 32-year-old man presents to the ED with a 2-month history of alternating diarrhea and constipation, rectal bleeding, a 20-lb weight loss, and worsening fatigue. What are the most common genetic mutations that could have led to the development of this patient’s colon cancer?

A. APC, deleted in colorectal carcinoma (DCC), p53

B. APC, BRCA1, K-ras

C. DCC, p53, and MYH gene on chromosome 6p

D. MYH gene on chromosome 1p, APC, K-ras

A

Answer: A

Mutations may cause activation of oncogenes (K-ras) and/or inactivation of tumor suppressor genes (APC, deleted in colorectal carcinoma [DCC], p53).

Colorectal carcinoma is thought to develop from adenomatous polyps by accumulation of these mutations in what has come to be known as the adenoma-carcinoma sequence.

The APC gene is a tumor suppressor gene. Mutations in both alleles are necessary to initiate polyp formation.

Mutation of K-ras results in an inability to hydrolyze guanosinetriphosphate (GTP), thus leaving the G-protein permanently in the active form. It is thought that this then leads to uncontrolled cell division.

MYH is a base excision repair gene, and biallelic deletion results in changes in other downstream molecules.

Since its discovery, MYH mutations have been associated with an AFAP phenotype in addition to sporadic cancers.

The tumor suppressor gene p53 has been well characterized in a number of malignancies.

The p53 protein appears to be crucial or initiating apoptosis in cells with irreparable genetic damage.

Mutations in p53 are present in 75% of colorectal cancers.

(See Schwartz 10th ed., pp. 1204–1205.)

283
Q

A 64-year-old man presents to the clinic with a 1-month history of 10-lb weight loss and rectal bleeding. His hematocrit is found to be 27. On colonoscopy he is found to have a large cecal mass.

What risk factors should he have modified to prevent development to colon cancer?

A. Eating a high-fat, high-protein, and low-carbohydrate diet and drinking a bottle of wine a day.

B. Eating a high-fiber, low-fat, and low-protein diet, drinking a six packs of beer a day.

C. Eating a high-fiber, low-fat, and high-protein diet and drinking only on holidays.

D. Eating a high-fat, low-fiber diet and drinking only on holidays.

A

Answer: C

A diet high in saturated or polyunsaturated fats increases risk of colorectal cancer, while a diet high in oleic acid (olive oil, coconut oil, fish oil) does not increase risk.

Animal studies suggest that fats may be directly toxic to the colonic mucosa and thus may induce early malignant changes.

In contrast, a diet high in vegetable fiber appears to be protective.

A correlation between alcohol intake and incidence of colorectal carcinoma has also been suggested.

Ingestion of calcium, selenium, vitamins A, C, and E, carotenoids, and plant phenols may decrease the risk of developing colorectal cancer.

Obesity and sedentary lifestyle dramatically increase cancer-related mortality in a number of malignancies, including colorectal carcinoma.

This knowledge is the basis for primary prevention strategies to eliminate colorectal cancer by altering diet and lifestyle. (See Schwartz 10th ed., p. 1204.)

284
Q

A 63-year-old man undergoes a screening colonoscopy and is found to have a polyp in his sigmoid colon. Which type of polyp is most associated with malignancy?

A. Tubular adenoma

B. Villous adenoma

C. Tubulovillous adenoma

D. 1 cm polyp

A

Answer: B

Adenomatous polyps are common, occurring in up to 25% of the population older than 50 years of age in the United States.

By definition, these lesions are dysplastic. The risk of malignant degeneration is related to both the size and type of polyp.

Tubular adenomas are associated with malignancy in only 5% of cases, whereas villous adenomas may harbor cancer in up to 40%.

Tubulovillous adenomas are at intermediate risk (22%).

Invasive carcinomas are rare in polyps smaller than 1 cm; the incidence increases with size.

The risk of carcinoma in a polyp larger than 2 cm is 35 to 50%.

Although most neoplastic polyps do not evolve to cancer, most colorectal cancers originate as a polyp.

(See Schwartz 10th ed., p. 1205.)

285
Q

A 50-year-old woman presents to the ED after having her first colonoscopy on her birthday. She endorses left lower
quadrant abdominal pain and is febrile to 38.5°C. What complications after a colonoscopy require emergent laparotomy?

A. Small perforation in a stable patient.

B. Hypotension and temperature of 38°C in an otherwise stable patient.

C. Rigid abdomen and severe abdominal pain in a patient with poor bowel preparation.

D. Findings of a large, fat sessile polyp.

A

Answer: C

Complications of polypectomy include perforation and bleeding.

A small perforation (microperforation) in a fully prepared, stable patient may be managed with bowel rest, broad-spectrum antibiotics, and close observation.

Signs of sepsis, peritonitis, or deterioration in clinical condition are indications for laparotomy.

Bleeding may occur immediately after polypectomy or may be delayed. The bleeding will usually stop spontaneously, but colonoscopy may be required to resnare a bleeding stalk or cauterize the lesion.

Occasionally angiography and infusion of vasopressin may be necessary. Rarely, colectomy is required. (See Schwartz 10th ed., p. 1206.)

286
Q

A 36-year-old woman with a complicated family history including her grandmother, mother, and sister who were diagnosed with colon cancer and a younger sister who was recently diagnosed with pancreatic cancer presents to the clinic for genetic counseling. What is a correct statement about this syndrome?

A. It is the result of mismatch repair genes such as FAP and screening criteria look for three family members who have related cancers or two family members diagnosed before age 50.

B. It is the result of an autosomal dominant condition and screening criteria look or three family members with the diagnosis of adenocarcinoma or two relatives diagnosed before age 50.

C. It is the result of mismatch repair genes such as PMS2 or MSH6 and three affected relatives with histologically verified adenocarcinoma of the large bowel in two successive generations of a family with one patient diagnosed before age 50.

D. It is the result of an autosomal dominant condition and three affected relatives with histologically verified adenocarcinoma of the large bowel in two successive generations of a family with one patient diagnosed before age 50.

A

Answer:C
Hereditary nonpolyposis colorectal cancer (HNPCC; Lynch syndrome) is extremely rare (1–3% of all colon cancers).

The genetic defects associated with HNPCC arise from errors in mismatch repair, the phenotypic result being MSI.

HNPCC is inherited in an autosomal dominant pattern and is characterized by the development of colorectal carcinoma at an early age (average age, 40–45 years).

Approximately 70% of affected individuals will develop colorectal cancer.

Cancers appear in the proximal colon more often than in sporadic colorectal cancer and have a better prognosis regardless of stage.

The risk of synchronous or metachronous colorectal carcinoma is 40%.

HNPCC may also be associated with extracolonic malignancies, including endometrial carcinoma, which is most common, and ovarian, pancreas, stomach, small bowel, biliary, and urinary tract carcinomas.

The diagnosis of HNPCC is made based on family history. The Amsterdam criteria or clinical diagnosis of HNPCC are three affected relatives with histologically verified adenocarcinoma of the large bowel (one must be a first-degree relative of one of the others) in two successive generations of a family with one patient diagnosed before age 50 years.

HNPCC results from mutations in mismatch repair genes, and like FAP, specific mutations are associated with different phenotypes.

For example, mutations in PMS2 or MSH6 result in a more attenuated form of HNPCC when compared to mutations in other genes.

MSH6 inactivation also appears to be associated with a higher risk for endometrial cancer.

Further significance of these specific mutations remains to be determined. (See Schwartz 10th ed., p. 1207.)

287
Q

A 67-year-old man presents to the ED with a 2-month history of nausea, emesis, 20-lb weight loss, and worsening diarrhea until 4 days ago, when he stopped passing flatus and having bowel movements.

A CT scan shows a large obstructing right colon mass that may be involving the omentum with two liver lesions. What should be resected in this case?

A. Right hemicolectomy, involved omentum, and as many of the peritoneal masses you can.

B. Right hemicolectomy with arterial supply and as many nodes as you can.

C. Right hemicolectomy with arterial supply, at least 12 nodes, the involved omentum, and resectable liver masses.

D. Don’t do a colon resection. This patient has distant metastases and should be diverted or palliation.

A

Answer: C

The objective in treatment of carcinoma of the colon is to remove the primary tumor along with its lymphovascular supply.

Because the lymphatics of the colon accompany the main arterial supply, the length of bowel resected depends on which vessels are supplying the segment involved with the cancer.

Any adjacent organ or tissue, such as the omentum, that has been invaded should be resected en bloc with the tumor.

If all of the tumor cannot be removed, a palliative procedure should be considered, although it important to note that “debulking” is rarely effective in colorectal adenocarcinoma.

If the metastatic disease is low volume (isolated or potentially resectable liver lesions) and the resection of the primary tumor is straightforward (segmental abdominal colectomy), it is probably reasonable to proceed with resection.

On the other hand, if the metastatic disease is high volume (carcinomatosis), especially if the primary tumor is minimally symptomatic, the operation should be aborted in order to facilitate early systemic chemotherapy.
(See Schwartz 10th ed., p. 1212.)

288
Q

A 30-year-old man presents to the ED after a witnessed syncopal episode. He has been having bloody diarrhea and intermittent crampy abdominal pain for the past 3 months. A week later he has a colonoscopy and is found to have ulcerative colitis based on colonoscopy findings and mucosal biopsies. Which feature of listed below is NOT seen in ulcerative colitis?

A. The terminal ileum shows inflammatory changes.

B. The colon is shortened and mucosa is replaced by
scars.

C. Rectal sparing with inflammation seen in the transverse and descending colon.

D. Atrophic mucosa with crypt abscesses.

A

Answer: C
Ulcerative colitis is a mucosal process in which the colonic mucosa and submucosa are infiltrated with inflammatory cells.

The mucosa may be atrophic, and crypt abscesses are common.

In long-standing ulcerative colitis, the colon may be foreshortened and the mucosa replaced by scar.

Ulcerative colitis does not involve the small intestine, but the terminal ileum may demonstrate inflammatory changes (“backwash ileitis”).

A key feature of ulcerative colitis is the continuous involvement of the rectum and colon; rectal sparing or skip lesions suggest a diagnosis of Crohn disease. (See Schwartz 10th ed., p. 1195.)

289
Q

The most appropriate treatment for massively bleeding Meckel diverticulum is:

A. Segmental resection anastomosis
B. Diverticulectomy
C. Diverticulectomy with purse-string closure
D. Stapled wedge resection of the diverticulum

A

A. Segmental resection anastomosis

290
Q

The most common cause of LGIB in the elderly is:

A. Colorectal cancers

B. Angiodysplasia

C. Diverticulosis

D. Peptic Ulcer

A

A. Colorectal cancers

291
Q

A 19-year-old man is admitted due to toxic colitis. He does not respond to treatment with high dose corticosteroids, bowel rest, and total parenteral nutrition. He had lost 15lbs in the last two months. The most appropriate surgical treatment:

A. Proctocolectomy with end ileostomy

B. Total abdominal colectomy with end ileostomy and Hartman closure of the rectum

C. Restorative proctocolectomy with ileoanal pouch

D. Total abdominal colectomy with ileorectal anastomoses

A

B. Total abdominal colectomy with end ileostomy and Hartman closure of the rectum

292
Q

Which of the ff risk factors poses the greatest risk for colon cancer development?

A. Low fiber diet

B. Diet high in animal fat

C. 65-year-old parent with colon cancer

D. HNPCC mutation

A

D. HNPCC mutation

293
Q

A 2-cm sessile polyp found during a routine screening colonoscopy is removed from a 72-year-old female with a history of hypertension and diabetes. Pathology showed a focus of adenocarcinoma through the muscular mucosa with lymphovascular invasion. What will you recommend:

A. Adjuvant chemotherapy due to the LV invasion

B. That she undergo formal colonic resection

C. That she have a repeat colonoscopy 1 year after

D. No further therapy; polyp removal was curative.

A

B. That she undergo formal colonic resection

294
Q

A 60-year-old male undergoes sigmoid colectomy for a moderately differentiated adenocarcinoma invasive into the muscular proprietary with 15 negative regional LNs. A 5mm smooth nodule in the pericolic fat reveals microscopic adenocarcinoma. The appropriate TNM stage of this lesions is:

A. T2N1M0
B. T2N0M1
C. T3N1M0
D. T3N0M1

A

A. T2N1M0

295
Q

Regarding CEA in the management of colorectal cancer, which is true:

A. In patients with lesions that have not penetrated the bowel wall, the CEA levels are usually not elevated.

B. A high CEA level preoperatively has a low correlation with prognosis and recurrence.

C. A rising CEA after treatment should warrant a second look surgery despite a negative work-up.

D. CEA may be used as a screening tool for colon or rectal cancer especially if there is a significant history in the family.

A

A. In patients with lesions that have not penetrated the bowel wall, the CEA levels are usually not elevated.

296
Q

Colonic perforation in a patient with acquired immunodeficiency syndrome is most likely due to:

A. Clostridium difficile
B. Eschericia coli
C. Cytomegalovirus
D. Salmonella typhi

A

C. Cytomegalovirus

297
Q

Which of the following has the least premalignant potential for colorectal cancers?

A. Tubular adenoma

B. Tubulo-villous adenoma

C. Villous adenoma

D. 2.5cm adenomatous polyp

A

A. Tubular adenoma

298
Q

Treatment of choice for anterior midline anal fissure is:

A. Excision of the fissure and posterior internal sphincterotomy

B. Posterior internal sphincterotomy

C. Lateral partial subcutaneous sphincterotomy only

D. Lateral internal sphincterotomy only

A

B. Posterior internal sphincterotomy

299
Q

A 48-year-old man known to have hemorrhoids reports having difficulty reducing his prolapsed hemorrhoidal piles, prompting you to recommend operative hemorrhoidectomy. In relation to this case, which is the best indication for surgery?

A. Grade 2 hemorrhoids
B. A bleeding internal hemorrhoid that has stopped
C. Thrombosed mixed hemorrhoids
D. External hemorrhoid and fibrotic skin tag

A

C. Thrombosed mixed hemorrhoids

300
Q

Diagnosis of fistula-in-ano is almost clinched with a persistent drainage from an internal opening that is connected to the external opening via an indurated tract. In regard with the use of Goodsall’s rule in determining the location of the internal opening, which of the following statements is true?

A. Fistulas with an external opening anteriorly connect to the internal opening by a short radial tract.

B. Fistulas with an external opening posteriorly track in a straight fashion to the anterior midline.

C. Fistulas with an anterior external opening greater than 3cm, track to the lateral midline.

D. Fistulas with an external opening near the rectal wall tracks around the internal sphincter and exits in the posterior midline.

A

A. Fistulas with an external opening anteriorly connect to the internal opening by a short radial tract.

301
Q

The limit between the two territories of the SMA and IMA is:

A. The proximal 1/3 and distal 2/3 of the transverse colon.

B. The proximal 2/3 and distal 1/3 of the transverse colon.

C. The area of the splenic flexure

D. The mid-transverse colon where the middle colic artery arises.

A

C. The area of the splenic flexure

302
Q

The following is false regarding colonic physiology:

A. Short-chain fatty acids (SCFAs) provide nourishment to the colonic mucosa.

B. SCFA are produced by bacterial fermentation of dietary lipids.

C. Intestinal gas arises from diffusion from the blood, swallowed air, and intraluminal production.

D. Colonic bacteria is necessary in the production of Vitamin K.

A

B. SCFA are produced by bacterial fermentation of dietary lipids.

303
Q

Diarrhea is most likely to occur after a right hemicolectomy than a left. The most likely explanation is:

A. The loss of salt and water absorption from the right colon

B. The loss of the ileocecal valve

C. The increase in bile salt spillage

D. Bacterial overgrowth in the small bowel.

A

A. The loss of salt and water absorption from the right colon

304
Q

Which is the most common anaerobic bacterium found in the normal colon?

A. Eschericia coli

B. Peptostreptococcus

C. Bacteroides fragilis

D. Clostridium difficile

A

C. Bacteroides fragilis

305
Q

The most common cause of large bowel obstruction in adult population is

A. Diverticulitis
B. Colorectal cancer
C. Adhesions
D. Volvulus

A

B. Colorectal cancer

306
Q

Massive bleeding from the lower GIT is occurring in a 55 year old man who is otherwise healthy. After the patient has been resuscitated, the next step in management should be:

A. Emergency laparotomy and total colectomy and ileoproctostomy

B. Emergency laparotomy and colostomy with operative endoscopy

C. Colonoscopy and EGD

D. Colonic irrigation with iced saline solution

A

C. Colonoscopy and EGD

307
Q

A 25yo male consulted for a small mass on the anal area, tumor is measured at 1.5cm in size located at the left anterior approx 5cm from the anal opening. Biopsy done revealed squamous cel carcinoma. The best treatment recommendation for this case is:

A. 5-FU + mitomycin + RT
B. Wide excision
C. Abdominoperineal excision with TME
D. Excision plus sentinel inguinal node biopsy

A

B. Wide excision

For anal SCCA:
Nigro protocol UNLESS
- If T1N0, well-differentiated (EXCISE)

308
Q

A 65-year-old male previously with anal SCCA followed up in your clinic with post-treatment MRI showing progression of tumor now involving the levator ani complex. Next appropriate plan:

A. Re-stage the disease, prepare for salvage abdominoperineal excision.

B. Refer back patient to radiation oncologist for boost RT with a dose of 16Gy.

C. Refer patient to medical oncologist for palliative chemotherapy.

D. No further intervention warranted.

A

A. Re-stage the disease, prepare for salvage abdominoperineal excision.

NCCN: Progressive

Evaluate for 8-12 weeks
4-6 weeks up to 28 weeks

For progressive disease, if biopsy proven, restage disease. Locally recurrent disease warrants APR + groin dissection, with positive inguinal nodes.

Metastatic disease warrants 5-FU or Carboplatin/paclitaxel or FOLFOX or modified DCF.

Surveillance involves inguinal node palpation every 3-6months for 5 years, Chest/abdominal/pelvic CT with contrast or chest CT without contrast and abdominal/pelvic MRI with contrast annually for 3 years.

309
Q

A 50-year-old female came into your clinic due to a 3-month history of anal pain and bleeding. On PE, you noted a pigmented, ulcerated anal mass with raised edges and multiple enlarged inguinal lymph nodes. The following are true of anal melanoma, except:

A. Anal melanoma arise from transitional epithelium of the anal canal, the anoderm, or mucocutaneous junction.

B. Anal melanoma has a good response to chemotherapy and radiotherapy.

C. Anal melanoma is an aggressive disease and prognosis is poor if perineurial invasion is observed.

D. If R0 resection is achieved during local excision for hemorrhoids, patients do not need further intervention.

A

B. Anal melanoma has a good response to chemotherapy and radiotherapy.

310
Q

A 45-yo primigravid on 30 weeks AGO complained of a perianal mass that is manually reducible with occasional bleeding.

A. Rubber band ligation
B. HAL-RAR
C. LIFT
D. Observe until pregnancy is terminated

A

D. Observe until pregnancy is terminated

311
Q

A 30-year-old male presents at the emergency room for severe anal pain. On evaluation, there are non-reducible violaceous circumferential hemorrhoidal piles. What is the best management for this patient:

A. Start the patient with diosmin+hesperidin tablet to alleviate the symptom

B. Do incision of hemorrhoids to relieve pain

C. Do emergent excision hemorrhoidectomy

D. Do colonoscopy to rule out other colonic pathology.

A

If <3 days: excisional

If >3 days: conservative

312
Q

A 55-yo male, previously diagnosed with hemorhoids, s/p hemorrhoidectomy x 3 had complaints of difficulty on defecation. On examination, the anus was constricted with difficulty in inserting the fifth digit of the examining finger. What is the best management for this patient?

A. Instruct patient to do warm sitz bath TID to relieve the symptoms

B. Schedule the patient for examination under anesthesia, possible anal advancement flap

C. Schedule the patient for partial lateral internal
sphincterotomy

D. Give psyllium fiber 3x/day

A

B

313
Q

88-year old female, chronically bedridden due to previous CVD infarct, had frequent episodes of soiling. On examination, a prolapsing circumferential mucosa was observed. For this case, the best surgical management is:

A. Altemeier procedure
B. Moschowitz repair
C. Ventral rectopexy
D. Manual reduction

A

A

314
Q

The major muscle that contributes to fecal continence is

A. External sphincter muscle
B. Internal sphincter muscle
C. Pubococcygeus
D. Puborectalis

A

D

315
Q

The following are true regarding rectal prolapse, except:

A. For patients presenting with rectal prolapse, it is paramount to determine history
B. Highest recurrence rates are observed in abdominal repair procedures
C. Perineal repair approach are mostly reserved for patients high risk in developing postop complications
D. All are true

A

B

316
Q

A 70yo female G9P9 with multiple comorbids (ASA 4) came in at the ER complaining of full rectal prolapse for 3 days duration. The surgical plan most suited for the patient is:

A. Laparoscopic resection and rectopexy
B. Moschowitz repair
C. Thiersch procedure
D. Ripstein and Wells rectopexy

A

C

317
Q

A 19-year-old male complained of severe anal pain on defecation. The patient refuses further examination due to pain. Examination under anesthesia was done. A Brodie’s triad was observed. 1. A break in anoderm exposing the internal anal sphincter; 2. hypertrophied internal papilla; 3. A sentinel pile. The treatment of choice for chronic anal fissure is:

A. Botulinum toxin injections
B. Calcium channel blockers
C. Psyllium fiber supplementation
D. Partial lateral internal sphincterotomy

A

D

318
Q

A 50-year-old male was diagnosed with anal fissure. The fissure is located at the posterior midline. The best management for this case is:

A. Do sphincterotomy at the midline to address the fissure
B. Do partial lateral internal sphincterotomy, right or left to avoid keyhole deformity postop
C. Botulinum toxin injections have a low recurrence rate
D. Calcium channel blockers are not helpful in acute attacks of fissure

A

B

319
Q

A 44-year-old male had fluctuant mass on the perianal area and underwent incision and drainage. three months after, he noted drainage of foul odor discharge on the postop site. The treatment of choice is:

A. Start metronidazole 500mg tab q8 x 7 days
B. Incision and drainage of abscess
C. Examination under anesthesia, ligation of intersphincteric fistula tract
D. Warm sitz bath

A

C

320
Q

A 12-year-old male came to your clinic for consult for a recurrent fistula in ano of 3 month duration in which he underwent a former fistulotomy, twice. what is the next appropriate management for the patient?

A. Schedule patient for EUA possible fistulectomy
B. Advise patient to undergo an endoscopic evaluation prior to any surgical intervention.
C. An EUA with LIFT
D. Start antibiotic with Metronidazole 500mg every 8 hour for two weeks, then schedule the
patient for fistulectomy.

A

B

321
Q

A 35-year-old male complaining of foul-smelling perianal discharge associated with occasional pain and tenderness. On examination, you noted an external fistula on the left anterior approximately 4cm FAV, with no palpable internal tract. The following is/are true of fistula-in- ano:

A. The most common type is the transphincteric type which comprises almost 50% of all fistulas-in-ano
B. Most common drainage of posteriorly located fistula-in-ano is on the posterior midline of the dentate line
C. Antibiotic management with coverage of anaerobic and gram negative bacteria are recommended for patients with fistula-in-ano prior to undergoing surgical procedures.
D. All of the above

A

B

322
Q

A 30-year-old male with multiple draining sinuses on his buttocks consulted a private MD. Skin thickening was also observed. He was advised surgery. The surgical plan for this patient is:

A. IV antibiotics
B. Radical excision of lesions with plastic reconstruction
C. Seton placement
D. Incision and drainage

A

B

323
Q

A 23-year-old male complains of pain on the superior gluteal fold. On examination he has hairy skin on the gluteal area and a fluctuant mass was observed on the middle of the gluteal folds. What is the next management for this patient?

A. Start amoxicillin 500mg/tab 3x/day for 7 days
B. Schedule the patient for a Karyadakis flap
C. Incision and drainage with incision lateral to the gluteal fold
D. Extensive resection of affected skin to prevent recurrence.

A

C

324
Q

A 33-year-old female just gave birth 3 weeks ago via NSD, had complaints of pain and fecaloid discharge per vagina. on exam, the communication is near the dentate line and vaginal orifice. What is your next step:

A. Admit the patient, plan for a fecal diversion.
B. Advise the patient that 50% of this case will heal spontaneously with proper wound care.
C. Admit the patient and schedule for an endorectal advancement flap.
D. Schedule the patient for colonoscopy to rule out inflammatory bowel disease.

A

B

325
Q

An 80-year-old female who is chronically constipated underwent flexible sigmoidscopy for evaluation of difficulty in defecation. On examination, a solitary ulcer was found. True statement regarding this condition:

A. Solitary rectal ulcer (SRU) is a premalignant lesion that warrants excision.
B. SRU can be due to frequent digitation, though there are lesions located beyond the reach
of the examining finger.
C. SRU is considered benign lesion and biopsy is not warranted.
D. SRU is response to Nigro protocol.

A

B

326
Q

A 45-year-old male with poor DM control consulted for an erythematous wound on the perianal area with involvement of the perineal area. A strong foul odor was also observed. What is the best management for this case?

A. Start broad-spectrum antibiotics and wait for clinical response.
B. Do emergent wound debridement and orchiectomy.
C. Do emergency wound debridement.
D. Do contrast MRI to delineate the involvement of the lesion.

A

C

327
Q

The most important superficial plane of the perineum that causes rapid spreading infection along fascial planes resulting to Fournier’s gangrene?

A. Buck’s fascia
B. Colle’s fascia
C. Waldeyer’s fascia
D. Denonvillier’s fascia

A

B

328
Q

A 52-year-old female consults for occasional fecal incontinence, mostly to liquid stools, occurring twice a week on average. She has had four vaginal deliveries with no fetal or maternal complications. she does not recall if a formal episiotomy had been performed in any of her pregnancies. she claims to have a history of constipation when she was younger, for which she developed the habit of straining to have a bowel movement even without the urge to defecate. At present, she is able to have unassisted bowel movements every other day, but would notice some liquid stool staining on her underwear. Which imaging modality would be the best diagnostic examination?

A. Endoanal ultrasound
B. Endovaginal ultrasound
C. Transperineal pelvic floor ultrasound
D. Dynamic pelvic MRI

A

A

329
Q

A 38-year-old female sought consultation because of a fecaloid smell after urination associated with a foul-smelling discharge after giving birth 4 weeks before. She was treated with antibiotics previously, but with no improvement in her condition. Barium enema revealed a rectovaginal fistula. The BEST option for this patient is:

A. Wait 6 months before doing any surgical intervention
B. Do an endorectal advancement flap as soon as possible
C. Treat with higher dose of antibiotics.
D. Do a diverting colostomy immediately.

A

A

330
Q

Which of the following is considered an acute complication of hemorrhoidectomy?

A. Urinary retention
B. Anal stenosis
C. Ectropion
D. Incontinence

A

A

331
Q

A 52-year-old male patient came in because of anal pain since 2 weeks ago. He occasionally experiences febrile episodes and difficulty of defecation because of the pain. It is described as deep into the anus and he cannot tolerate rectal examination. The most likely diagnosis is:

A. Fistula-in-ano
B. Engorged internal hemorrhoids
C. Perianal abscess
D. Anal fissure

A

C

332
Q

A 60-year-old female was diagnosed with malignant polyp at the descending colon, snare polypectomy was performed. The main purpose of endoscopic follow-up after treatment of malignant polyp is:

A. Detection of recurrence at treated site
B. Treating metachronous disease
C. Detection and removal of advanced adenomas to minimize interval colorectal cancer and
mortality
D. All of the above.

A

D

333
Q

A 66-year-old male is admitted for passage of fresh and clotted blood per rectum. For the past 6 months he has had 3 similar episodes. The most important aspect in the management of this patient is:

A. Fluid resuscitation and stabilization
B. Identification of the exact cause of bleeding
C. Localization of the site of bleeding
D. Nonoperative measures to stop the bleeding

A

B

334
Q

In patients with colorectal cancer, the serum CEA level is a clinically useful measure for the following reasons EXCEPT

A. Early diagnosis
B. Monitoring treatment response
C. Detection of loco-regional recurrence
D. Detection of distant mets

A

A

335
Q

A 62/F underwent colonoscopy for change in bowel habits. the scope was not able to pass beyond the tumor due to almost complete obstruction of the lumen. She subsequently underwent sigmoidectomy for the obstruction sigmoid colon cancer. Postoperative course was unremarkable. When should she have the full colonoscopy completed?

A. One year after surgery
B. Two weeks after surgery
C. Three months after surgery
D. No need to have it completed

A

C

336
Q

Which of the following is TRUE regarding the extent of resection of a malignant colon tumor?

A. It should be resected at 10cm proximal and 10cm distal to the site of cancer.
B. It should correspond to the lymphovascular drainage of the site of cancer.
C. It should depend on the ability to anastomose both bowel ends without tension.
D. It should not include any part of the rectum.

A

B

337
Q

The adjuvant treatment of a patient who has undergone appropriate surgery to resect a completely obstructing ascending colon cancer which is pT3N0M0 is

A. Chemotherapy and radiation
B. Fluorouracil-based chemotherapy
C. Targeted therapy with Bevacizumab
D. No additional therapy needed

A

D?

338
Q

After the appropriate evaluation, the patient diagnosed with descending colon cancer undergoes surgery. No intraoperative evidence of metastases is identified. Postoperatively, the pathology report reveals that the tumor is an adenocarcinoma invading into the pericolonic fat, with 2 involved lymph nodes out of 15. After the patient recovers from surgery, which of the ff is the most appropriate next step in his management?

A. Abdominal CT scan every 6 months
B. No further therapy is indicated, because the involved nodes were removed
C. Chemotherapy with 5-FU based regimen
D. Colonoscopy every 6 months

A

C

339
Q

Long course chemoRT is BEST for:
A. Tumors with high likelihood of R1 and R2 and (+) CRM on surgery
B. Nodal involvement outside of mesorectum
C. Increasing chance for sphincter preservation
D. All of the above.

A

D

340
Q

All of the following statements are true about total mesolectal excision (TME) EXCEPT:

A. It is a technique that uses sharp dissection along anatomic planes to ensure complete resection of the rectal mesentery during low and extended low anterior resections.

B. It is indicated for low, middle and upper rectum tumors.

C. This technique is associated with less blood loss and less risk to the pelvic nerves and presacral plexus than is blunt dissection.

D. It both decreases local recurrence rates and improves long-term survival rates.

A

B

341
Q

A 70-year-old man complaining of anal pain and bleeding is found to have an ulcerated warty growth occupying almost three fourths of the diameter of the anal canal. Biopsy reveals epidermoid carcinoma. The best treatment in this case is:

A. Low resection with coloanal anastomosis
B. Abdominoperineal resection
C. Chemoradiation
D. Wide excision

A

C

342
Q

A 68-year-old male diagnosed with sigmoid diverticulitis previously is for elective sigmoidectomy. The distal line of transection is determined by:

A. Disappearance of the taenia coli
B. 5cm above the anterior peritoneal reflection
C. Sacral promontory
D. 5cm below the diverticulitis line

A

A

343
Q

Emergency exploration is done for unlocalized massive hematochezia. Angiography did not reveal a source of bleeding. At surgery a large right-sided diverticulosis is seen with more blood pooling in the right colon than the left colon. the procedure of choice is:

A. Right hemicolectomy with intraoperative small bowel enteroscopy to rule out small bowel source
B. Inferior Mesenteric Artery ligation then observe patient for further bleeding episodes
C. Intraoperative emergency angiography and segmental colectomy
D. Blind Total Abdominal Colectomy

A

D

344
Q

A 42-year-old man presents with acutely thromboses, prolapsed hemorrhoids of 1 day duration. This patient:

A. Conservative treatment is an option
B. Either rubber band application or infrared photocoagulation is appropriate
C. Emergency hemorrhoidectomy should not be performed because of the likelihood of
complications
D. Hemorrhoidectomy may be safely performed

A

D

345
Q

A 35-year-old male consulted because of severe pain on defecation for the past six months. On anoscopy, there was an ulceration with white fibers at the base of the ulcer. Noted also was an external skin tag. The most appropriate management for this patient is:

A. Stool softener and warm sitz bath
B. Nitroglycerin ointment
C. Excision of ulcer
D. Lateral internal sphincterotomy

A

D

346
Q

Which of the following fistulae is NOT considered as complex?

A. High trans-sphincteric fistula with a high blind tract
B. High trans-sphincteric fistula without a high blind tract C. Horseshoe fistula on a patient with no previous surgery
D. Anteriorly located fistula in men.

A

D

347
Q

What is the Nigro protocol?

A

5-fluorouracil
Mitomycin C
30 Gy of external beam radiation

348
Q

What is resected in an Ileocolic Resection?

A

Terminal ileum +
cecum +
appendix

For: Ileocecal Crohn’s

349
Q

What is resected in a Right Colectomy?

A

10cm of terminal ileum +
ileocolic vessels +
right colic vessels +
right branch of middle colic

For: Proximal Colon CA

350
Q

What is resected in an Extended Right Colectomy?

A

Right colectomy +
ligation of middle colic vessels at the base with a
distal ileum + distal transverse anastomosis

For: Hepatic Flexure or Proximal Transverse Cancer

351
Q

What is resected in a Transverse Colectomy?

A

Colocolonic anastomosis +
Equivalent to extended right colectomy

For: Mid and Distal Transverse Lesions

352
Q

What is resected in a Left Colectomy?

A

Left branches of middle colic + left colic +
first branches of sigmoid
with a colocolonic anastomosis

For: Distal Transverse
Splenic Flexure, or
Descending Colon CA

353
Q

What is resected in an Extended Left Colectomy?

A

Left colectomy + right branches of middle colic

For: Distal Transverse lesions

354
Q

What is resected in a Sigmoid Colectomy?

A

Sigmoid branches + descending colon with an upper rectal anastomosis

For: Sigmoid Colon lesions

355
Q

What is resected in a Total and Subtotal Colectomy?

A

Ileocolic vessels +
right colic vein +
middle colic vein +
left colic vein

PRESERVE the superior rectal vessels.

For: Fulminant colitis
Attenuated FAP
Synchronous colon CA

356
Q

What is resected in a Total Proctocolectomy?

A

Colon +
rectum +
anus

with an end ileostomy.

357
Q

What is resected in a Restorative Proctocolectomy (ileal pouch anal anastomosis)?

A

Colon +
rectum

Anal sphincter muscles are PRESERVED.

With an ileoanal anastomosis (with neorectum as a J, S, W ileal pouch)

358
Q

What is resected in an Anterior Resection (High)?

A

Distal sigmoid +
upper rectum
with IMA + IMV ligation

For: Diverticulitis

359
Q

What is resected in a Low Anterior Resection?

A

Rectosigmoid +
IMA + IMV

For: Upper + mid rectum lesions (extraperitoneal rectum 6-10cm)

360
Q

What is resected in an Extended Low Anterior Resection?

A

Rectum is fully mobilized to level of levator ani muscle.

Anterior dissection is extended along rectovaginal septum (females)

or distal to the seminal vesicles and prostate (males)

with a coloanal anastomosis

361
Q

What is resected in a Hartmann’s procedure?

A

Colorectal resection +
colostomy/ileostomy +
distal rectum as blind pouch

alternative: mucus fistula (if length of remaining segment will allow)

For: Diverticulitis with peritonitis

362
Q

What is resected in an abdominoperineal resection?

A

Entire rectum +
anal canal + anus +
permanent colostomy from descending or sigmoid

363
Q

What is resected in an intersphincteric resection?

A
  • Intersphincteric resection (ISR): Remove the internal anal sphincter

Partial: Surgical anal margin
Subtotal:
Total: Until dentate line

364
Q

Haggitt Classification for pedunculated polyps?

A

Level 0: Noninvasive (severe dysplasia, carcinoma In situ, or Intramucosal carcinoma) (0% risk of LN mets)

Level 1: Invading through muscularis mucosa, but limited to the head of a pedunculated polyp (<1%)

Level 2: Cancer invading the neck of a pedunculated polyp (<1%)

Level 3: Cancer invading the stalk of a pedunculated polyp (<1%)

Level 4: Cancer invading into the submucosa of the bowel wall below the stalk of a pedunculated polyp. All session polyps with invasive cancer are level 4 (12-25%)

365
Q

Kikuchi classification of sessile polyps?

A

Distance of SM In Asian

SM1: Superficial 1/3 (2% nodal mets)

SM2: Superficial 2/3 (8%)

SM3: Deep 1/3 (23%)

366
Q

York-Mason Clinical Rectal Cancer Staging?

A

cS1: freely mobile (submucosa)

cS2: mobile with the rectal wall (muscularis propria)

cS3: tethered mobility (perirectal fat)

cS4: fixed (adjacent tissues)

367
Q
  1. A 25-year-old male develops diarrhea and
    colicky abdominal pain. Ulcertive colitus is
    diagnosed on colonoscopy. Which of the following findings is consistent with the diagnosis?
    (A) The rectum is not involved.
    (B) The disease is confluent, there are no
    skip areas in the colon and the rectum is
    involved.
    (C) The full thickness of the bowel wall is
    involved.
    (D) Microscopic examination of the mucosa
    reveals normal cells without evidence of
    dysplasia.
    (E) The incidence of colorectal cancer is
    equal to that of the general population.
A
  1. (B) Ulcerative colitis is a disease of unknown
    etiology, which involves the colon and rectum
    and spares the remainder of the GI tract. It’s
    clinical course is variable with inflammatory
    changes and clinical symptoms ranging from
    mild to severe. The process is confined to the
    mucosa and the submucosa and does not
    extend through the full thickness of the bowel
    wall. Inflammatory changes are confluent with
    no skip areas. The risk of dysplasia and colorectal cancer is higher in ulcerative colitis than
    in the general population.
368
Q
  1. A 35-year-old man has known ulcerative colitis. Which of the following is an indication for
    total proctocolectomy?
    (A) Occasional bouts of colic and diarrhea
    (B) Sclerosing cholangitis
    (C) Toxic megacolon
    (D) Arthritides
    (E) Iron deficiency anemia
A
  1. (C) Toxic megacolon is a fulminant exacerbation
    of ulcerative colitis, causing massive dilatation of
    the colon with perforation, fecal peritonitis, and
    death. Emergency total colectomy is indicated.
369
Q
  1. Ten years after diagnosis of total proctocolitis
    this patient undergoes colonscopy and biopsy
    reveals high-grade dysplasia in 2–10 specimens. What should the physician recommend?
    (A) Repeat colonoscopy in 1 year
    (B) Increase steroid dosage
    (C) Early repeat colonoscopy and biopsy
    area again
    (D) Total proctocolectomy
    (E) Resection of the involved segment
A
  1. (D) Risk of dysplasia and colorectal cancer is
    higher in ulcerative colitis than in the general
    poulation. The severity, duration, and anatomic
    extent of the inflammation are risk factors for
    the development of dysplasia and cancer. These
    cancers do not seem to follow the adenoma
    carcinoma sequence and can arise in flat
    mucosa making them difficult to detect even
    with regular colonoscopies. After 8–10 years of
    colitis survellance colonoscopy should be performed with multiple random biopsies. The
    finding of dysplasia is an indiction for immediate total protocolectomy. Centers have
    reported up to 42% of colons removed for dysplasia also had colon cancer.
370
Q
  1. A55-year-old man presents with left lower quadrant (LLQ) abdominal pain of 2-day duration,
    associated with constipation. On physical examination, he has tenderness localized to the LLQ
    with fullness in that area leukocyte count is
    22,000 and temperature is 101.5°F. Which would
    be the best diagnostic study to evaluate this man?
    (A) Diagnostic laparoscopy
    (B) Barium enema
    (C) Plain abdominal roentgenogram
    (D) Computed tomography (CT) of the
    abdomen/pelvis with orally (PO) and
    intravenous (IV) contrast
    (E) Colonoscopy
A
  1. (D) The man likely has diverticulitis. The differential includes irritable bowel, appendicitis,
    inflammatory bowel disease, pyelonephritis,
    ischemic colitis, and perforated carcinoma.
    Diverticulitis is an infectious complication of
    diverticulosis resulting from perforation of the
    colonic diverticulum. The resulting inflammation may be confined to the pericolonic tissue
    (incomplicated diverticulitis) or result in
    abscess, free perforation, fistulization, or obstruction (complicated diverticulitis). The clinical
    spectrum is correspondingly broad ranging
    from mild symptoms to peritonitis and sepsis.
    Patients with signs and symptoms of sepsis
    should be hospitalized and undergo diagnostic
    study. A CT scan is the best study to evaluate
    the extent of the inflammatory process as well
    as to exclude other pathology. Plain x-ray
    would not reveal specific pathology. Both
    barium enema and colonoscopy in the acute
    setting are risky and may cause free perforation
    and contamination of the peritoneal cavity there
    by converting a localized process to generalized peritonitis. Barium has the additional risk
    of a chemical peritonitis caused by the barium
    itself. Diagnostic laparoscopy is invasive and
    may risk spreading a localized process.
371
Q
  1. Complications of diverticulitis include:
    (A) Carcinoma of the colon
    (B) Extraintestinal manifestations such as
    arthritis, iritis, and skin rashes
    (C) Fistulisation to adjacent organs such as
    the bladder, with insueing colovesical
    fistula
    (D) Artheriovenous fistulae of the intestine
    (E) Sclerosing cholangitis
A
  1. (C) Diverticulitis results from acute inflammation of a colonic diverticula. The process may
    extend into adjacent organs (e.g., the urinary
    bladder and a fistula between the colon and
    bladder colovesical fistula may ensue). This
    leads to passage of colonic gas and fecal material into the bladder and urine resulting in
    pneumaturitis and fecaluria. Sigmoid resection
    and repair of the bladder fistula is indicated.
372
Q
9. Apatients CT scan reveals diverticulitis confined
to the sigmoid colon. There is no associated pericolic abscess. What is best course of treatment?
(A) Bowel rest, nasogastric suction, IV
fluids, and broad spectrum antibiotics
(B) Urgent surgical resection
(C) Steroids
(D) Diverting colostomy
(E) Ileostomy
A
  1. (A) Uncomplicated diverticulitis is treated with
    broad spectrum antibiotics and bowel rest.
    Surgery is not indicated—either resection or
    diversion of the fecal stream by colotomy or
    ilestomy. Anti-inflamatory agents are not indicated in the therapy of diverticulitis. The risk of
    a second episode is less than 30%. After a
    second episode, the risk is greater than 50%
    and resection may be advised at this stage.
373
Q
  1. An elderly nursing home patient is brought to
    the hospital with recent onset of colicky abdominal pain, distension and obstipation on examination, the abdomen is markedly distended and
    tympanitic. There is no marked tenderness. Plain
    abdominal x-ray shows a markedly distended
    loop located mainly in the right upper quadrant.
    The likely diagnosis is:
    (A) Small-bowel obstruction
    (B) Large-bowel obstruction
    (C) Gallstone ileus
    (D) Mesenteric vascular occlusion
    (E) Sigmoid volvulus
A
  1. (E) This patient has sigmoid volvulus. Plain
    abdominal x-ray shows a massively distended
    loop in the right upper quadrant, because the
    sigmoid colon, as it progressively distends, as a
    result of the twist of its mesentery, has no space,
    in the LLQ to occupy and flips over to the largest
    available area—namely the right upper quadrant. Given the clinical presentation and findings, the plain abdominal x-ray is diagnostic.
374
Q
11. The standard initial therapy for acute sigmoid
volvulus is:
(A) Laparotomy to reduce the volvulus and
replace the sigmoid colon to its normal
position
(B) IV neostigmine
(C) Colonoscopy
(D) Ileostomy
(E) Rigid sigmoidoscopy
A
  1. (E) Rigid sigmoidoscopy is effective in reduction and decompression of the volvulus, often
    resulting in a copious rush of gas and stool as
    decompression results. It also allows for evaluation of bowel viability. If the point of rotation
    is beyond the 25-cm rigid sigmoidoscopy, flexible endoscopy may be attempted by an experienced endoscopist using minimal inflation of
    air. A rectal tube should be placed to allow for
    bowel decompression. Laparotomy may occasionally be necessary in cases of perforation or
    compromised viability.
375
Q
  1. Protoscopy reveals nonbleeding grade I hemorrhoids and maroon stool and clots coming
    from the proximal colon. Which of the following is TRUE in the management of lower gastrointestinal (GI) bleeding?
    (A) Barium enema is a good tool in the early
    evaluation of massive GI bleeding.
    (B) Technetium sulfur colloid has excellent
    sensitivity in localizing lower GI bleeding.
    (C) Technetium sulfur colloid scan is useful
    because it may be repeated 24 hours
    later with single injection.
    (D) Colonoscopy should be avoided in the
    evaluation of acute lower GI bleeding.
    (E) Sensitivity specificity and accuracy rates
    vary widely and the exact role of red
    blood cell (RBC) scanning is controversial.
A
  1. (B) Technetium sulfur colloid scans have the
    advantage of immediate availability but the
    patient must be bleeding when the isotope is
    injected as the isotope is quickly cleared by the
    reticuloendothelial system of the liver and spleen.
376
Q
  1. The patient responds to resuscitation with normalization of vital signs but continues to bleed.
    He is taken to the angiography suite for further
    evaluation. Which of the following is TRUE?
    (A) The inferior mesenteric artery should be
    injected first because most diverticula
    are in the sigmoid colon.
    (B) Vasopressin be selectively infused into a
    bleeding mesentric vessel with virtually
    no risk to the patient.
    (C) Embolization with gel foam or autologous clots may be used to stop bleeding.
    (D) Since angiography is both diagnostic and
    therapeutic surgery will not be necessary.
    (E) A bleeding rate of 0.1 mL/min is
    necessary for a positive scan.
A
  1. (B) Vasopressin can be selectively infused into
    a bleeding mesentric vessel. A bleeding rate of
    .5 per minute is necessary for a positive angiogram. Temporary success in stopping the bleeding will not obviate the need for surgery. The
    angiodysplasia of the colon is one of the most
    common causes of lower GI bleeding in elderly
    patients. With diverticular disease, 75% of the
    patients will have only a single episode of hemorrhage, whereas angiodysplasia patients are
    very likely to have recurrent episodes of variable
    severity.
377
Q
14. A 60-year-old man undergoes sigmoid colectomy for cancer of the midsigmoid. Path specimen reveals the following involvement. What
is this patient’s stage?
(A) T1 No Mo—stage I
(B) T2 N1 Mo—stage II
(C) T3 No Mo—stage III
(D) T1 N1 Mo—stage III
(E) T2 N1 Mo—stage III
A
14. (C) This patient has a T2Ni stage III colon
cancer for colon cancer—staging is categorized
by TNM system. Where, T is depth of penetration through bowel wall, N = nodal involvement, and M = metastatic disease.
T 1’s carcinoma in situ
T 1 invades submucosa
T 2 invades muscularis propria
T 3 through the muscularis propria
T 4 through visceral peritonuem
N 0 no lymph node involvement
N 1 1–3 positive nodes
N 2 4 or more pericolic nodes
N 3 any node along the main vascular
M 0 no metastasis
M1 distant metastasis
Staging is a follows:
T1 or T2 No Mo = stage I
T3 or T4 No Mo = stage II
Avg T N1 Mo = stage III
Avg T Avg N M1 = stage IV
378
Q
15. The same patient is otherwise healthy. Which of
the following is TRUE?
(A) She does not need chemotherapy
because prognosis is largely related to
depth of tumor penetration and she has
an early tumor.
(B) Postoperative chemotherapy should be
offered even though there is no proven
benefit in stage III colon cancer.
(C) Oral chemotherapy (capecitabine) is as
effective as IV chemotherapy.
(D) Patient should be offered IV 5
fluorouracil(5 FU) chemotherapy.
(E) Patient should be offered 5 fluorouracil
(5 FU) and radiation therapy.
A
  1. (D) Patients with stage III colon cancer have
    5-year survival ranging from 20% to 50%.
    Prognosis is largely related to lymph node
    involvement. Recurrence is usually in liver, peritoneal cavity, or lungs. Adjuvant chemotherapy
    with 5 FU based regimens have proven benefit
    in decreasing recurrence and improving survival. Capecitabine is an oral fluoropyrimidine,
    which is converted to 5 FU in tumor cells. It’s
    role is still being defined in national clinical
    trials. Radiation therapy is not offered in stage III
    colon cancer as local failure is rare because adequate margins can be obtained.
379
Q

A 43-year-old woman undergoes investigation for
colitis. In her history, it is noted that 20 years earlier
she underwent a surgical procedure on the large
intestine.
18. The diagnosis is more likely to be Crohn’s disease rather than ulcerative colitis because the
previous operation was which of the following?
(A) Performed in a young patient
(B) Confined to the colon
(C) Followed by improvement after bypass
of the diseased segment
(D) Followed by improvement because
steroids were prescribed
(E) Grohn’s disease is more premaligent
than ulcerative cohitis

A
  1. (C) Crohn’s disease differs from ulcerative colitis in that clinical improvement usually occurs
    when a diseased segment is excluded from the
    fecal stream. Crohn’s disease involves the distal
    ileum in most patients, but almost any part of the
    alimentary tract could be affected. Steroids frequently result in improvement in patients with
    Crohn’s disease and ulcerative colitis. In Crohn’s
    disease, steroids are a double-edged sword,
    because they clearly allow initial improvement,
    but eventually their benefit is counteracted by
    adverse complications of steroids.
380
Q

A 43-year-old woman undergoes investigation for
colitis. In her history, it is noted that 20 years earlier
she underwent a surgical procedure on the large
intestine.
19. Is the diagnosis more likely to be ulcerative
colitis rather than Crohn’s disease because at
the previous operation?
(A) All layers of the bowel wall were involved
(B) There was evidence of fistula formation
(C) The serosa appeared normal on
inspection, but the colon mucosa was
extensively involved
(D) Skip lesions were noted
(E) The preoperative GI series showed a
narrowing string like stricture in the
ileum (string sign)

A
  1. (C) The serosa appeared normal on inspection,
    but the colon mucosa was extensively involved.
    In ulcerative colitis, the distal rectum and colon
    are primarily involved in continuity to the proximal extent of the lesion. In Crohn’s disease, a
    similar pattern may be found on rare occasions,
    but other features, such as small intestinal disease, transmural involvement, skip lesions, and
    fistula formation, favor Crohn’s disease. The
    small bowel is not primarily involved in ulcerative colitis, but a “backwash” ileitis may be
    encountered.
381
Q
  1. A 68-year-old retired female plastic surgeon
    underwent laparotomy through a midline
    abdominal incision. Intestinal infarction was found and a distal 60% small-bowel resection
    was performed with ileocecal anastomosis. She
    was placed on hyperalimentation. Seven days
    after the operation, she underwent a second
    operation through the same incision. Wound
    healing is further impaired by which of the
    following?
    (A) Incision through the same abdominal
    wall scar
    (B) Vitamin A administration
    (C) Zinc deficiency
    (D) Increased local oxygen tension
    (E) Incision through new area of abdominal
    wall
A
  1. (C) Both zinc and vitamin C (ascarbate) deficiency, impair wound healing. Vitamin A
    deficiency is also implicated in would healing and supplemental Vitamin A has been
    shown in experimental studies to prevent radiation included defects in wound healing.
    Incision through the same abdominal wall scar
    incision actually promotes wound healing,
    because the initial lag interval after creation of
    the wound is avoided (unless the whole scar of
    the incision is removed). Increase in local oxygen
    tension actually promotes wound healing.
382
Q
  1. A 68-year-old retired female plastic surgeon
    underwent laparotomy through a midline
    abdominal incision. Intestinal infarction was found and a distal 60% small-bowel resection
    was performed with ileocecal anastomosis. She
    was placed on hyperalimentation. Seven days
    after the operation, she underwent a second
    operation through the same incision. At the second operation an advanced carcinoma
    of the colon is detected. What factors would
    cause wound healing to be further impaired?
    (A) Doxorubicin is given.
    (B) Denervation of bowel or skin incision
    occur.
    (C) Mechanical lavage and oral antibiotics
    are given before surgery.
    (D) Steroids are not given.
    (E) Leavage with polyethylene glycol
    solution.
A
  1. (A) Doxorubicin cleaves diribonucleic acid
    (DNA) and has been shown to decrease wound
    healing. Treatment should be delayed at least
    4 weeks. Wound healing will improve by reducing wound infection rates. This is the rational for
    the use of antibiotic prophylaxis. The use of
    mechanical cleansing alone will not reduce
    wound infection and may actually increase
    complications. Mechanical preparation with oral
    nonabsorbable antibiotics does reduce microbial flora and has been shown to reduce surgical
    infectious complications. Denervation of tissue
    surrounding the incision does not influence
    wound healing. Steroids delay the rate of
    wound healing and decrease protein synthesis.
383
Q

A 74-year-old patient has a biopsy of the prostate
that shows malignancy. He is considering radical
prostatectomy or radiation therapy.

45. He is concerned about enterocolitis, which is
likely to occur when?
(A) After local treatment with 15 Gy
(B) After local treatment with 35 Gy
(C) After local treatment with 55 Gy
(D) Less frequently after previous surgery
(E) Less frequently in the presence of
adhesions
A
  1. (C) Irradiation of the abdominal cavity of more
    than 50 Gy is associated with a higher rate of complications. The incidence of symptomatic sigmoiditis may be as high as 75%, and histologically
    abnormal rectal biopsy findings occur in 11% of
    patients undergoing treatment for pelvic malignancy. Previous surgery with possible adhesion
    formation increases the risk of irradiation damage.
384
Q

A 74-year-old patient has a biopsy of the prostate
that shows malignancy. He is considering radical
prostatectomy or radiation therapy.

46. What complication should be anticipated in
this patient?
(A) Diverticulitis
(B) Hemorrhoids
(C) Complete occlusion of superior
mesenteric artery
(D) Complete occlusion of inferior
mesenteric artery
(E) Rectal bleeding
A
  1. (E) In most patients, ischemic colitis is a selflimiting illness that usually resolves within
    7–10 days. Patients may manifest pyrexia and
    peritonitis, have persistent symptoms, and
    develop complications, such as stricture formation, perforation, and bleeding. Unlike
    small-bowel ischemia, the main vessels are
    characteristically patent.
385
Q
  1. A 49-year-old computer technician receives
    irradiation to the pelvis for cervical cancer.
    Three months after irradiation, severe rectal
    proctitis may be shown by the presence of
    which of the following?
    (A) Ulcers
    (B) Strictures at anal verge
    (C) Mucosa prolapse
    (D) Multiple telangiectasis and polypoid
    tumor
    (E) Free air under the diaphragm
A
  1. (A) The mucosa is friable and bleeds readily.
    Ulcers vary in size and often tend to be transverse in position and surrounded by telangiectasis. They are often more prominent on the
    anterior wall around the anal verge. Rectal
    strictures usually are located about 8–12 cm
    above the anal verge. Rectovaginal fistula may
    develop in female patients. On barium enema,
    a narrow stricture is difficult to differentiate
    from a carcinoma.
386
Q

A 63-year-old man is admitted to the hospital for
abdominal pain and diarrhea of 6-day duration. Xray of the abdomen shows “thumbprinting” and
gaseous distention suggestive of ischemic colitis.

  1. What is true of colonic ischemia?
    (A) It occurs in a younger age group (40–60
    years of age).
    (B) In most cases, it occurs in patients with
    cardiac failure.
    (C) It usually causes severe abdominal pain.
    (D) It may have a predisposing associated
    colonic lesion in 20% of patients.
    (E) It results in the patient’s appearing seriously ill.
A
  1. (D) In 90% of cases with colonic ischemia, the
    patient is over 65 years of age. Precipitating
    causes, such as cardiac disease, are much less
    frequently encountered than in small-bowel
    ischemia. In 20% of patients, an underlying
    obstructive lesion of the colon is noted. Unlike
    small-bowel ischemia, the pain is often insidious in onset.
387
Q

A 63-year-old man is admitted to the hospital for
abdominal pain and diarrhea of 6-day duration. Xray of the abdomen shows “thumbprinting” and
gaseous distention suggestive of ischemic colitis.

49. To confirm the diagnosis of ischemic colitis,
what test should be requested?
(A) Selective angiogram of inferior mesenteric artery
(B) Angiogram of superior and inferior
mesenteric arteries
(C) CT scan of the abdomen
(D) Barium enema after 2 weeks
(E) Barium enema as soon as possible
A
  1. (E) The classic finding of thumbprinting may
    be missed if the barium enema study is
    deferred for more than 10 days after onset of
    symptoms. Unlike small-bowel ischemia, the
    main vessels are patent in most cases.
388
Q
  1. A 54-year-old man with diarrhea is found to
    have ulcerative colitis. Colectomy should be
    advised in patients with ulcerative colitis who
    have symptoms that persist for more than
    which of the following?
    (A) 1 month
    (B) 6 months
    (C) 1–5 years
    (D) 10–20 years
    (E) More than 25 years
A
  1. (D) After 10 years with ulcerative colitis, the
    chances of developing carcinoma increase fourfold. After 20 years, the cumulative risk is 12%,
    and at 25 years, it is 25%. Malignancy is often
    detected at a late stage and has a larger percentage of synchronous lesions as compared to
    that seen in patients with cancer who do not
    have ulcerative colitis. Patients with extensive
    disease and those in whom the disease occurs at
    an earlier age must undergo careful surveillance.
389
Q
  1. A 48-year-old woman develops colon cancer.
    She is known to have a long history of ulcerative
    colitis. In ulcerative colitis, which of the following is a characteristic of colon cancer?
    (A) Occurs more frequently than in the rest
    of the population.
    (B) Is more likely to occur when the ulcerative disease is confined to the left colon.
    (C) Occurs equally in the right and left side.
    (D) Has a synchronous carcinoma in 4–5%
    of cases.
    (E) Has an excellent prognosis because of
    physician awareness.
A
  1. (A) Occurs more frequently than in the rest of the
    population. The cumulative risk of developing
    cancer in patients with extensive ulcerative colitis is greater than in those with more localized
    disease (42% at 25 years). Children are more
    likely to have extensive disease. Colon cancer
    occurs more frequently in the sigmoid and
    rectum in ulcerative colitis, but cancer is more
    likely to occur in patients who have universal
    disease. Synchronous carcinomas in patients
    without ulcerative colitis occur in 4%, compared
    to 25% in those with colitis. Lesions usually are
    flat, are frequently missed at examination, and
    have a worse prognosis than sporadic colon cancers found in normal risk patients. Adults developing cancer under the age of 45 have a poorer
    prognosis than those who develop it later.
390
Q
  1. A 64-year-old train conductor is diagnosed as
    having carcinoma confined to the descending
    colon. Before operation, what should be told?
    (A) He will most likely require a colostomy.
    (B) He should have the cancer excised by
    cautery.
    (C) He should undergo left hemicolectomy.
    (D) Radiotherapy is the treatment of choice.
    (E) 40% of colorectal cancer involves the
    colon.
A
  1. (C) There has been an increase in incidence of
    colon cancer relative to that of the rectum in
    recent years. This observation may be related to
    the improved diagnostic techniques now available with colonoscopy. The higher mortality of
    some rectal cancer patients may be attributed to
    an incomplete resection of the tumor when it is
    close to the cut edge. Each year, 14,000 new
    cases are diagnosed and over 6000 deaths occur.
391
Q
  1. A 72-year-old woman is scheduled to undergo
    right hemicolectomy for cancer of the cecum. In
    this condition, she can anticipate subsequent
    recurrence
    (A) Of 20–30% if confined to the mucosa
    (B) Close to 100% if there is lymph node
    involvement
    (C) Which will not result in small-bowel
    obstruction
    (D) Which will not result in hydronephrosis
    (E) Which with microscopic lymph node
    metastasis would have a lower rate than
    that with macroscopic spread
A
  1. (E) Just under half of patients with local disease
    will also have associated metastatic disease.
    Patients with microscopic lymph node
    metastasis—adjacent as opposed to remote—
    and with one to three lymph nodes involved
    have a better prognosis than patients with more
    extensive disease.
392
Q
  1. A pathology specimen indicates that synchronous lesions are present. Which of the following statements are true regarding colon cancer
    with synchronous lesions?
    (A) Cancer occurs in 20% of patients.
    (B) Benign lesions occur in 20–30%.
    (C) Malignant lesions are usually adjacent
    to the primary cancer.
    (D) Benign lesions are usually adjacent to
    the primary cancer.
    (E) Lesions occur much less frequently than
    metachronous lesions.
A
  1. (B) Synchronous malignant lesions (present in
    4–5%) refer to those present at the time of surgery or found in investigations carried out
    within 6 months after operation. Metachronous
    lesions are those not detected during this period
    but subsequently identified. Metachronous carcinomas occur in about 5% of cases.
393
Q

A 68-year-old dentist undergoes anterior resection
(sigmoid resection) for cancer at the rectosigmoid
junction. The tests performed before her surgery were
colonscopy and biopsy. There were no other lesions
detected with sigmoidoscopy or in the pathology
specimen.
55. Following operation, she requires which of the
following within 2–3 months?
(A) Repeat rectal examination and
sigmoidoscopy
(B) Colonoscopy
(C) CT scan of the abdomen
(D) Angiography
(E) Bone scan

A
  1. (B) Synchronous carcinoma and polyps, of all
    types, occur at sites in the colon not included in
    an anterior or sigmoid resection. Both synchronous carcinomas and benign polyps occur
    mainly at sites in the colon that would not be
    included in the definite resection for the primary carcinoma. Thus, it is important to try,
    whenever possible, to perform colonoscopy
    before colon resection to facilitate planning of
    the operation should a synchronous lesion be
    detected. If this study is omitted, it is advisable
    to have a complete colonoscopy performed
    within the first 2–3 months after resection.
394
Q

A 68-year-old dentist undergoes anterior resection
(sigmoid resection) for cancer at the rectosigmoid
junction. The tests performed before her surgery were
colonscopy and biopsy. There were no other lesions
detected with sigmoidoscopy or in the pathology
specimen.
56. The patient requests information from her surgeon as to her subsequent prognosis. She is
informed that the prognosis for colon and rectal
cancer is favorably affected by which of the
following?
(A) Minimal serosal extension
(B) Minimal lymph node involvement
(C) Confinement to the mucosa
(D) Right-sided obstructing lesions
(E) Elevated carcinoembryonic antigen
(CEA) levels

A
  1. (C) Dukes A lesions have an excellent prognosis
    of 90% 5-year survival compared to that with
    serosal extension (B2), particularly if lymph nodes
    are heavily involved. Around 70 % of obstructing
    lesions occur on the left side and 30% proximal to
    the hepatic flexure. The CEAlevel correlates with
    the extent of encirclement of the tumor, Dukes
    classification, and the likelihood of recurrence.
395
Q
  1. An 83-year-old man is diagnosed on colonoscopy to have cancer of the colon. He refuses
    surgical intervention and after a 3-month followup period is admitted to the emergency department with large-bowel obstruction. Carcinoma of
    the colon is most likely to obstruct if found in the
    (A) Cecum
    (B) Ascending colon
    (C) Descending colon
    (D) Rectum
    (E) Transverse colon
A
57. (C) The most common sites of obstruction are
descending colon (21%), sigmoid (17%), and
splenic flexure (15%). The percentages for cases
with obstruction at a particular site are splenic
flexure, 37%; sigmoid, 16%; and right colon, 14%.
396
Q
  1. A43-year-old man is seen in his physician’s office
    for severe pain in the perineum. Examination
    reveals exquisite tenderness in the area to the
    right side of the anal verge due to a perianal
    abscess. Rectal examination is refused. What
    should be the next step in management?
    (A) Drainage of the abscess in the office
    under local anesthesia.
    (B) Excision of the vertical fold of Morgagni.
    (C) Drainage under general anesthesia and
    immediate colonoscopy.
    (D) CT scan of the abdomen.
    (E) Insertion of a rectal tube.
A
  1. (A) The ducts of the anal glands drain into the
    anus and are covered by the vertical columns of
    Morgagni. Infection of these glands may account
    for some cases of perianal abscess. The folds
    end distally at about the level of the dentate line.
    The lower third of the anus receives its nerve
    supply from the pudendal nerve (somatic). In
    order to minimize spread of infection, the local
    anesthetic should be confined to the skin immediately overlying the abscess. This should be
    performed in a hospital setting, in an operating
    room, with good lighting, in the lithotomy position, using a combination of IV sedation andlocal anesthesia. Protoscopy/sigmoidoscopy can
    be undertaken at he same time.
397
Q
  1. A 64-year-old man undergoes CEA surveillance
    for cancer, because his brother and father both
    had colon cancer. What information should he
    be provided?
    (A) CEA is highly sensitive for diagnosis.
    (B) If CEA is elevated preoperatively, it
    implies unresectable disease.
    (C) Increases in CEA after resection may
    indicate tumor recurrence.
    (D) CEA is highly specific for the presence
    of colon cancer.
    (E) CEA is present in normal adult colonic
    mucosa.
A
  1. (C) CEA is useful in the follow-up care of
    patients with colon carcinoma after resection.
    The levels of this antigen usually come to
    normal after complete resection of the tumor. A
    subsequent elevation may suggest a recurrence
    of the tumor either at the resection margin or at
    distant sites. The sensitivity and specificity of
    CEA for diagnosis of colon carcinoma is poor. It
    has no implications for resectability of the lesion.
398
Q
  1. A 70-year-old man presents with pallor and
    breathlessness on exertion. He does not complain of abdominal pain. He has microcytic,
    hypochromic anemia. What is the most probable cause?
    (A) Diverticulosis of the colon
    (B) Peptic ulcer disease
    (C) Crohn’s disease
    (D) Ulcerative colitis
    (E) Carcinoma of the right colon
A
  1. (E) Insidious development of a microcytic,
    hypochromic anemia is an important clue for
    the diagnosis of carcinoma of the right colon.
    Guaiac-positive stool with or without a palpable mass in the RLQ should raise the possibility. All the other possibilities listed may also
    cause lower GI bleeding but are characteristically associated with abdominal pain (peptic
    ulcer disease, Crohn’s disease, ulcerative colitis). Bleeding in sigmoid diverticulosis usually is bright red and painless.
399
Q
  1. A 25-year-old man has recurrent, indolent fistula
    in ano. He also complains of weight loss, recurrent attacks of diarrhea with blood mixed in the
    stool, and tenesmus. Proctoscopy revealed a
    healthy, normal-appearing rectum. What is the
    most likely diagnosis?
    (A) Crohn’s colitis
    (B) Ulcerative colitis
    (C) Amoebic colitis
    (D) Ischemic colitis
    (E) Colitis associated with acquired immunodeficiency syndrome (AIDS)
A
  1. (A) Recurrent fistulas in ano are a feature of
    Crohn’s colitis. The absence in the rectum
    eliminates the possibility of ulcerative colitis.
    Amebic colitis presents with recurrent episodes
    of diarrhea with bleeding. Ischemic colitis also
    presents with diarrhea.
400
Q
  1. A 65-year-old man presents with chronic constipation and abdominal distention of 5-day duration. He complains of lack of appetite and general
    malaise. Findings on physical examination are
    positive for a large distended abdomen with
    hyperactive bowel sounds. Rectal examination
    shows minimal stool that is guaiac-positive.
    Sigmoidoscopy does not reveal any further findings. Abdominal x-rays show a large 10-cm
    cecum and dilated, fluid-filled transverse and
    descending colon with very little gas in the
    rectum. What is the most probable cause of this
    condition?
    (A) Volvulus of the sigmoid colon
    (B) Pseudo-obstruction of the colon
    (C) Ischemic colitis
    (D) Carcinoma of the colon
    (E) Diverticulitis of the colon
A
  1. (D) The picture described suggests large-bowel
    obstruction in a patient with a competent ileocecal valve. The most likely cause is an obstructing
    carcinoma. The site of obstruction is in the sigmoid
    colon above the level of sigmoidoscopy. Sigmoid
    volvulus, ischemic colitis, and diverticulitis will
    present some findings on sigmoidoscopy. Pseudoobstruction of the colon will manifest as colonic
    distention down to the rectum (Fig. 6–5).
401
Q
  1. A 27-year-old homosexual male presents with
    a foreign body in the rectum. During the extraction of the foreign body, a large tear in the sigmoid colon with extensive devitalization and
    contamination is observed. What is the preferred method of treatment?
    (A) Observation
    (B) Proctoscopic repair
    (C) Laparotomy and closure of sigmoid
    colon tear
    (D) Laparotomy, closure of sigmoid, and
    proximal colostomy or exteriorization of
    perforation as a colostomy
    (E) Laparotomy, resection of sigmoid colon,
    and colostomy
A
  1. (D) Rectosigmoid injuries should promptly raise
    a high index of suspicion, warranting immediate sigmoidoscopy to confirm the diagnosis.
    Sigmoidoscopy, rigid or flexible, involves much
    manipulation and insufflation of air. This is
    hardly desirable or safe in the presence of a
    significant tear in the presence of a rectal foreign
    body, free air under the diaphragm, in a patient
    with an acute abdomen, is all that is necessary to
    warrant laparotomy. Following this, CT scan
    with gastrofin administered orally, will give the
    diagnosis. The best treatment is exteriorisation
    that is colostomy, at the perforated site. This will
    depend upon the location and extent of the perforation. If small and localized, colostomy at the
    site or proximally may be chosen. If the tear is
    massive, then resection with proximal colostomy
    and mucous fistula (Hartman) may be indicated.
402
Q
  1. A 65-year-old woman with a history of chronic
    constipation is transferred from a nursing
    home because of abdominal pain and marked
    abdominal distention. On examination, her
    abdomen is found to be distended and tender
    in the LLQ. What is the most likely diagnosis?
    (A) Appendicitis
    (B) Carcinoma of the colon
    (C) Volvulus of the sigmoid colon
    (D) Volvulus of the cecum
    (E) Small-bowel obstruction
A
  1. (C) Volvulus of the sigmoid (secondary type) is
    common in elderly patients who are chronically
    constipated. Redundancy of the sigmoid and a
    narrow mesenteric attachment predispose for
    the twisting. In the large bowel, the sigmoid is
    the most common site. Abdominal distention
    and tenderness are the common presenting
    symptoms. Volvulus of the sigmoid colon can
    usually be detected on a supine and erect
    abdominal x-ray. Sigmoidoscopy and contrast
    barium studies may be helpful to differentiate
    carcinoma from volvulus (Fig. 6–6).
403
Q
  1. A 40-year-old man with a long history of bloody
    diarrhea presents with increased abdominal
    pain, vomiting, and fever. On examination, he
    is found to be dehydrated and shows tachycardia and hypotension. The abdomen is markedly
    tender with guarding and rigidity. What is the
    most likely cause?
    (A) Toxic megacolon in ulcerative colitis
    (B) Small-bowel perforation from regional
    enteritis
    (C) Perforated carcinoma of the sigmoid
    colon
    (D) Volvulus of the sigmoid colon
    (E) Acute perforated diverticulitis
A
  1. (A) The long history of bloody diarrhea should
    suggest a diagnosis of inflammatory bowel disease. The acute onset of abdominal pain together
    with the findings of an acute abdomen and systemic manifestations should raise the suspicion
    of a devastating complication. The picture is
    characteristic of acute toxic megacolon in ulcerative colitis. All the other possibilities listed may
    present with an acute abdomen, but the long
    history should point to ulcerative colitis.
404
Q
66. Three days after undergoing an operation for an
abdominal aortic aneurysm, a patient has moderate fever, abdominal pain, and rectal bleeding.
What is the most helpful investigation?
(A) Angiography
(B) Upper GI endoscopy
(C) Abdominal ultrasound
(D) Sigmoidoscopy
(E) Abdominal CT scan
A
  1. (D) In a patient with abdominal aortic aneurysm
    resection, the most worrisome complication is
    inadequate blood supply to the sigmoid colon
    through the marginal artery. Sigmoid ischemia
    should be ruled out by sigmoidoscopy. In the
    clinical picture described, sigmoidoscopy should
    be the most important test.
405
Q
  1. A 55-year-old woman presents with pain in the
    LLQ of the abdomen and fever of 102°F. On
    examination, she is found to be dehydrated
    and has tenderness in the LLQ. A CT scan
    shows a mass in the LLQ involving the sigmoid colon. There is a minimal amount of free
    fluid and no free air. What should the initial
    treatment of this patient include?
    (A) IV fluids, penicillin, and steroids
    (B) IV fluids, cefoxitin, and nasogastric
    drainage
    (C) IV fluids, blood transfusion, and
    laparotomy
    (D) immediate laparotomy
    (E) bowel preparation followed by
    laparotomy
A
  1. (B) The findings described on physical examination and CT scan are suggestive of acute
    diverticulitis of the sigmoid colon. The initial
    treatment of this condition is expectant with
    antibiotics with or without nasogastric
    drainage. An antibiotic with specificity against
    the Bacteroides species (third-generation
    cephalosporin, metronidazole, or clindamycin)
    should be part of the regimen. Steroids have no
    place in the treatment. Laparotomy is indicated
    only after failure of conservative treatment.
406
Q

A 72-year-old woman presents with bright red rectal
bleeding, not associated with abdominal pain, of
2-day duration. She had previous similar episodes
but was never hospitalized. Examination reveals a
pale but alert individual with no significant abdominal findings. Findings on rectal examination are positive for bright red rectal bleeding. Her vital signs
are stable and her hemoglobin is 9.5 g.
68. What is the most probable cause of her bleeding?
(A) Diverticulitis of the colon
(B) Carcinoma of the sigmoid colon
(C) Meckel’s diverticulitis
(D) Adenomatous polyp of the colon
(E) Diverticulosis of the colon

A
  1. (E) The clinical picture of recurrent bright rectal
    bleeding that is not associated with abdominal
    pain is characteristic of diverticulosis of the colon.
    The bleeding in sigmoid carcinoma is often
    microscopic. Diverticulitis of the colon would
    present with associated pain. Adenomatous
    polyp may present with painless rectal bleeding,
    but the most common condition in this elderly
    age group is diverticulosis of the colon.
407
Q

A 72-year-old woman presents with bright red rectal
bleeding, not associated with abdominal pain, of
2-day duration. She had previous similar episodes
but was never hospitalized. Examination reveals a
pale but alert individual with no significant abdominal findings. Findings on rectal examination are positive for bright red rectal bleeding. Her vital signs
are stable and her hemoglobin is 9.5 g.
69. The patient continues to bleed per rectum and
becomes hypotensive to a systolic pressure of
60 mm Hg despite blood transfusion. What is
the optimal management plan?
(A) Emergency colonoscopy and
cauterization of bleeding vessels
(B) Mesenteric angiography and
embolization of the bleeder
(C) Bleeding scan to localize the bleeder
(D) Laparotomy and right colon resection
(E) Blood transfusion laparotomy and
subtotal colectomy with or without
ileoproctostomy

A
  1. (E) Laparotomy and subtotal colectomy should
    be the preferred approach in a hypotensive
    patient. There is no time for trying to localize
    the site of bleeding by scans, mesenteric angiography, or colonoscopy. Although the common
    site of massive diverticular hemorrhage is the
    right colon, a blind right colon resection in an
    elderly woman with hypotension is fraught with
    the danger of recurrent bleeding from the left
    colon. The safest and most expeditious management is subtotal colectomy. The decision for anastomosis or proximal ileostomy will depend on
    the stability of the patient.
408
Q
  1. A 60-year-old man complains of recurrent
    attacks of painless rectal bleeding. Colonoscopy
    reveals normal mucosa between the cecum and
    the anal verge. What is the most helpful test to
    determine the cause of bleeding?
    (A) Angiography to look for angiodysplasia
    (B) Technetium scan for Meckel’s
    diverticulum
    (C) Upper GI endoscopy for peptic ulcer
    (D) Small-bowel series for tumor
    (E) Ultrasound for abdominal aortic
    aneurysm
A
  1. (A) A common cause of lower GI bleeding that
    is recurrent and painless is angiodysplasia of
    the colon. In the absence of diverticula or hemorrhoids, the suspicion is even higher for these
    lesions. Peptic ulcer and Meckel’s diverticulum can cause predominantly lower GI bleeding. However, the bleeding is usually in the
    form of melena rather than bright red.
409
Q
71. The small intestine is characterized by basal
crypts and superficial villi (Fig. 6–2). Where
does cell division take place?
(A) Submucosa
(B) Crypts
(C) Villi
(D) Small-bowel lumen
(E) Lamina propria
A
  1. (B) Small-bowel turnover can be measured in rats by autoradiographic studies in which
    turnover of cells located in the crypts migrate
    along the villus toward the tip over a 2- to 3-
    day period. Intestinal villous mucosa undergoes hypertrophy and hyperplasia whenever
    an increased food load continuously enters the
    small intestine.
410
Q
  1. A 64-year-old man has a benign lesion of the
    colon. He is informed that the lesion does not
    predispose to colon cancer. What is the lesion
    he has?
    (A) Ulcerative colitis
    (B) Villous adenoma
    (C) Hyperplastic polyp
    (D) Adenoma in familial polyposis
    (E) Colon mucosa in a patient with colon
    carcinoma
A
  1. (C) All the choices listed except hyperplastic
    polyp are precancerous lesions. The carcinomas in ulcerative colitis and familial polyposis
    are multicentric. Large villous adenomas may
    have carcinomatous changes. Any patient with
    a colon carcinoma is predisposed to develop a
    metachronous lesion in the remaining colon, hence the importance of regular follow-up
    examinations in these patients.
411
Q
  1. A 25-year-old man complains of rectal bleeding, weight loss, and abdominal pain. He gives
    a history of similar complaints in his siblings as
    well as his mother. Findings on physical examination are unremarkable except for guaiacpositive stool. What is the most likely diagnosis?
    (A) Peutz–Jegher syndrome
    (B) Familial polyposis of the colon
    (C) Ulcerative colitis
    (D) Carcinoma of the stomach
    (E) Crohn’s colitis
A
  1. (B) All the clinical features mentioned and the
    strong family history should raise the possibility
    of familial polyposis. Although other possibilities
    listed may also cause rectal bleeding and abdominal pain, the strong familial history should give
    a clue to the diagnosis. The early onset of invasive carcinoma in these patients makes recognizing familial polyposis very important.
412
Q
  1. A 55-year-old man has had previous hemicolectomy for a carcinoma of the right colon. At this
    time, 3 years after the primary resection, a CT
    scan shows a solitary lesion in the right lobe of
    the liver. What is the next step in management?
    (A) Laser cauterization
    (B) Radiotherapy
    (C) Hepatic artery catheterization and local
    chemotherapy
    (D) Symptomatic treatment with analgesics,
    because the colon disease is now stage IV
    (E) Exploratory laparotomy and resection of
    the tumor
A
  1. (E) Many patients who have metastasis to the
    liver or lung have resectable tumors. A reasonable disease-free interval has been reported
    after such resections, especially with carcinoma
    of the colon as the primary lesion.
413
Q
  1. Following an appendectomy, a 28-year-old man
    is placed on ceftizoxime sodium (Cefizox). This
    antibiotic is unlikely to be effective against
    which of the following?
    (A) Pseudomonas
    (B) Staphylococcus aureus
    (C) Neisseria gonorrhoeae
    (D) Bacteroides fragilis
    (E) Haemophilus influenza
A
  1. (A) Cefizox is not effective against many
    strains of Pseudomonas. If the drug is used in
    pseudomonas infection a higher dosage may
    be indicated, and the antibiotic should be
    changed if a quick response does not occur.
    Complications include cross reactions in
    patients who are allergic to penicillin. It does
    not seem to have nephrotoxic side effects.
414
Q
  1. A 68-year-old man presents with crampy
    abdominal pain and distention with vomiting.
    Findings on physical examination are positive
    for healed abdominal scars. X-rays reveal multiple gas fluid levels. The WBC count is 12,000.
    What is the most likely diagnosis?
    (A) Small-bowel intestinal obstruction due
    to adhesions
    (B) Hernia
    (C) Appendicitis
    (D) Inflammatory bowel disease
    (E) Gallstones and ascites
A
  1. (A) The presence of distended loops of bowel
    indicate bowel obstruction. The clinical features favor mechanical obstruction rather than
    paralytic ileus due to infection. Obstruction
    due to adhesions is more common than
    obstruction due to hernia.
415
Q
  1. A 55-year-old woman presents with vague
    RLQ abdominal pain. A palpable mass is noted
    on abdominal examination. The mass is painless, well defined, mobile, and nonpulsatile.
    What is the most likely diagnosis?
    (A) A mesenteric cyst
    (B) Appendix mass
    (C) Perforated tubo-ovarian abscess
    (D) Cholecystitis
    (E) Meckel’s diverticulum
A
  1. (A) This is a relatively uncommon lesion. One
    sign that may be elicited with a mesenteric cyst
    is that the swelling moves freely in the direction
    between the left iliac fossa and the right
    hypochondria (i.e., perpendicular to the smallbowel mesentery axis) (Fig. 6–7)
416
Q
78. A 74-year-old woman complains of vomiting
and intermittent colicky abdominal pain. Xrays reveal fluid levels and air in the biliary
tree. What is the likely cause?
(A) Abdominal adhesions
(B) Gallstone ileus
(C) Carcinoma of the right colon
(D) Abdominal lymphosarcoma
(E) Previous choledochoduodenostomy
A
  1. (B) Gallstone ileus results in “tumbling” intestinal obstruction due to the intermittent nature
    of the condition. Previous choledochoduodenostomy could give air in the biliary tree but
    not obstruction.
417
Q
  1. Following resection of the left colon, a 67-
    year-old obese woman develops left-sided leg
    edema due to deep-vein thrombosis. She is
    placed on anticoagulants, but after 2 weeks of
    warfarin (Coumadin), she develops a pulmonary embolus with slight hypoxemia.
    What should the next step in management
    involve?
    (A) Increasing the dose of anticoagulants
    (B) Discontinuing anticoagulants
    (C) Use of an inferior vena cava (IVC)
    filter
    (D) CT scan of the leg and abdomen
    (E) Femoral vein ligation
A
  1. (C) In general, failure (or inability) to continue
    anticoagulants is an indication to insert an IVC
    filter to minimize the possibility of serious and
    possibly fatal pulmonary embolus.
418
Q
Questions 91 and 92
(A) Vitamin A
(B) Vitamin C
(C) Vitamin D
(D) Vitamin E
(E) Vitamin K
(F) Vitamin B1
(G) Chyle
(H) Sympathetic denervation
(I) Failure of rectal muscles to contract
(J) Gluten
(K) Peptides
(L) Bile salts
(M) Meissner and Auerbach plexus deficiency
(N) Vagus nerve excess
(O) Inferior mesenteric ischemia
91. Steatorrhea and megaloblastic anemia, occurring in a patient after bowel resection, is caused
by a failure to absorb what? SELECT ONE.
A
  1. (L) The jejunum is the first part of the alimentary
    tract and, therefore, is the primary site of absorption of nearly all nutrients. It is unable to absorb
    vitamin B12 and bile salts, which are absorbed
    exclusively in the ileum. If the ileum is transposed between the duodenum and the jejunum,
    it undergoes compensatory hypertrophy and
    takes over the function of the jejunum and
    becomes the primary site of nutrient absorption.
419
Q
(A) Vitamin A
(B) Vitamin C
(C) Vitamin D
(D) Vitamin E
(E) Vitamin K
(F) Vitamin B1
(G) Chyle
(H) Sympathetic denervation
(I) Failure of rectal muscles to contract
(J) Gluten
(K) Peptides
(L) Bile salts
(M) Meissner and Auerbach plexus deficiency
(N) Vagus nerve excess
(O) Inferior mesenteric ischemia
92. What does Hirschsprung’s disease involve?
SELECT ONE.
A
  1. (M) In Hirschsprung’s disease, there is an
    absence of myenteric plexus in the upper anal segment (i.e., the most distal portion of the
    cloaca). In 15%, the myenteric plexus involves
    only the upper anus; in 70%, the rectum is also
    involved; and in 15%, part of the colon is also
    involved. The abnormal segment is contracted;
    whereas, the dilated bowel is proximal to the
    diseased segment.
420
Q

(A) Spigelian hernia
(B) Direct inguinal hernia
(C) Femoral hernia
(D) Richter’s hernia
(E) Appendix
(F) Hydrocele
(G) Sliding hernia
(H) Bladder
(I) Liver
(J) Seminal vesicle
(K) An adrenal metastasis
(L) Ureter
(M) Prostate
(N) Pubic bone
(O) Cowper’s (bulbourethral) glands
93. An 84-year-old man has had a reducible hernia
in the right groin for 17 years. One day before
admission to the hospital, he complains of
abdominal pain; because of the swelling, the
hernia has become irreducible. At operation, part
of the wall of the cecum is noted to form a portion of the hernia sac. What is the hernia?

A
  1. (G) In this variety, the hernia does not have a
    complete covering of peritoneum. It is called a
    sliding hernia. It is important that the surgeon
    does not attempt to remove peritoneum from
    the circumference bowel wall where it does
    not exist, because the bowel will become
    devascularized.
421
Q
(A) Supralevator space
(B) Perianal space
(C) Levator ani muscle
(D) Intermuscular space
(E) External sphincter
(F) Ischioanal space
(G) Submucous space above the levator ani
muscle
(H) Marginal mucocutaneous space
  1. A 25-year-old patient with a 2-cm painful
    abscess in perianal region for 1 day. The patient
    does not have fever or leukocytosis. Which
    space is this lesion in? SELECT ONE.
A
  1. (B) Perianal abscess is most common type of
    anorectal abscess. It is superficial and lies in
    perianal space. Duration of symptoms is short
    and patient is unlikely to have fever or
    leukocytosis.
422
Q
(A) Supralevator space
(B) Perianal space
(C) Levator ani muscle
(D) Intermuscular space
(E) External sphincter
(F) Ischioanal space
(G) Submucous space above the levator ani
muscle
(H) Marginal mucocutaneous space
  1. A 30-year-old patient presents with a 5-day history of pain to right buttock. A 7-cm firm area is
    noted on the right buttock. Patient also describes
    purulence from rectum and has a temprature
    101°F. In which space is this lesion? SELECT
    ONE.
A
  1. (F) Ischiorectal abscesses are often large, erythematous indurated, and tender. They are
    often associated with fistula.
423
Q
(A) Supralevator space
(B) Perianal space
(C) Levator ani muscle
(D) Intermuscular space
(E) External sphincter
(F) Ischioanal space
(G) Submucous space above the levator ani
muscle
(H) Marginal mucocutaneous space
  1. A 28-year-old woman recently treated as an outpatient for pelvic inflammatory disease presents
    with fever, leukocytosis, and deep rectal pain. In
    which space is this lesion? SELECT ONE.
A
  1. (F) Supralovator abscesses are relatively rare.
    Most patients have a pelvic inflammatory condition such as salpingitis, diverticulitis or
    Crohn’s, or have had recent pelvic surgery.
424
Q
(A) Pilonidal sinus
(B) Posterior perianal sinus
(C) Single anterior perianal sinus
(D) Multiple anterior perianal sinus
(E) Periurethral abscess
(F) Bartholin gland abscess
(G) Prostatic abscess
(H) Rectovaginal fistula
97. Which opens into the anal mucosa in the midline? SELECT ONE.
A
  1. (B) A single or multiple sinuses that has an
    external opening in the posterior half of the
    skin that surrounds the anus will have an internal opening in the midline on the distal anus if
    a fistula has formed (Fig. 6–8).
425
Q
(A) Pilonidal sinus
(B) Posterior perianal sinus
(C) Single anterior perianal sinus
(D) Multiple anterior perianal sinus
(E) Periurethral abscess
(F) Bartholin gland abscess
(G) Prostatic abscess
(H) Rectovaginal fistula
  1. What has hair inside? SELECT ONE.
A
  1. (A) The most common site for a pilonidal
    abscess to develop is in the midline posteriorly
    in the natal cleft posterior to the sacrum.